Top Banner
Monthly Current Affairs of SEPTEMBER February 4 2019 Hey, are you looking for availing a best SSC coaching in Kolkata for passing the exam with top percentile? Then you are here at the very right place at Sen's School of Competitive Examinations(SSCE), your one-stop destination regarding getting best quality SSC coaching. They are indeed a premier name among all the SSC coaching centers in across the city that offers quality training on SSC and other competitive exams by all experienced faculties. Well, the best thing regarding them is that they used to take a very reasonable charge from students for each of the course they are conducting right now.Here you can find Best govt. Jobs Coaching Institutes in Hooghly.Here, one thing you need to keep in mind, and that is getting a government job like SSC and others is not an easy task to accomplish. As for that, you need first to clear the exam and then sit for the interview and thus without a proper training one cannot clear the exam and neither can clear the interview successfully. And when it comes to coaching SSC and other competitive exams Sen's School of Competitive Examinations (SSCE) is your ideal destination where they teach you all from the base level. So let’s check more details regarding the institute in this regard. SSCE
109

Monthly Current Affairs of SEPTEMBER - SSCE

Jan 25, 2023

Download

Documents

Khang Minh
Welcome message from author
This document is posted to help you gain knowledge. Please leave a comment to let me know what you think about it! Share it to your friends and learn new things together.
Transcript
Page 1: Monthly Current Affairs of SEPTEMBER - SSCE

Monthly Current Affairs of SEPTEMBER

February 4

2019 Hey, are you looking for availing a best SSC coaching in Kolkata for passing the exam with top percentile? Then you are here at the very right place at Sen's School of Competitive Examinations(SSCE), your one-stop destination regarding getting best quality SSC coaching. They are indeed a premier name among all the SSC coaching centers in across the city that offers quality training on SSC and other competitive exams by all experienced faculties. Well, the best thing regarding them is that they used to take a very reasonable charge from students for each of the course they are conducting right now.Here you can find Best govt. Jobs Coaching Institutes in Hooghly.Here, one thing you need to keep in mind, and that is getting a government job like SSC and others is not an easy task to accomplish. As for that, you need first to clear the exam and then sit for the interview and thus without a proper training one cannot clear the exam and neither can clear the interview successfully. And when it comes to coaching SSC and other competitive exams Sen's School of Competitive Examinations (SSCE) is your ideal destination where they teach you all from the base level. So let’s check more details regarding the institute in this regard.

SSCE

Page 2: Monthly Current Affairs of SEPTEMBER - SSCE

SSCE 8981426494/8296260082 Downloaded from :- www.onlinessce.com Page 2

SEN’S SCHOOL OF COMPETIVE EXAMINATIONS

SSCE Monthly Current Affairs (January)

Daily Current Affairs 1st January,2019

1) Name the Indian player who became the 1st ever player to take 20 catches in a Test series.

Ans:- Rishabh Pant

Explanation:21 years old Indian wicket-keeper Rishabh Pant became the 1st ever player to take 20 catches

in a Test series by surpassing former cricketers Naren Tamhane and Syed Kirmani who had recorded 19

catches in 1954-55 and 1979-80 respectively. Pant achieved the feat to take 20 catches in just 3 matches

whereas Naren Tamhane and Syed Kimani took 19 catches in 6 and 7 matches respectively. Rishabh Pant

took 11 catches 6 in first innings and 5 in second innings against Australia in the 3rd Test Match to break the

record of fellow wicket-keeper Wriddhiman Saha who took 10 catches in a match.

2) Name the Veteran actor who passed away in Canada at the age of 81.

Ans: Kader Khan

Explanation: Veteran actor-writer Kader Khan passed away in Canada at the age of 81. Kader Khan made

his acting debut in 1973 with Rajesh Khanna's "Daag" and has featured in over 300 films. He wrote

dialogues for over 250 movies. Before becoming an actor he had written dialogues for Randhir Kapoor-Jaya

Bachchan's "Jawani Diwani". As a screenwriter, Khan frequently collaborated with Manmohan Desai and

Prakash Mehra.

3) How many Indian-origin women scientists awarded by Australian government under Superstar of

STEM initiative?

Ans:- 3

Explanation: Three Indian-origin women scientists Onisha Patel, Devika Kamath and Asha Rao are among

the 60 Superstars of STEM (Science, Technology, Engineering & Mathematics) list awarded by Australian

government. Under the initiative, due to gender gap in STEM courses, the government there is planning to

train these scientists for one year to become influencers for female students.

4) Which of the following pension management software was launched by Narendra Modi for

pensioners of telecom department?

Ans:- System for Authority and Management of Pension.

Explanation: Prime Minister Narendra Modi visited Varanasi and Ghazipur in Uttar Pradesh for one day. In

Varanasi, he launched the System for Authority and Management of Pension (SAMPANN) software which

is developed to ease the problems of pensioners.

5) Prime Minister Narendra Modi attended the One District, One Product Regional Summit in

__________.

Ans:-Varanasi.

Explanation: Prime Minister Narendra Modi visited Varanasi and Ghazipur in Uttar Pradesh for one day.

He attended the One District, One Product Regional Summit at Deendayal Hastakala Sankul in Varanasi.

Page 3: Monthly Current Affairs of SEPTEMBER - SSCE

SSCE 8981426494/8296260082 Downloaded from :- www.onlinessce.com Page 3

Here he laid foundation stone for 14new development projects worth around 98 crores. Also, in the event,

loans worth Rs. 2000 crore were disbursed.

6) Name of the Portuguese airline which introduced single-use plastic free flight on 29 December 2018.

Ans:- Hi Fly

Explanation:- Portuguese airline 'Hi Fly' introduced single-use plastic free flight on 29 December 2018.

The flight took off from Lisbon to Natale, Brazil, without a single piece of single-use plastic on board. There

were no plastic cups, plastic silverware, plastic cocktail stirrer, and plastic food containers.

7) SBI Life Insurance join hands with this bank for bancassurance partnership recently.

Ans:- Allahabad Bank

Explanation:- A 'bancassurance' pact was signed between State-owned Allahabad Bank and SBI Life

Insurance to sell the policies of the insurer from over 3,238 branches of the Allahabad Bank. The agreement

is considered as one of the largest Bancassurance partnerships in the country. The agreement was signed in

the presence of Allahabad Bank Managing Director (MD) and Chief Executive Officer (CEO) Mallikarjuna

Rao and SBI Life MD & CEO Sanjeev Nautiyal.

8) Deen Dayal Veterinary University proposed to set up the first goat fertility lab in the state of

__________ under Rastriya Krashi Vikas Yojna at the cost of Rs 22 crore.

Ans:- Uttar Pradesh

Explanation: The Deen Dayal Veterinary University in Uttar Pradesh proposed to set up the first goat

fertility lab in the state under Rastriya Krashi Vikas Yojna at the cost of Rs 22 crore. The University has

entered into a contract with Pradeshik cooperative Dairy Federation for marketing. Rastriya Krashi Vikas

Yojna or National Agriculture Development Programme is a State Plan Scheme of Additional Central

Assistance launched in August 2007 as a part of the 11th Five Year Plan by the Government of India. It aims

to achieve 4% annual growth in agriculture through development of Agriculture and its allied sectors.

9) Which Union Ministry has released a booklet titled "A Handbook for Students on cyber Safety" to

inform the school children about the different aspects of cyber safety?

Ans:- Ministry of Home Affairs

Explanation: he Ministry of Home Affairs, Government of India has released a booklet titled "A Handbook

for Students on cyber Safety" to inform the school children about the different aspects of cyber safety. The

booklet aims to create awareness among citizens especially students about cyber threats and provides details

about the various safeguards that can be adopted by the children in order to protect themselves from cyber

grooming.

10) An exhibition titled 'Dandi Yatra' was inaugurated in __________.

Ans: New Delhi

Explanation: An exhibition titled 'Dandi Yatra' was inaugurated in New Delhi on 29 December 2018. It was

organised by the National Gallery of Modern Art (NGMA). The exhibition is part of the celebration of the

150th birth anniversary of Mahatma Gandhi. The exhibition showcases the unseen drawings of artist

Chhaganlal Jadav and also the masterworks of NGMA's collection of contemporaneous artists.

11) Who was appointed the next Ambassador of India to the Republic of Liberia on 28th December

2018?

Ans:-Y.K. Sailas Thangal

Explanation: Ministry of External Affairs (MEA), Government of India announced that Mr. Y.K. Sailas

Thangal who is the ambassador of the India to the Republic of Cote d' Ivoire has been concurrently

accredited as the next Ambassador of India to the Republic of Liberia with residence in Abidjan. Y. K.

Page 4: Monthly Current Affairs of SEPTEMBER - SSCE

SSCE 8981426494/8296260082 Downloaded from :- www.onlinessce.com Page 4

Thangal, IFS Officer of 1996 batch was appointed as the Ambassador of India to Republic of Cote d' Ivoire

on 14th September 2018. India is represented in Liberia through its embassy in Abidjan (Ivory Coast) and an

active honorary consulate in Capital of Liberia, Monrovia since 1984.

12) Who among the following took the command of the Indian Army's Gajraj Corps on 30 December

2018?

Ans:-Lt Gen Manoj Pande

Explanation: he Gajraj Corps (IV Corps) is a military field formation of the Indian Army, created in 1961.

Lt Gen Manoj Pande took the command of the Indian Army's Gajraj Corps on 30 December 2018. He

replaced Lt Gen Gurpal Singh Sangha. Prior to this, he was the Chief of Staff in Headquarters Southern

Command.

13) Which state government is going to create a spiritual Department

Ans:- Madhya Pradesh

Explanation: The Congress-led Madhya Pradesh Government announced that it is going to create an

Adhyatmik Vibhag (spiritual department) by merging several existing department. The proposed Adhyatmik

Vibhag (Spiritual Department) going to be formed by merging Dharmik Nyas Evam Dharmasv Department

(Religious Trust and Endowment Department), Anand Vibhag (Happiness Department) in addition to the

Directorate of Religious Trust and Endowment, Madhya Pradesh Teerth Evam Mela Pradhikaran and Rajya

Anand Sansthan.Madhya Pradesh had become the first state in the country to create Happiness Department.

14) Which Asian country has recently submitted its 6th National Report (NR6) to the Convention on

Biological Diversity?

Ans: India

Explnation: India submitted its Sixth National Report (NR6) to the Convention on Biological Diversity

(CBD). The report was submitted online to the CBD Secretariat by the Environment Minister, Dr. Harsh

Vardhan, during the inaugural session of the 13th National Meeting of the State Biodiversity Boards (SBBs)

in New Delhi. With this, India is among the first five countries in the world, the first in Asia and the first

among the biodiversity-rich mega diverse countries to have submitted the report. The submission of national

reports is a mandatory obligation on parties to international treaties, including the CBD.

15) High-Level Committee approved an assistance of Rs 1,146.12 crore from the National Disaster

Response Fund (NDRF) to this state, recently.

Ans:-Tamil Nadu

Explanation: The High-Level Committee (HLC) chaired by Union Home Minister Rajnath Singh approved

additional assistance of Rs 1,146.12 crore from the National Disaster Response Fund (NDRF) to Tamil

Nadu, which was affected by cyclone 'Gaja' recently. The Central government had released assistance of Rs

353.70 crore from the State Disaster Response Fund (SDRF) as an interim relief to the state. Union Finance

Minister Arun Jaitley, Union Minister for Agriculture Radha Mohan Singh and Vice Chairman of NITI

Aayog Dr Rajiv Kumar are the members of the HLC.

Daily Current Affairs 2nd January, 2019

1) DRDO day is observed on :

Ans:- 1st January

Explanation: Defence Research and Development Organisation (DRDO) day is observed on 1st January.

The DRDO was established in 1958 with just 10 laboratories to enhance the research work in defence sector.

Dr. G Satheesh Reddy is the Chairman of DRDO. DRDO works under the Department of Defence Research

and Development of Ministry of Defence.

2) Who has been appointed as the new chairman of Railway Board?

Page 5: Monthly Current Affairs of SEPTEMBER - SSCE

SSCE 8981426494/8296260082 Downloaded from :- www.onlinessce.com Page 5

Ans:- V K Yadav

Explanation: General Manager, South Central Railway, V K Yadav has been appointed the national

transporter's senior-most bureaucrat by the Appointments Committee of the Cabinet. Yadav belongs to the

1980 batch of the Indian Railway Service of Electrical Engineers.

3) How many new Information Commissioners are appointed by Ram Nath Kovind?

Ans:-4

Explanation: Sudhir Bhargava, an information commissioner in the Central Information Commission as the

new chief information commissioner. The government has also appointed four new information

commissioners. President Ram Nath Kovind has approved the appointment of former bureaucrats

Yashwardhan Kumar Sinha, Vanaja N Sarna, Neeraj Kumar Gupta and Suresh Chandra as the new

information commissioners. The appointments came weeks after the Supreme Court asked the Centre to

disclose the process of appointment of the members of the Central Information Commission, including

details on applicants and search committees.

4) Which two countries quits UNESCO, recently?

Ans:-United States and Israel

Explanation: The United States and Israel officially quit the United Nations Educational, Scientific and

Cultural Organization (UNESCO) with the close of 2018. The countries had announced their decisions in

2017 of withdrawing from the agency, accusing it of bias against Israel. America has demanded fundamental

reform in UNESCO, which was co-founded by it after World War II to foster peace. Israel joined the agency

in 1949. It is best known for its World Heritage program, which designates cultural sites and aims to

preserve traditions.

5) Who will be honoured with the National Nritya Shiromani award?

Ans:- Anindita Neogy Anaam

Explanation: US-based Kathak dancer Anindita Neogy Anaam will be honoured with the National Nritya

Shiromani award for promoting the dance form across the world. This award of national repute has been

given in the past to the outstanding Indian classical dance performers and musicians of India and abroad.

The award would be conferred on her in Cuttack on the opening day of the 10th Cuttack Mahotsav:

International Dance and Music festival in Cuttack on 2nd January, 2019.

6) Delhi High Court has allowed the Transgenders to file sexual harassment case under which section?

Ans:- Section 354A

Explanation: Delhi High Court has allowed that Section 354A of Indian Penal Code (IPC) can be used by

transgenders to register complaints of sexual harassment. According to the police, there was a lack of an

appropriate penal section under which they could register such a complaint.

7) Jair Bolsonaro was sworn in as the president of this country recently.

Ans:-Brazil

Explanation:- Jair Bolsonaro was sworn in as Brazil's President in a ceremony at Brazil's National Congress

Building. Bolsonaro is a former Army captain and admirer of the country's 1964-1985 military dictatorship.

Bolsonaro, 63, was a seven-term fringe congressman who rode a wave of anti-establishment anger to

became Brazil's first far-right president since a military dictatorship gave way to civilian rule three decades

ago.

8) Name the Indian Cricketer who became the highest international wicket-taker across formats in

2018?

Ans:- Jasprit Bumrah

Page 6: Monthly Current Affairs of SEPTEMBER - SSCE

SSCE 8981426494/8296260082 Downloaded from :- www.onlinessce.com Page 6

Explanation: The 25-year-old fast bowler Jasprit Bumrah became the highest international wicket-taker

across formats in 2018. He took a total of 78 wickets in the calendar year. He led the Indian cricket team to a

historic victory in the Boxing Day Test at the Melbourne Cricket Ground. Bumrah surpassed South Africa's

Kagiso Rabada's tally of 77 wickets to reach the milestone. While on the third position is Bumrah's

countryman Kuldeep Yadav who took 76 scalps.

9) Which airline gets its first longer range A321neo plane?

Ans:- IndiGo

Explanation: India's largest airline IndiGo has become the first domestic airline to have longer range Airbus

A321neo (new engine option) plane in its fleet with the first aircraft arriving from the European aviation

major Airbus' Hamburg facility in Germany. This is part of its 150-aircraft order. The new aircraft will help

the Rahul Bhatia-led airline to fly to Europe, and add destinations in Asia. IndiGo now has 204 planes in its

fleet, including one A321neo, 65 A320neos, 126 A320ceos (current engine option), and 12 ATRs. IndiGo at

present operates over 1,300 daily flights to 52 domestic and 15 international destinations.

10) Name the Arab country which has withdrawn its membership from OPEC, recently.

Ans:-Qatar

Explanation: Qatar has withdrawn its membership from Organization of the Petroleum Exporting Countries

(OPEC). The country had sent an official notification to the OPEC expressing its wish to pull out and to

focus on its liquefied natural gas production in December. Qatar has been in OPEC for 57 years. The

decision came amid the ongoing diplomatic and economic blockade imposed on the country by its Persian

Gulf neighbours and several Arab states.

11) As per Confederation of Indian Industry (CII)'s report titled 'Growth Outlook for 2019', What is

the GDP rate of India in FY19?

Ans: 7.5%

Explanation: Growth Outlook for 2019', Confederation of Indian Industry (CII) stated that India will

remain the fastest growing major economy with GDP growth estimated to be 7.5% in FY19. The reasons

cited for the growth are: Better demand conditions, settled GST implementation, capacity expansion

resulting from growing investments in infrastructure and better credit inflow especially in the service sector

(that is of 24%). Additionally, CII had suggested the following: only three slabs for GST - a standard rate, a

higher rate for demerit goods and a lower rate for some mass consumption items - and the inclusion of fuels,

real estate, electricity and alcohol in the ambit of the levy. RBI to have lending restrictions on banks under

the PCA (prompt and corrective action) and create a limited special liquidity window to meet emergencies

of financial institutions. the digitization of land records.

12) The government of India constituted a committee to suggest recommendations to strengthen the

'Real Estate Regulatory Authority' (RERA) and to remove difficulties in its implementation, who will

be the head of committee?

Ans:- Shiv Das Meena

Explanation: The government of India constituted a committee to suggest recommendations to strengthen

the 'Real Estate Regulatory Authority' (RERA) and to remove difficulties in its implementation. It will be

headed by the Union Housing & Urban Affairs Joint Secretary, Shiv Das Meena. RERA, which protects the

interest of home buyers, empowers states to notify their rules & appoint regulatory authorities.

13) Germany approves USD 8 billion rail and road tunnel link to which of the following country?

Ans:- Denmark

Explanation:- German authorities gave their approval to an US$8 billion (6.30 billion pounds) rail and road

tunnel linking Denmark and Germany, despite environmental objections. It is part-funded by the European

Page 7: Monthly Current Affairs of SEPTEMBER - SSCE

SSCE 8981426494/8296260082 Downloaded from :- www.onlinessce.com Page 7

Union. The 19 km (11.8 mile) Fehmarnbelt link, connecting the Danish island of Lolland to the island of

Fehmarn on the German side, was due to be completed in 2024.

14) Which Union Minister launched Ujjwala Sanitary Napkins scheme in Odisha?

Ans:-Shri Dharmendra Pradhan

Explanation:- Petroleum and Natural Gas Minister, Dharmendra Pradhan launched 'Ujjwala Sanitary

Napkins' initiative saying it will go a long way in making Odisha's 2.25 crore women empowered and self-

reliant. Under the novel initiative, which will cover 93 blocks in 30 districts of Odisha, oil marketing

companies (OMC) will set up 100 manufacturing units at the Common Service Centres (CSC) at an

estimated cost of Rs 2.94 crore.

15) Who is the head of the government committee that suggested AICTE to stop setting up new

engineering colleges from 2020?

Ans:- B V R Mohan Reddy

Explanation: A government committee, headed by IIT-Hyderabad chairman B V R Mohan Reddy, has

advised the All India Council for Technical Education (AICTE) to stop setting up new colleges from 2020

and review the creation of new capacity every two years after that. The panel has also suggested that no

additional seats should be approved in traditional engineering areas such as mechanical, electrical, civil and

electronics and that institutes should be encouraged to convert current capacity in traditional disciplines to

emerging new technologies. This recommendation has been justified on the ground that current capacity

utilisation in traditional disciplines is just 40% as opposed to 60% seat occupancy in branches such as

computer science and engineering, aerospace engineering and mechatronics.

16) Center has given an extension of 6 months to complete the ongoing exercise for updating the NRC

in __________.

Ans:- Assam

Explanation: The Center given an extension of six months to complete the ongoing exercise for updating

the National Register of Citizens (NRC) in Assam by 30th June 2019 as the enumeration of citizens in the

NRC cannot be completed within the earlier specified date of 31st December 2018. The Registrar General of

India, Mr. Sailesh issued a notification that stated the Central Government considers the extension necessary

in lieu of public interest to complete the update of the National Register of Citizens. The first notification

regarding the same was issued by the Government on 6th December 2013 and had set a deadline of 3 years

to complete the entire process but since then five extensions have been given but the exercise had not been

completed.

17) Sports Ministry has allotted how much rupees for the funding of athletes under the Target

Olympic Podium Scheme (TOPS) for their preparations for 2020 Olympics?

Ans:- Rs 100 crore

Explanation:- The Sports Authority of India Director General Neelam Kapur announced that The Sports

Ministry has earmarked Rs 100 crore for the funding of athletes under the Target Olympic Podium Scheme

(TOPS) for their preparations for 2020 Olympics. Launched in September 2014, TOPS, is a flagship

programme of the sports ministry to provide financial assistance to potential Olympic medal winners besides

giving the financial assistance to prepare for Commonwealth Games and Asian Games. On September 2018,

a TOPS Secretariat which consists of a team of professionals with management and sporting expertise was

established in a bid to fine tune India's preparations for the 2020 Olympics and build a sustainable sports

ecosystem in the country.

18) India has cut import taxes on crude and refined palm oil from ASEAN countries from 44% to

__________

Ans:- 40%

Page 8: Monthly Current Affairs of SEPTEMBER - SSCE

SSCE 8981426494/8296260082 Downloaded from :- www.onlinessce.com Page 8

Explanation:- The duty on crude palm oil from Malaysia, Indonesia and other members of the Association

of South East Asian Nations (ASEAN) was cut to 40% from 44%, while the tax on refined palm oil was cut

to 45% from 54% if imported from Malaysia and to 50%, if purchased from Indonesia or other member-

nations of Asean. In March 2018, India had raised import tax on crude palm oil to 44 percent from 30

percent and lifted the tax on refined palm oil to 54 percent from 40 percent.

19) Who of the following was appointed as the MD and CEO of Axis bank after Shikha Sharma?

Ans:- Amitabh Chaudhry

Explanation: Axis Bank announced its Managing Director and CEO Shikha Sharma has retired from the

post and Amitabh Chaudhry will be the new managing director and CEO of the bank with effect from

January 1, 2019. Chaudhry, the former MD and CEO of HDFC Standard Life Insurance Company, was in

September named the MD and CEO of Axis Bank for a period of three years.

20) Who was sworn-in as 1st Chief Justice of Telangana High Court?

Ans:- TB Radhakrishnan

Explanation:- Justice Thottathil Bhaskaran Nair Radhakrishnan had been serving as the Chief Justice of the

combined high court of the two Telugu states since July last year. Telangana High Court came into existence

on January 1, 2019 after the bifurcation of the High Court at Hyderabad. Justice Praveen Kumar was sworn

in as the acting Chief Justice of Andhra Pradesh HC, which also came into existence the same

Daily Current Affairs 3rd January, 2019

1)Pankaj Sharma was appointed as the chief operations officer of this bank recently.

Ans:- RBL Bank

Explanation:-Private sector lender RBL Bank appointed Pankaj Sharma as its chief operations officer

(COO). Prior to this, Sharma was heading retail operations of Axis Bank. Under his leadership, the ATM

channel, phone banking and branch automation and digital initiatives at Axis Bank took shape.

2)ISRO launched an initiative for students to engage them on space science activities. It was named

as;

Ans:-Samwad

Explanation:-The Indian Space Research Organisation (ISRO) has recently launched Samwad with

Students as part of its outreach programme. The Indian Space agency will engage youngsters across the

country in activities concerning space science under this programme. The new conversation mission will

inspire students cutting across schools and colleges. During the inauguration in Bengaluru, 40 students and

10 teachers from select schools interacted with ISRO Chairman Dr. K Sivan about the Indian Space

Programme and their benefits to the common man.

3)The number of complaints registered at banking ombudsman offices for 2018 have increased with

the percentage of;

Ans:-25%

Explanation:-The number of complaints registered at banking ombudsman offices have seen a 25%

increase in the fiscal year 2018, with the majority of these complaints coming from urban centres owing to

increased awareness and poor internal redressal mechanisms of banks. "The banking ombudsman offices in

tier-1 cities like New Delhi, Mumbai, Chennai, Kolkata, Bengaluru and Hyderabad accounted for more than

57% of the total complaints received by all BO offices", the RBI Report on Trend and Progress of Banking

in India has stated.

4)Prime Minister Narendra Modi has inaugurated the 106th Indian Science Congress in which of the

following city?

Page 9: Monthly Current Affairs of SEPTEMBER - SSCE

SSCE 8981426494/8296260082 Downloaded from :- www.onlinessce.com Page 9

Ans:-Jalandhar

Explanation:-Prime Minister Narendra Modi has inaugurated the 106th Indian Science Congress (ISC-

2019) at Jalandhar, Punjab. The ISC is an annual congregation that witnesses deliberations by top scientific

minds of the country. The theme for which is 'Future: India-Science and Technology'. This will be his 5th

address to Indian Science Congress after assuming office as Prime Minister. Three Nobel laureates will

participate in this conference. Students will showcase their projects to scientists around the world.

5)Veteran actor CH Lokanath who addressed as 'Uncle Lokanath' was passed away at the age of 91.

He belongs to which film industry?

Ans:-Sandalwood

Explanation:-Veteran Kannada actor CH Lokanath passed away after suffering cardiac arrest. He was 91.

Lokanath was in the Kannada film industry for nearly six decades and was affectionately addressed as

'Uncle Lokanath' by his fans. He had acted in over 600 movies and more than 1,000 plays. His famous

movies are Bhootayyana Maga Ayyu, Minchina Ota, Nagarahaavu, Nodi Swami Navidru Heege, Katha

Sangama and Kittu Puttu.

6)Which spacecraft of NASA broke the record for smallest space object ever orbited, after

successfully entering into orbit around the asteroid Bennu?

Ans:-OSIRIS-Rex

Explanation:-NASA's OSIRIS-REx spacecraft broke a record after successfully entering into orbit around

the asteroid Bennu - the smallest cosmic object ever to be orbited by a spacecraft. NASA's OSIRIS-REx

spacecraft, 110 million kilometers away, carried out a single, eight-second burn of its thrusters. The

manoeuvre placed the spacecraft into orbit around the asteroid Bennu. The spacecraft will circle Bennu

about 1.75 kilometres from its centre, closer than any other spacecraft has come to its celestial object of

study.

7)Which country ranked first as per the Climate Change Performance Index (CCPI)?

Ans:-Sweden

Explanation:-Climate Change Performance Index (CCPI) reported that Sweden has been named the first

best performing country in the CCPI 2019 list. CCPI evaluates the climate protection performance of 60

countries, responsible for over 90% of global energy-related CO2 emissions. Morocco has been named

second. Morocco, the North African country significantly increased the share of renewables over the past

five years and increased new renewable energy capacity. Lithuania is in the third position in CCPI 2019.

The bottom five in the list are Saudi Arabia, U.S., Iran, South Korea, and Taiwan. With the connection of

the world's largest solar plant to the grid, Morocco is on track for achieving its target of 42% installed

renewable energy capacities by 2020.

8)Veteran cricket coach Ramakant Achrekar passed away at the age of 87. For whom he trained?

Ans:-Sachin Tendulkar

Explanation:-Veteran cricket coach Ramakant Achrekar, who trained Bharat Ratna Sachin Tendulkar and

Vinod Kambli, among others, passed away following a heart attack. He was 87 and breathed his last at his

home in Dadar. Achrekar, honoured with Padma Shri in 2010 and Dronacharya Award in 1990, was

instrumental in shaping the careers of Tendulkar, Kambli and over a dozen other top cricket stars.

9)Name the Indian origin student who has bagged two gold medals at the World Memory

Championships in Hong-Kong.

Ans:-Dhruv Manoj

Explanation:-A 12-year-old Indian-origin student Dhruv Manoj from Singapore has bagged two gold

medals at the World Memory Championships held in Hong Kong from 20th December to 22nd December

Page 10: Monthly Current Affairs of SEPTEMBER - SSCE

SSCE 8981426494/8296260082 Downloaded from :- www.onlinessce.com Page 10

2018. Dhruv Manoj won in the "names and faces" and "random words" disciplines, beating 56 other

contestants in the "kids category". Dhruv has mastered the Roman memory technique of creating "memory

palaces", which works by associating the ideas or objects to be memorized with scenes imagined at familiar

locations, such as one's house.

10)Saurabh Kumar has been appointed as the Director General of Ordnance Factories (DGOF) and

chairman of this department.

Ans:-OFB

Explanation: Saurabh Kumar has been appointed as the Director General of Ordnance Factories (DGOF)

and chairman of the Ordnance Factory Board (OFB). Kumar, a 1982-batch Indian Ordnance Factory Service

officer, is an M-Tech in mechanical engineering from the Indian Institute of Technology (IIT), Kanpur. An

expert in the manufacture of ordnance, Kumar was on deputation to the Ministry of Defence as the director

of planning and coordination from 2002 to 2009.

11) The West Bengal government announced __________ welfare initiatives for farmers.

Ans:-2

Explanation: The West Bengal government announced two welfare initiatives for farmers. Both initiatives,

which are part of the 'Krishi Krishak Bondhu' scheme, will come into effect from 1st January 2019. The first

of the two initiatives will provide Rs 2 lakh to the family of a deceased farmer. Under the second initiative,

farmers would get Rs 2,500 twice a year for growing a single crop on one acre of land.

Daily Current Affairs 4th January, 2019

1)Which Atomic Power Station set world record for longest uninterrupted operation?

Ans:-Kaiga Atomic Power Station

Explanation:-Karnataka's Kaiga Atomic power station created a world record for the longest uninterrupted

operation for 962 days on December 10. It had earlier set a record of 941 days. The earlier record of 940

days by Heysham-2 unit 2 was set by the United Kingdom. Generating Station-1 (KGS-1) at Kaiga in Uttara

Kannada, Karnataka has now been shut down for maintenance after 962 days of operation. It will be

commissioned again after the maintenance activities. Nuclear Power Corporation of India (NPCIL) said that

during its continuous operation of 962 days KGS-1 had produced 5 billion units of electricity at a plant load

factor of 99.3%. Kaiga has four indigenously developed Pressurised Heavy Water Reactors of 220 MW each

fuelled by domestic fuel. Of them, KGS-2 has been operating since the last 661 days.

2)The 44th Global Summit of the International Advertising Association was held in __________.

Ans:-Kochi

Explanation:-The 44th Global Summit of the International Advertising Association (IAA) will be held in

Kochi from February 20 to 22, 2019. The theme of the summit is 'Brand Dharma'. It is for the first time an

Indian city is hosting the summit. The last three summits were held in Washington DC, Moscow and

Beijing.

3)Which of the following become the country's largest asset management company?

Ans:HDFC Mutual Fund

Explanation:-HDFC Mutual Fund has pipped ICICI Prudential MF to become the country's largest asset

management company after a gap of over two years. As of December-end, HDFC MF manages assets to the

tune of Rs 3.35 lakh crore, while those of ICICI Prudential MF stood at Rs 3.08 lakh crore, as per the latest

data available with Association of Mutual funds in India. The assets under management (AUM) of HDFC

MF have grown over 9% in October-December quarter from the previous three-month period, however,

ICICI Prudential MF's AUM slipped by 0.6% during the period under review.

4)The World's 1st female who amputee to scale Antarctica's Highest Peak is;

Page 11: Monthly Current Affairs of SEPTEMBER - SSCE

SSCE 8981426494/8296260082 Downloaded from :- www.onlinessce.com Page 11

Ans:-Arunima Sinha

Explanation:-Arunima Sinha, the first female amputee to have conquered Mt Everest, created yet another

record as she became world's first woman amputee to climb Mount Vinson, the highest peak in Antarctica.

Apart from Mount Everest, the 30-year-old has climbed the highest peaks of five continents and plans to do

so in all seven.

5)Name of the IIT professor who was honoured with the National Geospatial Award for Excellence -

2017.

Ans:-Jayanta Kumar Ghosh

Explanation:-IIT Roorkee professor Jayanta Kumar Ghosh was felicitated with the National Geospatial

Award for Excellence -2017 (Life Time Achievement Award) by the Indian Society for Remote Sensing for

his contribution towards the development of geospatial science and applications. The award which carries a

citation, a medal and Rs.1,00,000 was conferred to J. K. Ghosh, a professor at the civil engineering

department of the IIT Roorkee on 5th December 2018 at SAC (Space Application Centre) Ahmedabad. The

Indian Society for Remote Sensing is an organisation known in the area of space science, remote sensing

and geo-spatial technology.

6)What is the theme of the 106th edition of Indian Science Congress?

Ans:-Future India: Science & Technology

Explanation:-The 106th Indian Science Congress (ISC-2019) has been inaugurated by Prime Minister

Narendra Modi at Lovely Professional University (LPU) in Jalandhar, Punjab from 3rd January 2019. The

theme of the 5-day science congress is 'Future India: Science & Technology'. Approx 30,000 delegates,

including 300 top scientists and Nobel Laureates from across world are attending the congress. In it, around

100 plus conferences and events of scientific and technology origin will also be held, where eminent

personalities from DRDO, ISRO, DST, AIIMS, UGC, AICTE, and many elite universities of the USA, UK,

India and other countries will participate. The event will play a pivotal role in stimulating scientific research

effort and raising the scientific temperament in the country and the world. There will also be four parallel

mega events, including the children's science conference; women's science conference; science

communicators' meet; and, science exhibition. The exhibition will see participation from the national and

international science companies, especially from the US, Australia, Germany and Switzerland.

7)The Union Home Ministry renamed Allahabad as;

Ans:-Prayagraj

Explanation:-The Union Home Ministry approved a Uttar Pradesh government proposal to rename

Allahabad as Prayagraj. In October 2018, the UP government had proposed the change in name of

Allahabad to Prayagraj. The Home Ministry clears the name change proposals after the Ministry of

Railways, the Department of Posts and the Survey of India confirm that no city, town or village has similar

names.

8)Which life insurance uses WhatsApp to deliver policy, renewal premium?

Ans:-Bharti AXA Life Insurance

Explanation:-Bharti AXA Life Insurance started delivering policies and renewal premium receipt to

customers via instant messaging platform WhatsApp as a part of its alternative service option to the

policyholders. Bharti AXA Life Insurance, a joint venture between Bharti Enterprises and French Insurance

major AXA is one of the first few insurers to provide the option of policy contract, renewal premium

receipts and claim intimation through WhatsApp.

9)This ministry imposed an anti-dumping duty for 18 months on a Chinese synthetic rubber, recently.

Ans:-Ministry of Commerce and Industry

Page 12: Monthly Current Affairs of SEPTEMBER - SSCE

SSCE 8981426494/8296260082 Downloaded from :- www.onlinessce.com Page 12

Explanation:-The Ministry of Commerce and Industry, Government of India, is mulling to impose anti-

dumping duty for 18 months on a Chinese synthetic rubber 'fluoroelastomers' used in automobile and other

industries in order to minimize the impact of dumped imports. The proposal in this regard was considered

after the investigative arm of Commerce ministry, Directorate General of Trade Remedies (DGTR) initiated

the probe in January 2018 following a complaint from Gujarat Fluor chemicals which alleged dumping of

the product. The DGTR said that the product has been exported to India from China at below its normal

value, which has resulted in dumping and recommended a duty which ranges between $0.078 -$7.31 per

kilogram. The final decision to impose the duty would be taken by Ministry of Finance. The duty which is

imposed under the under the multilateral regime of World Trade Organization (WTO) is aimed at ensuring

fair trading practises and creating a level-playing field for domestic producers with regard to foreign

producers and exporters.

10)Which banks signs a bancassurance pact with SBI Life Insurance to offer a comprehensive

financial planning solution to its customers?

Ans:-Syndicate Bank

Explanation:-Syndicate Bank and SBI Life Insurance signed a bancassurance pact aiming to offer a

comprehensive financial planning solution to its customers. The pact was signed between Syndicate Bank

MD & CEO, Mrutyunjay Mahapatra and MD and CEO of SBI Life, Sanjeev Nautiyal. Through the pact,

Syndicate Bank will provide penetration in the market with its 3,000 branches and SBI Life's diverse range

of protection, wealth creation and savings insurance products will be brought to the table to the customers.

11)Who will allows Custodial Services In Commodity Market?

Ans:-SEBI

Explanation:-Markets regulator Securities and Exchange Board of India (SEBI) has allowed custodial

services in the commodity derivatives market. The move is aimed at enabling the participation of

institutional investors such as mutual funds and portfolio managers in the commodity derivatives market.

Under the new framework, existing custodians will be permitted to add commodities as an asset class and

provide physical delivery of both the securities and commodities. Currently, the regulation on the custodian

of securities provides safekeeping of securities, gold or gold related instruments, title deeds of real estate and

incidental services.

12)The Union Cabinet approved a MoU between India and this country on Cooperation in the area of

Cyber Security.

Ans:-Morocco

Explanation:-The Union Cabinet chaired by Prime Minister Narendra Modi approved an MoU between

India and Morocco on Cooperation in the area of Cyber Security. It also approved an Apprised of Progress

under NHM and Decisions of Empowered Programme Committee and Mission Steering Group of NHM.

Proposal to allot land to Chandigarh Housing Board for construction of apartments for 3930 allottees under

Self-Financing Housing Scheme for UT Employees.

13)Which state CM launched a KALIA scheme for farmers?

Ans:-Odisha

Explanation:-Chief Minister of Odisha, Naveen Patnaik, launched a Rs 10,180-crore scheme,for overall

development of small, marginal and landless farmers, dubbed as 'The Krushak Assistance for Livelihood

and Income Augmentation (KALIA)' scheme. This scheme consists of a life insurance of Rs.2 lakh for 57

lakh farmers and would grant them crop loans at zero percent interest. Also,financial assistance of Rs 10,000

per farmer family per annum to around 30 lakh small and marginal farmers' families. Furthermore,Rs. 5000

would be given to farmers for Rabi cultivation and equal amount will be given for Kharif Cultivation.

14)Which football team have been awarded the Legion of Honour in the New Year's honours list?

Page 13: Monthly Current Affairs of SEPTEMBER - SSCE

SSCE 8981426494/8296260082 Downloaded from :- www.onlinessce.com Page 13

Ans:-France

Explanation:-France's football team which last year defeated Croatia to win the World Cup were awarded

the Legion of Honour, France's highest award for "exceptional service" to the nation, in the New Year's

honors list. The football World Cup 2018 at Russia was France's second World Cup title, after the 1998

World Cup. Each of the 23 players of the French football team, including star strikers Kylian Mbappe and

Antoine Griezmann, received France's top medal for their part in beating Croatia in the World Cup final in

Moscow.

15)What type of improved aircraft did the Indian Air Force (IAF) ask Hindustan Aeronautics

Limited (HAL) to make after getting clearance from military airworthiness certifier CEMILAC?

Ans:-Light Combat Aircraft

Explanation:-Light Combat Aircraft (LCA) Tejas progressed towards manufacture in an enhanced, battle

standard format on December 31. The LCA is being designed and developed by the Aeronautical

Development Agency (ADA) in Bengaluru. A new limited clearance from military airworthiness certifier

CEMILAC for the Indian fighter green-lights its production in a superior lethal version. HAL aims to get the

first aircraft out in late 2019. The Indian Air Force (IAF) has asked Hindustan Aeronautics Limited (HAL)

to make 40 LCA aircraft. Of this, 20 will be in the advanced Final Operational Clearance (FOC) format.

Another 20 are in the earlier Initial Operational Clearance (IOC) version. The IAF has modified and

upgraded its trainer requirement in its old package order of 40 LCA aircraft.

16)Who signed an Asia Reassurance Initiative Act Into Law recently?

Ans:Donald J. Trump

Explanation:-U.S. President Donald J. Trump signed into law the Asia Reassurance Initiative Act (ARIA),

which passed the U.S. Senate earlier in December. According to the White House, the act establishes a

multifaceted U.S. strategy to increase U.S. security, economic interests, and values in the Indo-Pacific

region. Specifically, the ARIA will authorize $1.5 billion in spending for a range of U.S. programs in East

and Southeast Asia and develop a long-term strategic vision and a comprehensive, multifaceted, and

principled United States policy for the Indo-Pacific region, and for other purposes.

17)This city aims for 7 star rating under the Swachh Bharat Mission.

Ans:-Mysuru

Explanation:-The Mysuru City Corporation (MCC) aims for a 7-star under the Swachh Bharat Mission

2019. Under this mission, the garbage-free cities will be awarded star ratings as per the revised protocols of

the Ministry of Housing and Urban Affairs (MoHUA). The new survey is launched across the country from

4th January 2019. MoHUA has set various parameters such as door-to-door garbage collection, waste

disposal, scientific handling of landfills, disposal of construction and demolition debris, and segregation of

waste at the source for the rankings. As per the vision enunciated under the Swachh Bharat Mission, 100%

of waste generated in a city should be scientifically managed, and measures are taken for visible

beautification of the city.

18)Who has been appointed as the new Chief Justice of Pakistan?

Ans:-Asif Saeed Khan Khosa

Explanation:-Justice Asif Saeed Khosa will take oath of his office on January 18 a day after incumbent CJP

Justice Saqib Nisar retires on January 17. Justice Khosa has been a senior Justice of the Supreme Court of

Pakistan since 31 December 2016. He will remain as the top judge of Pakistan till December 20, 2019.

Daily Current Affairs 5th January, 2019

1)Who among the following was named as the acting deputy defence secretary of United States?

Page 14: Monthly Current Affairs of SEPTEMBER - SSCE

SSCE 8981426494/8296260082 Downloaded from :- www.onlinessce.com Page 14

Ans:-David Norquist

Explanation:-Pentagon comptroller David Norquist was named acting deputy defence secretary of United

States. He has been the department's chief financial officer and comptroller since May 2017. In addition to

that, Deputy Secretary of Defence Patrick Shanahan was appointed to serve as the acting secretary of

defence.

2)Govt launches Green-Ag, GEF-assisted project, in partnership with;

Ans:-FAO

Explanation:-The Minister of State for Agriculture & Farmers Welfare Shri Parshottam Rupala informed

that the Government had launched a Global Environment Facility (GEF) assisted project namely, "Green -

Ag: Transforming Indian Agriculture for global environment benefits and the conservation of critical

biodiversity and forest landscapes" in collaboration with the Food and Agriculture Organisation (FAO). The

project was launched in high-conservation-value landscapes of five States namely (i) Madhya Pradesh:

Chambal Landscape, (ii) Mizoram: Dampa Landscape, (iii) Odisha: Similipal Landscape, (iv)Rajasthan:

Desert National Park Landscape and v) Uttarakhand: Corbett-Rajaji Landscape. The project is aimed to

mainstream biodiversity, climate change and sustainable land management objectives and practices into

Indian agriculture. Besides this it is also supposed to catalyse transformative change of India's agricultural

sector to support achievement of national and global environmental benefits and conservation of critical

biodiversity and forest landscapes. The project will be funded through 33.5 million dollar grant from the

Global Environment Facility (GEF) implemented by the government of India and the Food and Agriculture

Organization (FAO) of the United Nations.

3)On which date, the World Braille Day is observed?

Ans:-4th January

Explanation:-The United Nations observed the first official World Braille Day on 4 January. The aim of the

day is to raise awareness of the importance of Braille. It also aims to raise awareness about the human rights

of visually-impaired and partially-sighted people, The day was proclaimed by the UN General Assembly in

November 2018.

4)The scientists from this country launched a Telescope to detect cosmic X-rays.

Ans:-US

Explanation:-US scientists have successfully launched a telescope from Antarctica that analyses X-rays

arriving from distant neutron stars, black holes, and other exotic celestial bodies. The name of the telescope

is 'X-Calibur' instrument. It is carried by a helium balloon and will reach an altitude of 130,000 feet. Neutron

stars are objects of very small radius.

5)Name of the public sector miner, launched the social media wall on the YouTube Channel Singareni

Siren.

Ans:-SCCL

Explanation:-Singareni Collieries Company Limited (SCCL), a public sector miner, launched the social

media wall on the YouTube Channel Singareni Siren, Facebook application, Instagram and Twitter handles,

all at one place. It aims at bringing together all the social media applications introduced to disseminate

information on the developments. The application would incorporate information pertaining to coal

production, transport and details of power generation by the National Thermal Power Corporation (NTPC)-

run plants in addition to welfare activities taken up by the company for the employees as well as those

residing in the mining areas. Those interested can click the social media wall icon placed on the website

www.scclmines.com to view/download information about its activities.

6)The Kerala State Cabinet approved which of the following Act to provide a regular income for Non-

Resident Keralites?

Page 15: Monthly Current Affairs of SEPTEMBER - SSCE

SSCE 8981426494/8296260082 Downloaded from :- www.onlinessce.com Page 15

Ans:-Kerala Pravasi Welfare Act

Explanation:-The Kerala State Cabinet decided to recommend the Governor to promulgate an Ordinance

amending the Kerala Pravasi Welfare Act. The Act aims at providing a regular income for Non-Resident

Keralites (NoRKs) returning home. The amount collected through the scheme will be used to fund

development activities taken up by Kerala Infrastructure Investment Fund Board (KIIFB) and other

agencies. The Cabinet resolved to enhance the monthly remuneration for special Government pleaders to

Rs.1,20,000, senior government pleaders to Rs.1,10,000 and that for government pleaders to Rs.1,00,000.

The proposed scheme will enable the Kerala Pravasi Welfare Board to receive deposits from non-resident

Malayalis and use it, along with a matching share from the government, to provide a monthly dividend for

investors.

7) PayU India CEO Amrish Rau joined as the head of which of the following company?

Ans:-Naspers

Explanation:-PayU India's CEO Amrish Rau joined as the head of financial technology partnerships and

investments for Naspers' fin tech business. Nasper is South African parent of PayU. Rau will continue to

report to Laurent le Moal the chief executive of PayU, which is headquartered in the Netherlands.

8)The mega international food and beverage show 'Indus Food 2019' will be held in __________.

Ans:-Noida

Explanation:-The mega international food and beverage show 'Indus Food 2019' will be held in Greater

Noida in Uttar Pradesh on 14-15 January 2019. The event is organised jointly by Trade Promotion Council

of India and Department of Commerce. The event is a platform to promote India as a strong exporter of food

and beverage products to the world. Odisha will participate as a partner state of the event.

9)Who inagurated the 'Swachh Survekshan 2019' scheme recently?

Ans:-Hardeep Singh Puri

Explanation:-Housing and Urban Affairs Minister, Hardeep Singh Puri inaugurated the 'Swachh

Survekshan 2019' on 4 January 2019. The cleanliness survey will cover 40 crore people across over four

thousand towns and cities. For the first time ever, inspection will be conducted without giving any prior

information to the concerned official for real-time check and survey will be based on people's feedback.

10)Global Healthcare Summit 2019 will be held in __________.

Ans:-Hyderabad

Explanation:-The 13th edition of Global Healthcare Summit 2019 is to be held in Hyderabad from July 21

to 24. The summit is organized in partnership with the Global Association of Physicians of Indian Origin.

The organizing chapters of the conference include Osmania, Gandhi, Kakatiya and Telangana Medical

alumni, Association of Telugu Medical Graduates of the U.S.A., Gandhi Medical College Global Alliance,

and Kakatiya Medical College Old Students' Association. More than 500 physicians are expected to attend

the summit. The summit would have an international research competition for which the American

Association of Physicians with Indian Origin (AAPI) would work in collaboration with a leading academic

medical college in Hyderabad.

11)How many free cooking gas (LPG) connections has the government provided under the Pradhan

Mantri Ujjwala Yojana (PMUY)?

Ans:-6 crore

Explanation:-The government gave out the 6-croreth free cooking gas (LPG) connection under the Pradhan

Mantri Ujjwala Yojana (PMUY) to make available cleaner fuel in every household kitchens. Vice President

M Venkaiah Naidu handed over papers for an LPG connection to Jasmina Khatoon from Shivpark, Khanpur

in New Delhi to mark the 6 crore LPG connections under PMUY. Under the scheme, the government

Page 16: Monthly Current Affairs of SEPTEMBER - SSCE

SSCE 8981426494/8296260082 Downloaded from :- www.onlinessce.com Page 16

provides a subsidy of Rs 1,600 to state-owned fuel retailers for every free LPG gas connection that they give

to poor households. The 5-croreth free LPG connection under the scheme was given to Takrdiran, a resident

of Sangam Vihar of the national capital, in August 2018 by Lok Sabha Speaker Sumitra Mahajan.

12)How many oil companies signed an MoU with IIT Bombay for setting up Centre of Excellence in

Oil, Gas and Energy?

Ans:-7

Explanation:-IIT Bombay and 7 public sector oil companies inked a memorandum of understanding (MoU)

for setting up Centre of Excellence in Oil, Gas and Energy. The 7 company includes Indian Oil Corp; Oil

and Natural Gas Corp (ONGC); GAIL India Ltd; Bharat Petroleum Corp Ltd (BPCL); Hindustan Petroleum

Corp Ltd (HPCL); Oil India Ltd and Engineers India Ltd. This would link up energy industry and academics

to help jointly address energy requirements with new and innovative, indigenous technology.

13)Prime Minister Narendra Modi inaugurated ___________ projects worth over Rs. 1500 crore on

his one-day visit to Imphal, Manipur.

Ans:-12

Explanation:-On the Prime Minister, Shri Narendra Modi's one day visit to Imphal, Manipur, he

inaugurated 12 projects worth over Rs. 1500 crore. The 12 projects consisting of eight key development

projects related to water supply and tourism and four schemes in Manipur. These include: An Integrated

Check Post at Moreh, Imphal, at a cost of over Rs. 125 crores, that will facilitate customs clearance, foreign

currency exchange, immigration clearance. the Dolaithabi Barrage Project which completed at a total cost of

Rs. 500 crore, the FCI Food Storage Godown at Sawombung that had begun in December 2016, He

dedicated to the nation the 400 kV Double Circuit Silchar-Imphal Line that was made at a cost of Rs. 700

crore. A buffer water reservoir at Ukhrul was mentioned which would be helping the citizens till 2035. He

inaugurated improved and upgraded water supply for Churachandpur zone which will help the citizens till

2031.

14)India floats mega tender of 7.5 GW for this state.

Ans:-Jammu & Kashmir

Explanation:-Solar Energy Corporation of India has invited solar power developers to construct 7.5 GW

grid connected projects in Leh and Kargil Districts in the state of Jammu & Kashmir under a competitive

bidding process. Out of the 7.5 GW capacity, 2.5 GW will be constructed in the Kargil district and 5 GW in

Leh district. The tender is part of Prime Minister Narendra Modi's goal of building 100 gigawatts of solar

capacity in the country by 2022.

15)Indian Oil Corp (IOC)'s first LNG import terminal was commissioned in __________, Tamil Nadu.

Ans:-Ennore

Explanation:-Indian Oil Corp (IOC), the nation's biggest oil firm, announced commission of its first Rs

5,151-crore worth Liquefied Natural Gas (LNG) import terminal in Ennore, Tamil Nadu. This is the first

LNG import terminal IOC has built on its own and will become operational by the end of January 2019. IOC

holds 95 per cent stake in the Ennore LNG import terminal. Tamil Nadu Industrial Development

Corporation (TIDCO) has 5 per cent. The firm is also working on laying a 1,385 km natural gas pipeline

originating from the Ennore terminal to Nagapattinam in Tamil Nadu via Puducherry. Also, branch pipelines

will be laid in Madurai, Tuticorin, and Bengaluru to meet the LNG demand.

16)For how many stations, Indian Railways plans to launch a WiFi facility?

Ans:-2000

Explanation:-Railway Minister Piyush Goyal instructed officials to provide WiFi facilities at a minimum of

2,000 stations as soon as possible. He also called for setting up a single helpline number for all non-security

railway complaints by the end of January. Additionally, Point of Sale (PoS) machines will be distributed in

Page 17: Monthly Current Affairs of SEPTEMBER - SSCE

SSCE 8981426494/8296260082 Downloaded from :- www.onlinessce.com Page 17

all trains to each catering staffer by March 31 to address complaints related to overcharging by the caterer.

The government will also replace conventional coaches with modern Linke Hofmann Busch (LHB) design

coaches in all long distance trains.

17)Scientists have developed a device named WAND that works like a Pacemaker for which this part

of the body?

Ans:-Brain

Explnation:-Scientists have developed a device named WAND which stands for "Wireless artifact-free

neuromodulation device" that works like a "Pacemaker for the brain" to monitor the brain's electrical activity

and potentially deliver fine-tuned treatments to patients suffering from diseases like epilepsy and

Parkinson's. The most important feature of the device is that it is both wireless and autonomous, which

means when it learns to recognize the signs of tremor or seizures then it can adjust the stimulation

parameters on its own to prevent the unwanted movements. The device has custom integrated circuits to

record the full signal from both the subtle brain wave and the strong electrical pulses.

18)Who is appointed as the MD & CEO of Thyssenkrupp Industries India?

Ans:-Vivek Bhatia

Explanation:-Thyssenkrupp, the German industrial giant, appointed Vivek Bhatia as the Managing Director

(MD) & Chief Executive Officer (CEO) of Thyssenkrupp Industries India. Mr.Bhatia was earlier the CEO -

Asia Pacific at Thyssenkrupp AG, driving group activities for all Thyssenkrupp companies in the region.

Prior to joining Thyssenkrupp, Mr. Bhatia was earlier with Boston Consulting Group, India.

19)Which Chinese space probe touched down on the far side of the moon?

Ans:-Chang'e-4 lunar probe

Explanation:-Chang'e-4 lunar probe, a Chinese space probe, touched down on the far side of the moon. The

probe, which has a lander and a rover, that was launched in December made the soft landing and transmitted

the first-ever close-range image of the far side of the moon. It touched down at a targeted area near the

moon's south pole in the Von Karman Crater. The purpose of the Chang'e-4 includes astronomical

observation, surveying the moon's terrain, landform, and mineral makeup, and measuring the neutron

radiation and neutral atoms to study the environment of its far side. The moon is tidally locked to Earth,

rotating at the same rate as it orbits our planet, so most of the far side or dark side is never visible to us.

Previous spacecraft have seen the far side, but none has landed on it. China's space agency hailed the event

as a historic first and a major achievement for the country's space programme.

Daily Current Affairs 6th January, 2019

1)Where was the 10th R R Lakshya Cup 2018 shooting tournament held?

Ans:-Navi Mumbai

Explanation:-Shooter Divyansh Singh Panwar from Rajasthan and shooter Nupur Patil from Maharashtra

have won 10th R R Lakshya Cup 2018 in senior and junior category respectively. Dronacharya Awardee and

former National Coach Prof. Sunny Thomas was the chief guest of the event. The tournament was organised

in Lakshya Shooting Club, Karnala Sports Academy, Panvel, Navi Mumbai. The Club was founded by ace

shooter Suma Shirur. R R Lakshya Cup being only a premier invitational tournament, it's record wouldn't be

count for International records. The event has prize money of Rs. 1,00,000 in senior category and Rs. 50,000

in junior category.

2)How many monumnets are declared under the Ancient Monuments and Archaeological Sites and

Remains Act, 1958, in 2018?

Ans:-6

Page 18: Monthly Current Affairs of SEPTEMBER - SSCE

SSCE 8981426494/8296260082 Downloaded from :- www.onlinessce.com Page 18

Explanation:-The Ministry of Culture announced that Archaeological Survey of India (ASI) declared 6

monuments as 'protected and of national importance' under the Ancient Monuments and Archaeological

Sites and Remains Act, 1958, in 2018. These sites are: 125-year-old Old High Court Building in Nagpur,

Maharashtra, 2 Mughal era monuments in Agra- Haveli of Agha Khan and Hathi Khana, The Neemrana

Baori in Alwar district of Rajasthan, The Group of Temples in Ranipur Jharail in Bolangir district of Odisha,

The Vishnu Temple in Kotali, Pithoragarh district in Uttarakhand. After this the total number of centrally

protected sites or monuments under ASI are 3693, increasing from 3686. Among that, Uttar Pradesh has 745

sites, Karnataka has 506 and Tamil Nadu has 413 sites, marking them as the states having the highest

number of ASI-maintained sites.

3)Who was appointed the secretary of Competition Commission of India (CCI)?

Ans:-P K Singh

Explanation:-P K Singh was appointed secretary of the Competition Commission of India (CCI). Prior to

this, he was adviser (law) to the competition watchdog. After completing 5 years of experience as the

adviser to the CCI, according to the norms of the institution, he was chosen as the candidate. His scope of

responsibilities include acting as the nodal officer for the Commission, making or receiving all statutory

information and entering into the any formal relationships including signing of memorandums with any

foreign agency or other agency with prior approval from Commission and government.

4)Scientists from which institute have developed a simple paper kit that can test freshness of milk and

tell how well it has been pasteurized?

Ans:-IIT, Guwahati

Explanation:-Scientists at Indian Institute of Technology, Guwahati have developed a simple paper kit that

can test freshness of milk and tell how well it has been pasteurized. The kit is sided with smart phone app to

ensure that milk is consumed before it turns too sour. The team has prepared a kit by attaching probe discs

onto a 2 cm square transparent cellulose acetate film. The probe is then covered with another cellulose

acetate film. Colour reaction takes place when milk is injected through a tiny hole in the cover and a smart

phone can be used to get the results. It takes just about 15 minutes to detect raw milk from pasteurized one.

5)SBFC and this bank signed a Memorandum of Understanding to provide loans worth Rs. 1 crore to

MSMEs for 15-year tenure.

Ans:-ICICI Bank

Explanation:-ICICI Bank signed a Memorandum of Understanding (MoU) with Small Business FinCredit

India Pvt Ltd (SBFC) on 4 January 2019. Under the MoU, it will jointly provide loans worth up to Rs 1

crore to the Micro, Small and Medium Enterprises (MSMEs). ICICI Bank will co-originate loan against

property with SBFC at a mutually agreed ratio. The loans of up to Rs 1 crore is for a tenure of 15 years.

6)Which Indian state ranked in the ranking of Singapore's Asia Competitiveness Institute's (ACI),

Ease of Doing Business (EDB) index 2018?

Ans:-Andhra Pradesh

Explanation:-In the latest ranking of Singapore's Asia Competitiveness Institute's (ACI), Ease of Doing

Business (EDB) index 2018, Andhra Pradesh emerged on top of the 21 states of India that were considered

for the ranking, while Maharashtra and Delhi came in second and third place, respectively. The ACI's EDB

index is based upon three parameters called ABC -Attractiveness to Investors, Business Friendliness and

Competitiveness Policies. iii. The index captured the business environment at the sub-national level on the

basis of 72 hard and soft indicators. The Ease of Doing Business (EDB) list is compiled by a team of

researchers led by Tan Khee Giap, who is the co-director of Asia competitiveness institute (ACI), and

associate professor at Lee Kuan Yew School of Public Policy, National University of Singapore.

7)The theme of 27th New Delhi World Book Fair is;

Page 19: Monthly Current Affairs of SEPTEMBER - SSCE

SSCE 8981426494/8296260082 Downloaded from :- www.onlinessce.com Page 19

Ans:-Readers with Special Needs

Explanation:-To mark Mahatma Gandhi's 150th birth anniversary, there will be a special book exhibition of

books on and by Mahatma Gandhi. Over 20 countries will participate in the fair. The 27th New Delhi World

Book Fair has begun at Pragati Maidan. Human Resources Development Minister Prakash Javadekar

inaugurated the 9-day annual event. The theme of this year is 'Readers with Special Needs'.

8The bill passed by the Parliament that seeks to do away with the no-detention policy in schools.

Ans:-Right of Children to Free and Compulsory Education (Amendment) Bill, 2018

Explanation:-The Parliament passed the Right of Children to Free and Compulsory Education

(Amendment) Bill, 2018 that seeks to do away with the no-detention policy in schools. The Right of

Children to Free and Compulsory Education (Amendment) Bill, was moved in the Rajya-Sabha by Union

minister for Human Resources Development Prakash Javadekar and was passed by a voice vote. It is already

passed by the Lower House (Lok-Sabha) on July 18 2018. Under current provisions of the Section 16 of the

Act, no student can be detained up to class 8th but as per the amendment, it would be left to the States to

decide whether to continue the no-detention policy. 25 states have favoured abolishing the no-detention

policy. The bill provides for regular examination in classes V and VIII. The MHRD Minister assured that no

child will be excluded from school in case he or she fails and such children will be provided two-month

remedial teaching to perform better in another examination to be conducted by their schools. If the students

still do not pass the exam, the state government may decide to detain them.

9)Which city is the venue of the 2019 Indian Panorama Film Festival (IPFF)?

Ans:-New Delhi

Explanation:-The Indian Panorama Film Festival (IPFF) has started at Siri Fort Auditorium II, New Delhi

from 4th January. The 10-day Festival is being organized by the Directorate of Film Festivals, Ministry of

Information & Broadcasting (I&B). The directors of the opening feature film 'Olu' and of the opening non-

feature film 'Kharvas' viz. Shri Shaji N Karun and Shri Aditya Suhas Jambhale were presented at the

inauguration of the fest. All the films selected under the Indian Panorama section in the 49th International

Film Festival of India (IFFI-2018) will be screened during the Festival. The entry for the Festival is free and

seating is on the first-come-first-served basis.

10)The 80th National Table Tennis Championship begins in which city?

Ans:-Cuttack

Explanation:-The 80th National Table Tennis Championship began at Jawaharlal Nehru Indoor Stadium in

Cuttack. As many as 35 teams, including 550 players from different parts of the country are participating in

the mega championship.

11)Chhabilendra Roul was appointed as the Secretary for which of the following department?

Ans:- Department of Fertilizers

Explanation:-Chhabilendra Roul has assumed the charge of Secretary, Department of Fertilizers, Ministry

of Chemicals & Fertilizers from 31 December 2018. He replaced Bharathi S. Sihag, who retired on 31

December 2018. He was the Special Secretary of Department of Agricultural Research and Education. He is

an Indian Administrative Services Officer of 1985 batch, from Punjab Cadre.

12)The Ministry of Human Resource Development constituted a Task Force to look into the JNV

suicides. It was headed by __________.

Ans:-Dr. Jitendra Nagpal

Explanation:-The Ministry of Human Resource Development constituted a Task Force to look into the

reasons of suicides reported in the JNVs. The Task Force will be headed by Psychiatrist, Dr. Jitendra

Nagpal. The Jawahar Navodaya Vidyalayas (JNVs) are fully residential schools managed and run by the

Page 20: Monthly Current Affairs of SEPTEMBER - SSCE

SSCE 8981426494/8296260082 Downloaded from :- www.onlinessce.com Page 20

Navodaya Vidyalaya Samiti, an autonomous organisation under the Ministry of Human Resource

Development.

13)Tropical Storm Usman hits this country with heavy rains and triggered deadly landslides, on 29th

December 2018.

Ans:-Philippines

Explanatiobn:-Tropical Storm Usman hit Philippines' Bicol region in Manila. The tropical depression

brought heavy rains and triggered deadly landslides all over the country. The Philippines is affected by 15 to

20 typhoons every year during the rainy season, which usually begins in May or June and concludes in

November or December.

14) Name the Indian batsman who has faced highest deliveries in a test series in Australia with 1258

balls, beating India's Former Captain, Rahul Dravid's record of 1203 balls.

Ans:- Cheteshwar Pujara

Explanation:-30 years old Indian Batsman, Cheteshwar Pujara has faced 1258 balls in the ongoing test

series so far against Australia to become the batsman who had faced highest deliveries in a test series in

Australia beating India's Former Captain, Rahul Dravid's record of 1203 balls. He achieved the feat by

scoring 193 runs of 373 deliveries on the 2nd day of 4th Test Match against Australia in Sydney.

15) Name the Indian origin boy who was selected by Indian Council for Child Welfare (ICCW) for

receiving National Bravery Award.

Ans:-Wahengbam Lamnganba Singh

Explanation:-15-year-old class IXth student, Wahengbam Lamnganba Singh, of Kumbi area in Manipur,

has been selected by the Indian Council for Child Welfare (ICCW) for receiving National Bravery Award.

He will be honoured because he had saved his two and half year's old cousin brother from drowning in a

pond, on May 16, 2018. He would receive the award from Prime Minister Narendra Modi in Delhi. The

National Bravery Awards are given under the aegis of the Indian Council for Child Welfare (ICCW) every

Republic Day. These awards are broadly classified into five categories, including the Bharat Award, Sanjay

Chopra Award, Geeta Chopra Award, Bapu Gaidhani Award and the General National Bravery Awards.

Daily Current Affairs 7th January, 2019

1)Which country successfully tested its Mother of All Bombs?

Ans:-China

Explanation:-China successfully tested its most powerful non-nuclear weapon, dubbed as the Mother of All

Bombs. The bomb, whose strength is only second to that of a nuclear weapon, was dropped by a Chinese H-

6K bomber. The bomb is five to six metres long (16 to 20 feet), but weighs less than the American version.

In a short video published on the website of state arms manufacturer Norinco, the massive bomb drops on to

a plain and produces a gigantic ball of fire and black smoke.

2)The government has amalgamated how many Regional Rural Banks into a single RRB?

Ans:-3

Explanation:-The government has amalgamated three Regional Rural Banks - Punjab Gramin Bank, Malwa

Gramin Bank and Sutlej Gramin Bank - into a single RRB. The sponsor banks of the RRBs are National

Bank for Agriculture and Rural Development (NABARD), the Government of Punjab and Punjab National

Bank, State Bank of India and Punjab and Sind Bank.

3)Who signs MoU With SAMEER for Boosting Communication?

Ans-AAI

Page 21: Monthly Current Affairs of SEPTEMBER - SSCE

SSCE 8981426494/8296260082 Downloaded from :- www.onlinessce.com Page 21

Explanation:-Airports Authority of India (AAI) signed MoU with Society for Applied Microwave

Electronics Engineering and Research (SAMEER) for joint research programs in the field of

Communication Navigation and Surveillance (CNS). The MOU, on the one hand, will provide for the

transfer of skill by SAMEER experts in the areas of Radio Frequency, Millimetre Wave and Antenna-related

solutions to AAI domain experts, while on the other hand, it will pave way for exposure for the SAMEER

researchers to the challenging Aviation field.

4)Which state CM announced interest-free loans of up to Rs. 3 lakh for women self-help groups

(WSHGs)?

Ans:-Odisha

Explanation:-Odisha Chief Minister Naveen Patnaik announced interest-free loans of up to Rs. 3 lakh for

women self-help groups (WSHGs) in the state. There are around six lakh WSHGs in Odisha. He stated the

initiative will benefit about 70 lakh women. The chief minister also distributed financial assistance of Rs.

3,000 each to six lakh WSHGs as part of the state government's efforts to digitally empower women. During

the convention, Mr. Patnaik launched a scheme to distribute Rs. 15,000 seed money per group to three lakh

new self-help groups. The chief minister announced incentives of Rs. 500 per month for Anganwadi workers

and Rs. 200 for helpers.

5)The rate of reservationapproved by Union Cabinet for 'economically backward' upper castes in

government jobs.

Ans:-10%

Explanation:-The Union Cabinet approved a 10% reservation for 'economically backward' upper castes in

government jobs. Ahead of the General Elections this year, the government has announced reservation in

jobs for those belonging to the upper castes and earning less than Rs 8 lakh per year. A bill to amend Article

15 and 16 of the constitution will be moved in Parliament to this effect.

6)Who was appointed as the new Chief Information Commissioner?

Ans:-Sudhir Bhargava

Explanation:-Centre has appointed Sudhir Bhargava as the new Chief Information Commissioner (CIC).

Four new members have also been appointed to CIC, taking its strength to seven. Four vacancies still

remain. Mr. Bhargava, a former Secretary to the Social Justice Ministry, has been a Commissioner since

June 2015. He will be the ninth chief of the CIC. The CIC is the highest appeal body under the Right to

Information Act. It has been functioning with just three members since the former chief and several

members completed their tenure last month.

7)Govt Allocates Rs __________ Cr for Development of 40 Satellite launch vehicles for the next 4

years.

Ans:-10,900

Explanation:-Indian Space Research Organisation (ISRO) Chairman Dr. K Sivan stated that the Union

Government has allocated 10,900 crore rupees for the development of 40 satellite launch vehicles in the next

4 years. Dr. Sivan also stated that Chandrayan Mission will be launched within three months. It will land in

one part of the Moon which is still unexplored. He also said that the human spaceflight Gaganyaan is

planned to be launched by 2022, the 75th anniversary of Independence.

8)Which amendment bill will allow individuals to offer voluntarily biometric ID for opening a bank

account and getting a mobile phone connection?

Ans:-Aadhaar and Other Laws (Amendment) Bill, 2018

Explanation:-The Lok Sabha passed a Bill to amend the Aadhaar and Other Laws (Amendment) Bill, 2018.

The amendment will allow individuals to offer voluntarily biometric ID as a means of identity verification

for obtaining services such as opening a bank account and getting a mobile phone connection. The Bill also

Page 22: Monthly Current Affairs of SEPTEMBER - SSCE

SSCE 8981426494/8296260082 Downloaded from :- www.onlinessce.com Page 22

gives a minor the option to opt out of the 12-digit identity scheme on attaining the age of 18. It provides for

stiff penalties for violation of norms set for the use of Aadhaar and bans service providers from storing core

biometric information and Aadhaar number of individuals who voluntarily offer it as a means of

authentication.

9)Pratap Simha demands change in IBPS recruitment pattern recently. He was the the MP of

__________.

Ans:-Karnataka

Explanation:-Karnataka MP Pratap Simha on Union Finance Minister Arun Jaitley and requested him to

review the existing recruitment policy of Institute of Banking Personnel Selection (IBPS) In an official

communication Simha urged Jaitley to reinstate the provision of Local language' which was dropped in 2015

notification of the Central recruitment agency IBPS. Referring to the language barriers in the banking sector,

the Mysuru-Kodagu MP observed that officers from other states who are not well-versed with Kannada are

facing trouble in daily business. Simha wrote that notification of 2012-14 which demands the knowledge of

local language should be reinstated to resolve the problems faced by local job aspirants.

10)When was the World Day for War Orphans observed?

Ans:-January 6

Explanation:-The World Day of War Orphans was initiated by the French organisation, SOS Enfants en

Detresses. Held on 6th January each year, this special day enables the International Community to recognise

the plight of a particularly vulnerable group. The usual definition of an orphan is a child who has no

surviving parent to care for him or her, having lost both parents, either as a result of bereavement or by

being abandoned.

11)Who unveiled 6 projects worth Rs 5,300 crore in Jodhpur, recently?

Ans:-Shri Nitin Gadkari

Explanation:-Union Road Transport and Highways Minister Nitin Gadkari inaugurated six road projects,

claiming that these roads won't have potholes for the next 150-200 years. While at the event, he also

emphasized on the use of bio-fuel, electricity and ethanol as an alternatives to the conventional fossil fuel.

12)Which country wins the first-ever cricket Test series in Australia?

Ans:-India

Explanation:-India won the four-match Test series against Australia 2-1 as the Sydney Test ended in a draw

after the fifth day's play was washed out due to rain. This is the first time ever that India have won a Test

series in Australia. Before this, India had played 11 Test series in Australia since 1947, losing eight and

drawing three. Cheteshwar Pujara, who made 193 runs in the Sydney Test, was the Man of the Match and

Series.

13)Name the journalist who was conferred with the lifetime achievement award for his lifelong

contributions in journalism, by the Mantralaya and Vidhimandal Vartahar Sangh.

Ans:-Dinu Randive

Explanation:-Veteran journalist Dinu Randive was conferred with the lifetime achievement award for his

lifelong contributions in journalism, who started his journalistic career in 1956. He also participated in the

freedom struggle, the Sanyukta Maharashtra movement and the Goa liberation movement. The award was

given by the Mantralaya and Vidhimandal Vartahar Sangh which is an association of journalists covering

Maharashtra legislature and secretariat. Award winners in other categories included Vishwas Waghmode,

Mahesh Tiwari and Prajakta Pol.s

14). Who is First Tycoon to be named "Fugitive Economic Offender"?

Ans:-Vijay Mallya

Page 23: Monthly Current Affairs of SEPTEMBER - SSCE

SSCE 8981426494/8296260082 Downloaded from :- www.onlinessce.com Page 23

Explanation:-Vijay Mallya became the first tycoon to be charged under a new anti-fraud law, with a court

in Mumbai naming him as an offender under the Fugitive Economic Offenders Act, 2018. The anti-

corruption court was hearing an application by the Enforcement Directorate, seeking a direction to declare

Mr. Mallya a fugitive economic offender.

15)Who pesented the Ramnath Goenka Excellence in Journalism awards recently?

Ans:-Rajnath Singh

Explanation:-Home Minister Rajnath Singh presented Ramnath Goenka Excellence in Journalism awards

in 18 categories across print, broadcast, and purely-digital for outstanding work done in 2017. The Express

Group instituted the Ramnath Goenka Excellence in Journalism Awards in 2005 as part of the centenary

year celebrations of its founder, Ramnath Goenka. The awards aim to celebrate excellence in journalism,

recognize courage and commitment and showcase the outstanding contributions of journalists from across

the country.

16)For which state, National Green Tribunal imposed a fine of 100 crore rupees for its failure to curb

illegal coal mining in the state?

Ans:-Meghalaya

Explanation:-National Green Tribunal imposed a fine of 100 crore rupees on Meghalaya government for its

failure to curb illegal coal mining in the state. The amount of the fine has to be deposited with the Central

Pollution Control Board within two months. The action comes after a high-level committee submitted an

adverse report to a bench headed by NGT chairperson AK Goel. The report stated that the majority of the

mines in the state are operating without a lease or license. Senior advocate Raj Panjawani is assisting the

tribunal as an amicus curiae in the matter. He said, during the hearing, Meghalaya government admitted that

a large number of mines are operating illegally.

17)Roger Federer and __________ won a Hopman Cup and retain a Title for Switzerland.

Ans:-Belinda Bencic

Explanation:-Switzerland's Roger Federer and Belinda Bencic secured victory over Germany's Angelique

Kerber and Alexander Zverev to win the Hopman Cup for the second year in a row. In a rerun of 2018 final,

Federer beat Zverev to give the Swiss an early 1-0 lead. Wimbledon champion Kerber then overcame

Bencic to level the tie. However, the Swiss claimed a tight win in the mixed doubles to claim their fourth

Hopman Cup title.

18)The government of India issued an __________ alert for the Andaman and Nicobar Islands due

Cyclonic storm 'Pabuk'.

Ans:-Orange

Explanation:-The government of India issued an 'Orange' alert for the Andaman and Nicobar Islands due

Cyclonic storm 'Pabuk' on 6 January 2019. 'Pabuk' is now over the Andaman sea and its neighbourhood. An

'orange' weather warning means people should be prepared and there is an increased likelihood of bad or

extreme weather which may disrupt road and air travel and threaten life and property.

19)The government formed a High-Level committee for implementation of Clause 6 of Assam Accord.

It was headed by

Ans:-M. P. Bezbaruah

Explanation:-The government had notified a High-Level committee for implementation of Clause 6 of

Assam Accord. Retired IAS officer M.P. Bezbaruah is chairman of the 9-member Committee. The

committee will examine the effectiveness of actions since 1985 to implement Clause 6 of the Assam Accord.

The clause envisaged that appropriate constitutional, legislative and administrative safeguards, should be

provided to protect, preserve and promote the cultural, social, linguistic identity and heritage of the people

of Assam.

Page 24: Monthly Current Affairs of SEPTEMBER - SSCE

SSCE 8981426494/8296260082 Downloaded from :- www.onlinessce.com Page 24

20) Which railway station became the first railway station of South Central Railway (SCR) zone to

hoist a huge tricolour flag on a 100-foot tall pole, on 2nd January 2019?

Ans:-Secunderabad Railway Station

Explanation:-Secunderabad Railway Station, in Hyderabad, became the first railway station of South

Central Railway (SCR) zone to hoist a huge tricolor flag on a 100-foot tall pole. This was done according to

the Railway Board's order to hoist the national flag at 75 A1 category railway stations. The flag would be

guarded by the Railway Protection Force (RPF). Furthermore according to the order, SCR has taken the

initiative to hoist the National Flag at 4 other 'A1' category Railway Stations - Kacheguda, Hyderabad,

Vijaywada and Tirupati.

Daily Current Affairs 8th January, 2019

1)The Central Government has given nod for Centre for Classical Language in which city of Kerala?

Ans:-Tirur

Explanation:-The Union Ministry of Human Resource Development (HRD) has recently approved the

establishment of a Centre for Classical Language at the Thunchath Ezhuthachan Malayalam University in

Tirur, Kerala. The Ministry has communicated the decision to the Central Institute of Indian Languages,

Mysuru. In view of this, the State government would be required to prepare a detailed project report and sign

a Memorandum of Understanding (MoU) with the Ministry. The aim of the proposed centre is to study and

research and other aspects of Malayalam language & will prove beneficial in implementing various projects

for Malayalam language development. The centre would also assist the Malayalam University in its various

initiatives.

2)The Polavaram irrigation project created two Guinness world records for 'pouring the highest

amount of concrete' and 'largest continuous concrete pour in 24 hours'. This belongs to the state of;

Ans:-Andhra Pradesh

Explanation:-The Polavaram irrigation project in Andhra Pradesh created two Guinness world records for

'pouring the highest amount of concrete' and 'largest continuous concrete pour in 24 hours'. The Polavaram

irrigation project is located in West Godavari district of Andhra Pradesh. The project has entered the

Guinness World Records for pouring 32,100 cubic meters of concrete non-stop in 24 hours. The total storage

capacity of the water reservoir is 120,000 million cubic feet (TMC). The current record is held by a Dubai-

based contracting group which poured 21,580 cu m of concrete in about 35 hours.

3)The fourth edition of the Raisina Dialogue has begun in __________.

Ans:-New Delhi

Explanation:-The fourth edition of the Raisina Dialogue has begun in New Delhi. Norway Prime Minister

Ms. Erna Solberg delivered inaugural address in presence of PM Narendra Modi and External Affairs

Minister Sushma Swaraj. The theme of the Dialogue this year is "A World Reorder: New Geometries; Fluid

Partnerships; Uncertain Outcomes". Ministry of External Affairs in partnership with Observer Research

Foundation has organized the flagship annual geopolitical and geostrategic conference.

4)The first Indian state that has established a Brahmin Corporation aimed to help poor people of

Brahmin families.

Ans:-Andhra Pradesh

Explanation:-Andhra Pradesh Chief Minister Chandrababu Naidu unveiled various welfare scheme for the

marginalized and economically backward Brahmins through AP Brahmin Welfare Corporation such as

distribution of Cars to the Unemployed Brahmin youths, loans and subsidies. Brahmin youths who had

undergone training as drivers were given Maruti Suzuki Swift Dzire for the first time. 30 cars were

distributed on 4th January to the Brahmin youths. Youth bore 10% of the total on-road cost the diesel run

cars. Government has started at least half dozen schemes for Brahmins and around 10000 poor Brahmins

Page 25: Monthly Current Affairs of SEPTEMBER - SSCE

SSCE 8981426494/8296260082 Downloaded from :- www.onlinessce.com Page 25

have been given loans and subsidies. Andhra Pradesh is the first state of the country which has established a

Brahmin Corporation which aims to help poor people of Brahmin families by providing financial assistance

for education, coaching, entrepreneurship, skill development and welfare and culture.

5)Vice-Chairman of Rajya Sabha, Harivansh Narayan Singh launched a book titled "A Crusade

Against Corruption" at the Press Club of India in New Delhi. The Author of the book is;

Ans:-Manohar Manoj

Explanation:-Vice-Chairman of Rajya Sabha, Harivansh Narayan Singh launched Manohar Manoj`s book

'A Crusade Against Corruption' at the Press Club of India in New Delhi.

6)Srei Equipment Finance and this bank entered into an agreement for financing the Micro, Small

and Medium enterprises (MSME) and agriculture sector.

Ans:-UBI

Explanation:-The United Bank of India (UBI) and Srei Equipment Finance have entered into an agreement

for financing the Micro, Small and Medium enterprises (MSME) and agriculture sector. Under the

agreement UBI plans to leverage the co-lending model to offer credit at blended interest rate of 11-12 per

cent. The Bank will leverage low its low cost loans with Srei Equipment finance to enable purchase of

equipment and vehicle in the seamless manner. The Bank is eying around 200 crore business over the period

of three months by this agreement.

7Which state government announced distribution of free sops to encourage farmers to enroll in the

Atal Solar Krishi Pump Yojana?

Ans:-Maharashtra

Explanation:-Maharashtra Government announced distribution of free sops to encourage farmers to enrol in

the Atal Solar Krishi Pump Yojana. The things included: two LED bulbs, a DC fan and a mobile charging

socket.

8)Who won a Best Actor in Motion Picture-Drama of 76th Annual Golden Globe Awards?

Ans:-Rami Malek

Explanation:-The 76th Annual Golden Globe Awards were announced at the Beverly Hilton Hotel in

California, the USA. These annual awards honour the best in Motion Pictures and Television for the year

2018. Rami Malek won a title of Best Actor in Motion Picture-Drama for Bohemian Rhapsody. And

Bohemian Rhapsody won a Best Motion Picture Drama in the 76th Annual Golden Globe Awards.

9)Who conferred conferred textiles awards in New Delhi and presented commemorative stamps on GI

registered handicrafts

Ans:-Shri Venkaiah Naidu

Explanation:-The Vice President of India, Shri M Venkaiah Naidu conferred awards on 17 persons for their

outstanding contribution in different fields of textiles sector during the day long 'National Conclave on

Accomplishments and Way Forward for Textiles Sector' in New Delhi. The national conclave which was

organised to address the issues faced by textile sector was inaugurated by the Union Textiles Minister,

Smriti Irani and was also addressed by the Minister of State, Ajay Tamta and Textiles Secretary,Raghvendra

Singh. During the ceremony Vice President, Shri Naidu was presented commemorative stamps on GI

registered handicrafts. The inaugural session was followed by panel discussions on technical textiles, ease of

doing textiles business, handicrafts, access to global markets and supply chains and fashionizing weaves.

10)Which ministry has introduced a 70-point grading index to assess the quality of school education

offered by states?

Ans:-Ministry of Human Resource Development

Page 26: Monthly Current Affairs of SEPTEMBER - SSCE

SSCE 8981426494/8296260082 Downloaded from :- www.onlinessce.com Page 26

Explanation:-The HRD ministry has introduced a 70-point grading index to assess the quality of school

education offered by states. A state performance grading will be done wherein the states will be marked out

of 1,000 scores on 70 parameters. HRD Minister Prakash Javadekar stated that it will give the correct picture

of where every state stands. There will be a fair competition to improve each other's performances.

11)Who Resigns as a President of The World Bank?

Explanation:-Jim Yong Kim

Explanation:-World Bank President, Jim Yong Kim has made the surprise announcement that he is

stepping down after six years in the post. His resignation will take effect from 1 February. Mr. Kim, 59, was

not due to leave until 2022 after he was re-elected for a second five-year term in 2017. The World Bank

stated that he will join a firm and focus on increasing infrastructure investments in developing countries.

12)The second edition of Khelo India Youth Games will starts in __________, Maharashtra.

Ans:-Pune

Explanation:-Maharashtra is all set to host the second edition of Khelo India Youth Games which will

officially begin tomorrow in Pune. More than 9000 players from different states will participate in 18

different games at Khelo India Youth Games. It has already been announced that Youth Olympic Gold

Medallist in shooting Manu Bhaker and Saurabh Chaudhari, as well as weightlifter Jeremy Lalrinnunga, will

be among the cream of India's youth sporting talent competing at these games. Three 10-year-old athletes

including footballer Pratima Kumar from Jharkhand, hockey player Laltlanchhungi from Mizoram and West

Bengal's 10m air rifle shooter Abhinav Shaw, will pick up their first experience of competing in a large-

scale multi-discipline event.

13)Who among the following was sworn as the new Chief Justice of Nepal's Supreme Court?

Ans:-Justice Cholendra Shamsher J B Rana

Explanation:-Justice Cholendra Shamsher J B Rana took over as the new Chief Justice of Nepal's Supreme

Court. Nepal's President Bidya Devi Bhandari administered the oath to Rana at the swearing-in ceremony at

Sheetal Niwas, the official residence of the President. Rana, who will have a tenure of four years, has

succeeded Chief Justice Om Prakas Mishra.

14)Which country declared the ancient Hindu religious site of Panj Tirath as national heritage?

Ans:-Pakistan

Explanation:-The provincial Khyber Pakhtunkhwa government in northwest Pakistan has declared the

ancient Hindu religious site of Panj Tirath in the provincial capital Peshawar as national heritage. The

announcement which stated that a fine of 2 million Pakistani rupees and five years imprisonment to be

awarded to anyone found guilty of damaging the historic site was made under the Khyber Pakhtunkhwa

Antiquities Act of 2016. 'Panj Tirath' is named for the five pools of water present at the site and houses a

Hindu temple "Shawal Thakardawara" and a lawn with date palm trees. Devotees believe that mythical king

Pandu from the epic Mahabharata bathed in the pools during the month of Kartik and worshipped under the

date palm trees in the area for two days. The site was damaged during the reign of the Afghan Durrani

dynasty in 1747, however it was restored by local Hindus during the period of Sikh rule in 1834 and worship

started again.

15) Who has been appointed as the director general of the Sashastra Seema Bal (SSB)?

Ans:-Shri. Kumar Rajesh Chandra

Explanation:-Civil Aviation Security head and senior IPS officer Kumar Rajesh Chandra was appointed as

the director general of the Sashastra Seema Bal (SSB).Chandra, a 1985-batch IPS officer of Bihar cadre, is

at present the director general of the Bureau of Civil Aviation Security (BCAS). He has been appointed as

SSB DG up to December 31, 2021.

Page 27: Monthly Current Affairs of SEPTEMBER - SSCE

SSCE 8981426494/8296260082 Downloaded from :- www.onlinessce.com Page 27

16)Name the Indian women who has joined International Monetary Fund (IMF) as its chief

economist, recently.

Ans:-Gita Gopinath

Explanation:-Mysore-born Gita Gopinath has joined International Monetary Fund (IMF) as its chief

economist, becoming the first woman to occupy the top IMF post. She succeeds Maurice Obstfeld, who

retired on December 31. Ms Gopinath considers the perceived retreat from globalisation as one of the top

challenges being faced by the IMF.

17) Six professors awarded Infosys Prize 2018 for science and research in __________.

Ans:-Bengaluru

Explanation:-The global software major's Infosys Science Foundation awarded its 10th Infosys Prize 2018

to six eminent professors for winning across different categories of science and research at an awards

ceremony in Bengaluru. The Infosys annual award includes a pure gold medal, a citation and a tax-free prize

purse worth $100,000 (or its equivalent in Indian rupees). The winners were selected by a six-member jury

of renowned scientists and professors, namely Kaushik Basu, Pradeep Khosla, Shrinivas Kulkarni, Amartya

Sen, Mriganka Sur, and Srinivasa Varadhan from 244 nominations received in six categories.

Daily Current Affairs 9th January, 2019

1)Recently, Airport Authority of India (AAI) banned single-use plastic items for how many airports?

Ans:- 16

Explanation:-Airport Authority of India (AAI) banned single-use plastic items at 16 airports. This entails

air travellers not getting any single-use plastic items like straws, plastic cutlery and plastic plates etc at

passenger terminals and city side. The 16 airports declared as single-use plastic free are: Indore, Bhopal,

Ahmedabad, Bhubaneswar, Tirupati, Trichy, Vijayawada, Dehradun, Chandigarh, Vadodara, Madurai,

Raipur, Vizag, Pune, Kolkata and Varanasi. The first batch of 16 airports was selected on the basis of third-

party assessment carried out by Quality Council of India.

2)Which among the followinh collaborate with Oil Marketing Companies for issuance of FASTags

through petroleum retail outlets?

Ans:-National Highway Authority of India

Explanation:-Indian Highways Management Company Ltd. (IHMCL), a company promoted by NHAI, has

signed MOUs with state-run Oil Marketing Companies (IOCL, BPC, and HPC). It will ensure availability of

FASTags at petroleum outlets across India. Indian highways management company ltd. (IHMCL) had

launched the national electronic toll collection program (NETC) under the brand name "FASTag" in April

2016. This program has gain success and running at 450 toll plaza along with national highway and selected

state toll plazas. The enhanced adoption of FASTag program will increase user convenience by offering

seamless toll-fee payments at Plazas and thus help save time, money and fuel. IHMCL is also planning to

launch two mobile applications named MYFASTag and IHMCLPOS , will enable customers to link the

FASTags with their preferred bank account.

3)Who is the recipient of the Ramnath Goenka Excellence in Journalism Award for Investigative

Reporting(Print category)?

Ans:-S Vijay Kumar

Page 28: Monthly Current Affairs of SEPTEMBER - SSCE

SSCE 8981426494/8296260082 Downloaded from :- www.onlinessce.com Page 28

Explanation:-S. Vijay Kumar, Senior Deputy Editor with The Hindu in Chennai, was presented the

Ramnath Goenka Excellence in Journalism Award for Investigative Reporting (Print category) for his

reports on how taxmen alerted officials about payoffs in the Tamil Nadu gutka scam - but were ignored. The

award was presented by Union Home Minister Rajnath Singh at a ceremony in New Delhi on January 7. The

award for Investigative Reporting in the broadcast category was won by Anand Kumar Patel of India Today

TV. The awards for Political Reporting went to Brajesh Rajput of ABP News Network and Sushant Kumar

Singh of the Indian Express. The award in the Environmental Reporting category were presented to Sushil

Chandra Bahuguna of NDTV and Sandhya Ravishankar of the online magazine The Wire.

4)Which city is the venue o f the All India Radio National Poets Conference 2019?

Ans:-Chennai

Explanation:-The All India Radio National Poets Conference will be held in Chennai on January 10, 2019.

It will be inaugurated by the Tamil Nadu Governor Bunwarilal Purohit. This is the first time the city would

be hosting the Sarva Basha Kavi Sammelan since its launch in 1956. In it, 22 poets representing as many

languages included in the eighth schedule of the Constitution would be presenting their poems. Scholars in

Hindi and Tamil will present the translated version of the poems in the Conference itself. The programme

will be aired on the 25th of this month.

5)Which company introduces Kumbh Jio Phone app for pilgrims?

Ans:-Reliance Jio

Explanation:-Reliance Jio Infocomm launched the Kumbh JioPhone application, specially designed for

millions of pilgrims who will congregate during this year's Ardh Kumbh Mela at Prayagraj. The new app

which will be available to both existing and new JioPhone users will contain information on Kumbh, real-

time travel information (special trains, buses etc.), booking tickets and receiving updates, 'Yatri Ashray' at

stations, emergency helpline numbers, area routes and maps, pre-published bath and religious day schedules

and railway camp mela. The new app has a feature called 'Khoya Paya' that will help users stay close to near

and dear ones by finding their exact location.

6)Who has taken charge as India's new ambassador to China?

Ans:-Vikram Misri

Explanation:-India's new ambassador to China, Vikram Misri, took charge of the post. He reported to Hong

Lei, Deputy Director General of Protocol at the Ministry of Foreign Affairs in Beijing. He is a 1989-batch

India Foreign Service (IFS) officer, and replaced Gautam Bambawale who retired in November 2018. Prior

to this appointment, Misri served as Indian envoy in Myanmar. Earlier, he has served in various capacities at

the headquarters of the Ministry of External Affairs (MEA) as well as in the Prime Minister'ss Office.

7)Which country opened West Asia's largest cathedral in its New Administrative Capital?

Ans:-Egypt

Explanation:-Egypt opened West Asia's largest cathedral in its New Administrative Capital, 45 kms away

from its capital, Cairo. President Abdul Fattah al-Sisi inaugurated the newly-built Cathedral of Nativity on

the eve of Coptic Christmas.

8)The GST panel approved levy of 1%'calamity cess' by Kerala for a period of two years to fund

rehabilitation work in the state of Kerala. It was led by;

Page 29: Monthly Current Affairs of SEPTEMBER - SSCE

SSCE 8981426494/8296260082 Downloaded from :- www.onlinessce.com Page 29

Ans:-Sushil Kumar Modi

Explanation:-The Group of States' Finance Ministers (GoFM) led by Deputy Chief Minister of Bihar,

Sushil Kumar Modi approved levy of 1%'calamity cess' by Kerala for a period of two years to fund

rehabilitation work in the state of Kerala. It will recommend the same to the GST council meeting which

will be held in January 10, 2019. The goods and services, which will attract the 1% cess, would be decided

by Kerala and any other state considering to do the same would have to approach the council. It also

suggested the GST council to allow additional borrowing over the permitted limit by states hit by natural

calamity.

9)Reserve Bank of India (RBI) has constituted a High-Level Committee on Deepening of Digital

Payments to encourage digitisation of payments and enhance financial inclusion. It was headed by;

Ans:-Nandan Nilekani

Explanation:-Reserve Bank of India (RBI) has constituted a High-Level Committee on Deepening of

Digital Payments to encourage digitisation of payments and enhance financial inclusion. The five-member

committee headed by UIDAI's former Chairman Nandan Nilekani will include RBI's former Deputy

Governor Mr. H.R. Khan and Former Secretary, Ministry of Information Technology, Mrs. Aruna Sharma

among others. RBI stated that the committee will review the existing status of digitisation of payments and

suggest ways to bridge any gaps in the ecosystem if any. The committee will also undertake cross country

analysis to identify best practices that can be adopted to accelerate digitisation of the economy and financial

inclusion through greater use of digital payments.

10)The bill cleared by Union Cabinet to provide Indian citizenship to non-Muslims from Bangladesh,

Afghanistan and Pakistan is;

Ans:-Citizenship Amendment Bill

Explanation:-The Union Cabinet chaired by Prime Minister Narendra Modi cleared the redrafted

Citizenship Amendment Bill thus providing Indian citizenship to non-Muslims from Bangladesh,

Afghanistan and Pakistan. First introduced in Parliament in 2016, the bill seeks to amend the Citizenship Act

1955. It enables grant of Indian nationality to people from minority communities - Hindus, Sikhs, Buddhists,

Jains, Parsis and Christians- from Afghanistan, Bangladesh and Pakistan after six years of residence in India

instead of 12 even if they don't possess any proper document. This would, however, nullify the 1985 Assam

Accord under which any foreign national, irrespective of religion, who had entered the state after 1971

should be deported.

11)With which country India signed an MoU on Ocean Dialogue?

Ans:-Norway

Explanation:-Prime Minister of Norway Erna Solberg and Prime Minister Narendra Modi held bilateral

talks in New Delhi. Prime Minister Modi said that both the countries have vibrant trade ties. MoU were

exchanged on India-Norway Ocean Dialogue. Erna Solberg also called on President Ram Nath Kovind and

Vice President M Venkaiah Naidu. She also delivered the Inaugural address at the Raisina Dialogue and

addressed India-Norway Business Summit in New Delhi.

12)Global Aviation Summit 2019 will be held in __________ with the theme "Flying for all".

Ans:-Mumbai

Page 30: Monthly Current Affairs of SEPTEMBER - SSCE

SSCE 8981426494/8296260082 Downloaded from :- www.onlinessce.com Page 30

Explanation:-In a first, the 2-day long Global Aviation Summit 2019 with the theme "Flying for all" will be

organized by the Ministry of Civil Aviation (MoCA) in collaboration with FICCI from January 15-16, 2019

in Mumbai, Maharashtra. The summit is supported by International Civil Aviation Organization (ICAO),

Federal Aviation Administration of US (FAA), International Air Transport Association (IATA), CiviI Air

Navigation Services Organisation (CANSO), Airports Council International (ACl) and Association of Asia

Pacific Airlines (AAPA).

13)Who announces cash awards for triumphant Team India members?

Ans:-BCCI

Explanation:-Board of Control for Cricket in India (BCCI) announced cash awards of Rs 15 lakh per match

for each of the playing eleven that featured in India's maiden Test Cricket series win on Australian soil.

Virat Kohli and his team defeated Australia 2-1 in the four-match Test series, ending India's 71-year-old

wait for the rare achievement. Congratulating the team, the BCCI also announced cash awards of seven and

a half lakh rupees for all the reserve players, as well as the members of the support staff.

14)Name the mobile application launched by North Central Railway (NCR) to help navigate the

attendees of Kumbh Mela to be held in Prayagraj city.

Ans:-Rail Kumbh Seva Mobile App

Explanation:-The North Central Railway, (NCR) launched a mobile application the 'Rail Kumbh Seva

Mobile App' to help navigate the attendees of the Kumbh Mela in Prayagraj city from January 15- March 4,

2019. It will also provide information regarding all the 'Mela special' trains that will be run during the

period. The app will provide a link to the user to buy both unreserved and reserved train tickets. Also, the

users will not only get to know their current location, but will also be able to reach to all railway stations, the

mela zone, major hotels, bus stations and other facilities within Prayagraj. A photo gallery, containing

pictures of previous Kumbh Melas can also be accessed through the app.

15)Name the small Finance Bank Launches Current Account with Unique Auto-Sweep Facility.

Ans:-Jana Small Finance Bank

Explanation:-Jana Small Finance Bank launched Current Account with Auto-Sweep facility. The facility

will enable bank's existing and potential new customers to auto sweep-in and sweep-out the idle funds in the

current account to Sweep Fixed Deposit and earn high interest. For a tenor of 365 days on sweep deposit, the

customers will be able to earn interest of 8.5 percent.

16)Who becomes the 1st Indian to Head Global Solar Council?

Ans:-Pranav R Mehta

Explanation:-A visionary in the field of Solar Energy and the chairman of National Solar Energy

Federation, Pranav R Mehta, has taken over as the president of Global Solar Council (GSC). He has become

the first Indian to head the GSC. The Global Solar Council (GSC) was launched on December 6, 2015,

following the historic United Nations Climate Change Conference (UN COP 21). The GSC came into being

as International Coalition of more than 30 nations, utilising maximum solar energy, decided to harness the

renewable energy for the greater good.

17)Indian Army signed an MoU on the Defence Salary Package with which of the following bank?

Page 31: Monthly Current Affairs of SEPTEMBER - SSCE

SSCE 8981426494/8296260082 Downloaded from :- www.onlinessce.com Page 31

Ans:- Punjab National Bank

Explanation:-An MoU was signed between the Indian Army and Punjab National Bank on the Defence

Salary Package. The signing ceremony was chaired by the Adjutant General, Lt Gen Ashwani Kumar.

Salient features of the improved and updated MoU are free 'RAKSHAK PLUS' Scheme for serving pers and

veterans, overdraft facility range from Rs 0.75 to 3.0 lakhs, multicity 'At Par' cheques, instant credit of

outstation cheques, preferential allotment of lockers at concessional AMC and free SMS alerts.

18)This country passed law to sinicize and make Islam compatible with socialism.

Ans:-China

Explanation:-China passed law to make Islam compatible with socialism and 'sinicize' it within next five

years. The law was passed after Chinese government officials met representatives from eight Islamic groups.

In 2018, China held over a million people of the ethnic Uighur and Muslim minorities in prison in the

Xingjian region.

19)Choose the one which is the recepient of NBFC -P2P Certification from RBI.

Ans:-IndiaMoneyMart

Explanation:-Reserve Bank of India (RBI) granted NBFC - P2P (Non-Banking Financial Company - Peer

to Peer) Certificate to IndiaMoneyMart and this certification will enable IndiaMoneyMart to expand

operations and target loan disbursals worth INR 100 Crore by the end of financial year 2018-19. The step

will validates IndiaMoneyMart's strong framework to make lending and borrowing transparent, digital and

convenient in confrontation to the regulatory requirement. IndiaMoneyMart is offering loans starting from

Rs. 10 thousand to Rs 10 Lakhs for a tenure ranging between 3 months to 36 months.

Daily Current Affairs 10th January, 2019

1)Which country's cricketing body was announced as the 105th member of ICC?

Ans:-US

Explanation:-USA Cricket's application to become the 93rd Associate Member in accordance with the ICC

Constitution was approved by ICC Members following the Membership Committee's recommendation to the

ICC Board late last year and is implemented with immediate effect. As a Member of the ICC, USA Cricket

is now eligible to receive funding in accordance with the ICC Development Funding Policy and can sanction

domestic and international cricket in the United States.

2)Who was appointed as the Director General of the Sashastra Seema Bal?

Ans:-Shri Kumar Rajesh Chandra

Explanation:-Kumar Rajesh Chandra was appointed as the Director General of the Sashastra Seema Bal

(SSB). He has been appointed as SSB DG up to 31 December 2021. He is a 1985-batch IPS officer of Bihar

cadre. He is at present the Director General of the Bureau of Civil Aviation Security (BCAS). The SSB,

which has about 80,000 personnel, guards India's borders with Nepal and Bhutan.

3)What is the theme of the first-ever Global Aviation Summit (GAS-2019)?

Ans:-Flying for all

Page 32: Monthly Current Affairs of SEPTEMBER - SSCE

SSCE 8981426494/8296260082 Downloaded from :- www.onlinessce.com Page 32

Explanation:-The first-ever Global Aviation Summit (GAS-2019) with the theme "Flying for all" is

scheduled to be held in Mumbai from January 15-16, 2019. Organised by the Ministry of Civil Aviation

along with FICCI, the GAS-2019 will discuss how to ride this strong tailwind of growth and pave the way

forward for the aviation sector. It will provide a platform to the aviation fraternity to showcase the

challenges in newly developing growth spots and understand how technology-driver innovations will change

air travel in future.The summit is supported by International Civil Aviation Organization (ICAO), Federal

Aviation Administration of US (FAA), International Air Transport Association (IATA), CiviI Air

Navigation Services Organisation (CANSO), Airports Council International (ACl) and Association of Asia

Pacific Airlines (AAPA).

4). This country will host the 2019 Africa Cup of Nations.

Ans:-Egypt

Explanation:-Egypt will host the 2019 Africa Cup of Nations between June 15 and July 13, 2019. The

Confederation of African Football (CAF) President Ahmad Ahmad made the announcement in Senegalese

capital Dakar on January 8, 2019. The CAF executive committee preferred Egypt to South Africa as

replacement for original hosts Cameroon, who were dropped due to delays in preparations and concerns over

security.

5)Which company conferred with the title of world's biggest planemaker for the seventh straight

year?

Ans:-Boeing

Explanation:-US-based Boeing has retained its position as the world's largest planemaker for the seventh

year in a row, delivering a record 806 aircraft in 2018. Overall, the company saw 893 net orders valued at

$143.7 billion. Boeing was followed by its European rival Airbus which delivered 800 aircraft last year.

6)As per latest estimate of Central Statistics Office, what is the rate of Indian economy for FY 19?

Ans:-7.2%

Explanation:- The Central Statistics Office (CSO) has recently released the First advance estimates of

National Income for the current fiscal FY 19, under the Ministry of Statistics and Programme

Implementation. In it, Indian economy is expected to grow at 7.2% in FY19, against 6.7% in FY18, due to

improvement in the performance of agriculture and manufacturing sectors. However, the CSO estimate is a

bit lower than 7.4% growth projected by the Reserve Bank of India (RBI) for the current fiscal. According to

the estimates, farm sector is set to grow at 3.8% against 3.4% last year, at constant or inflation-adjusted

prices. At current prices, the sector expected to grow at an identical 3.8%, which could be emblematic of an

agri-commodity price crash across wholesale mandis.

7)The 86th National Billiards and Snooker Championship will be held in __________.

Ans:-Indore

Explanation:-The 86th National Billiards and Snooker Championship will be held in Indore from 18

January 2019. The event will be hosted by Madhya Pradesh Billiards and Snooker Association. Top cueists,

including Pankaj Advani, will take part in the event. The prize money of the event is 10 lakh rupees. The

event will conclude on 10 February 2019.

Page 33: Monthly Current Affairs of SEPTEMBER - SSCE

SSCE 8981426494/8296260082 Downloaded from :- www.onlinessce.com Page 33

8)Prime Minister Narendra Modi launched development projects worth Rs 1,100 cr at which city of

Maharashtra?

Ans:-Solapur

Explanation:-Prime Minister Narendra Modi launched development projects worth Rs 1,100 cr at Solapur

in Maharashtra. Referring to the passage of The Constitution (One Hundred And Twenty-Fourth

Amendment) Bill in the Lok Sabha, PM, Modi described it as a landmark moment in the nation's history.

PM Modi also laid the foundation stone for an over 1800 crore rupee housing project of 30,000 units under

the Pradhan Mantri Awas Yojana. It will benefit the homeless poor like rag pickers, rickshaw pullers, textile

and beedi workers. PM Modi commissioned the four-laned over 98 km Solapur- Tuljapur-Osmanabad

section on NH-52.

9)For which country, Reserve Bank of India (RBI) has agreed to provide $400 million under swap

deal?

Ans:-Sri Lanka

Explanation:-Reserve Bank of India (RBI) has agreed to provide $400 million to the Central Bank of Sri

Lanka (CBSL) under the SAARC swap facility, to boost the island nation's reserves. CBSL has requested a

further bilateral swap arrangement of $1 billion from RBI which is "under consideration".

10)Who will provide early-stage capital to agriculture and rural enterprises through its venture

capital entity?

Ans:-NABARD

Explanation:-NABARD announced that it will provide early-stage capital to agriculture and rural

enterprises through its venture capital entity, 'NABVENTURES Ltd'. NABVENTURES Ltd has been

incorporated by NABARD under the Companies Act. It will launch its maiden fund offer by the middle of

February 2019. NABARD's headquarter is in Mumbai and its Chairman is Harsh Kumar Bhanwala.

11)Which city police launched the 'Police-E-Eye' mobile application to alert police on traffic

violators?

Ans:-Coimbatore

Explanation:-The Coimbatore city police launched the 'Police-E-Eye' mobile application on 7 January

2019. The app will enable the public to take photographs of traffic rule violators and upload them to the

police. Based on this, the traffic department would issue photo challans to the violators' place to collect the

fine amount.

12)The 6th Women Of India Organic Festival 2019 was held in which city?

Ans:-Chandigarh

Explanation:-The 6th 'Women of India Organic Festival 2019 will be hosted by Ministry of Women and

Child Development (WCD) at Leisure Valley in Chandigarh from 12th to 14th January to celebrate and

promote women farmers and entrepreneurs in the organic sector from the remotest parts of India. The

purpose of the festival is to encourage Indian women entrepreneurs and farmers to connect with more buyers

and thus, empower them through financial inclusion, while promoting organic culture in India. Over 100

women are coming together at Leisure Valley, which will become the epicentre of an organic extravaganza.

Page 34: Monthly Current Affairs of SEPTEMBER - SSCE

SSCE 8981426494/8296260082 Downloaded from :- www.onlinessce.com Page 34

The Festival will showcase over 1,000 varied organic products, including fabric, wellness, grains, seeds,

jewellery, bakery items and a lot more.

13)New planet that NASA's Transiting Exoplanet Survey Satellite (TESS) found is;

Ans:-HD 21749b

Explanation:-Transiting Exoplanet Survey Satellite (TESS), NASA's latest planet-hunting probe, has

discovered a new world outside our solar system, orbiting a dwarf star 53 light years away. This is the third

new planet confirmed by TESS since its launch in April 2018. The planet has been named as HD 21749b.

The planets orbits a bright, nearby dwarf star about 53 light years away, in the constellation Reticulum, and

appears to have the longest orbital period of the three planets so far identified by TESS.

14)The Lok Sabha passed the 'DNA Technology (Use and Application) Regulation Bill, 2019' on

__________.

Ans:-8 January 2019

Explanation:-The Lok Sabha passed the 'DNA Technology (Use and Application) Regulation Bill, 2019' on

8 January 2019. It seeks to provide regulation for the use and application of DNA technology for the

purpose of establishing the identity of a certain category of persons. The bill also seeks to establish a

'National DNA Data Bank' and 'Regional DNA Data Banks'.

15)Name of the first monarch of Malaysia to resign before completing tenure.

Ans:-Muhammad V

Explanation:-The Malaysian King, Sultan Muhammad V has become the Malaysia's first monarch to step

down from the throne before completing his five-year tenure on 6th January 2019. No reason was given and

palace officials did not respond to requests for comment. Malaysia is the only country in the world to have a

rotational monarchy, in place since the country became independent from Britain in 1957. Here, the king

assumes a largely ceremonial role, his assent is needed for the appointment of a prime minister and various

other senior officials. The country has nine royal households, who typically take turns to sit on the throne.

The selection of the next king can be decided by a vote in the Council of Rulers, made up of all nine royal

households.

16)The Supreme court set up a constitution Bench to hear the Ram Janmabhoomi-Babri Masjid land

title dispute case. It consists of how many judges?

Ans:-Five

Explanation:-The Supreme Court on January 8, 2019 set up a five-judge Constitution Bench to hear the

Ram Janmabhoomi-Babri Masjid land title dispute case.The bench will be headed by Chief Justice Ranjan

Gogoi and also comprise of Justices S A Bobde, N V Ramana, U U Lalit and D Y Chandrachud. The

Constitution Bench is scheduled to hear the matter on January 10. On January 4, the top court had said that

further orders in the matter would be passed on January 10 by "the appropriate bench, as may be

constituted".

17)On which date, 2019 Non-Resident Indian Day is observed?

Ans:-January 9

Page 35: Monthly Current Affairs of SEPTEMBER - SSCE

SSCE 8981426494/8296260082 Downloaded from :- www.onlinessce.com Page 35

Explanation:-The Pravasi Bharatiya Divas (PBD), also known as Non-Resident Indian Day, is observed

every year on 9th of January to mark the contribution of the overseas Indian community towards the

development of India. The day commemorates the return of Mahatma Gandhi from South Africa to Mumbai

on 9 January 1915.

18)Who is the author of the book "We Are Displaced: My Journey and Stories from Refugee Girls

Around the World"?

Ans:-Malala Yousafzai

Explanation:-Nobel Peace Prize winner and Pakistani activist Malala Yousafzai pens a new book, titled

'We Are Displaced: My Journey and Stories from Refugee Girls Around the World,' charting her

experiences traveling the world and visiting refugee camps.

Daily Current Affairs 11th January, 2019

1)Who has been ranked World's top woman boxer by the International Boxing Association (AIBA) in

the 48kg category?

Ans:-Mary Kom

Explanation:-M C Mary Kom has attained number one position in the International Boxing Association's

(AIBA) latest world rankings. She became the most successful boxer in world championships' history when

she claimed the 48kg category top honors in Delhi in November 2018. In the 51kg category list, Pinki Jangra

is placed eighth. Asian silver-medallist Manisha Maun is placed eighth in the 54kg category. In the 57kg

division, former world silver-medallist Sonia Lather was placed at the second spot.

2)Who has been named as the 2018 Confederation of African Football Player of the Year?

Ans:-Mohamed Salah

Explanation:-Liverpool's Mohamed Salah has been named as the 2018 Confederation of African Football

Player of the Year. For the second consecutive year he has won the award. Egypt forward Salah, 26, beat

Liverpool team-mate Sadio Mane of Senegal and Arsenal and Gabon striker Pierre-Emerick Aubameyang to

the title. He received the award at a ceremony in Senegal. Houston Dash and South Africa forward Thembi

Kgatlana was named Women's Player of the Year. Salah scored 44 goals for Liverpool during the 2017-18

season, helping the Reds to the Champions League final before scoring twice for Egypt at the World Cup in

Russia.

3)This organisation set up CARE to strengthen Research Publication.

Ans:-UGC

Explanation:-The University Grants Commission (UGC) has approved a list of journals, in that the UGC

has decided to establish a Consortium for Academic and Research Ethics (CARE). The good quality

Research Journals in disciplines under Social Sciences, Humanities, Languages, Arts, Culture, Indian

Knowledge Systems etc., will be maintained by CARE and referred to as 'CARE Reference List of Quality

Journals'. This will be used for all academic purposes.

4)Prime Minister Narendra Modi inaugurated the 'Gangajal Project' in __________.

Ans:-Agra

Page 36: Monthly Current Affairs of SEPTEMBER - SSCE

SSCE 8981426494/8296260082 Downloaded from :- www.onlinessce.com Page 36

Explanation:- Prime Minister Narendra Modi inaugurated the 'Gangajal Project' in Agra on 9 January 2019.

The programme will provide a supply of better drinking water which will benefit the residents as well as

tourists. Gangajal project aims to bring 140 cusecs of Ganga water to Agra. He also launched civic projects

worth Rs 2,980 crore in Agra.

5)This is set to become the first Indian state to roll out Universal Basic Income (UBI)?

Ans:-Sikkim

Explanation:-The Sikkim Democratic Front (SDF) ruled government has recently decided to include UBI

in its manifesto ahead of 2019 state assembly polls and aims to implement the scheme by 2022. If

everything goes according to the plan, it will be the Indian first state to roll out UBI scheme aimed at

providing a fixed monthly income to residents irrespective of their economic, social status. The Universal

Basic Income (UBI) is a programme for providing all citizens of a country or other geographic area/state

with a given sum of money, regardless of their income, resources or employment status. The main idea

behind UBI is to prevent or reduce poverty and promote equality among citizens. As per Basic Income Earth

Network (BIEN), the essential principle behind UBI is the idea that all citizens are entitled to a livable

income, irrespective of the circumstances they're born in.

6)Which will be the 1st Island to operate seaplanes?

Ans:-Andaman and Nicobar Islands

Explanation:-The Union Ministry of Civil Aviation approved operations on 13 water aerodromes under

Ude Desh ka Aam Nagrik-3 (UDAN-3). The very first seaplane could run within Andaman and Nicobar

Islands. Four different islands from Andaman and one island from Nicobar have been selected to fly

seaplanes. These are Havelock, Long Island, Neil Island and Hutbay Island in Andaman and Car Nicobar in

Nicobar.

7)The exemption limit of which tax is doubled to give relief to small businesses.

Ans:-Goods and Service Tax

Explanation:-Finance Minister Arun Jaitley raised the annual turnover for availing composition scheme to

Rs 1.5 crore from Rs 1 crore effective from April 1, 2019, to provide relief to small businesses under Goods

and Services (GST) regime. The Goods and Services Tax (GST) exemption limit has been doubled to Rs. 40

lakh annual turnover and Rs 20 lakh for northeastern states.

8)What will be the position of India in Consumer Market By 2030?

Ans:-3rd

Explanation:-A report by World Economic Forum stated that India is set to become the world's third largest

consumer market behind only USA and China by 2030. The report was titled as 'Future of Consumption in

Fast-Growth Consumer Market - India'. As per the report, consumer spending in India is expected to grow

from 1.5 trillion US dollar at present to six trillion US dollar by 2030. With the annual GDP growth rate of

7.5 percent, India is currently the world's sixth largest economy.

9)Recently SEBI has constituted a "Research Advisory" committee to undertake research relevant for

development and regulation of capital markets. It was headed by;

Ans:-Sankar De

Page 37: Monthly Current Affairs of SEPTEMBER - SSCE

SSCE 8981426494/8296260082 Downloaded from :- www.onlinessce.com Page 37

Explanation:-Market regulator Security and Exchange Board of India (SEBI) has constituted a "Research

Advisory" committee which will assist in formulating policy to undertake research relevant for development

and regulation of capital markets. According to SEBI, the committee would comprise prominent financial

economists and market practitioners. It will be headed by Sankar De. "In order to strengthen its research

function and enhance its linkage to policy making, SEBI has constituted a 'Research Advisory Committee'

headed by Sankar De,"a SEBI statement stated. Another key task of the committee will be to maintain

databases relevant for capital market regulation research.

10)Sydney and Brisbane chosen to host $15 million 24 team ATP Cup in which year?

Ans:-2020

Explanation:-Australian cities Sydney and Brisbane have been announced as the first two host locations for

the newest team event in men's tennis for the multi-million dollar ATP Cup from 2020. The third Australian

city for the tournament is yet to be determined.

11)Nicolas Maduro was sworn in as the President of __________.

Ans:-Venezuela

Explanation:-Nicolas Maduro was sworn in for a second term as Venezuela's President amid the economic

and humanitarian crisis faced by the Latin American country. Maduro was re-elected last year. The US, EU

and 13 other countries have stated that they would not recognise Maduro's Presidency.

12)Who inagurated India's Longest Single Lane Steel Cable Bridge, recently?

Ans:-Shri Pema Khandu

Explanation:-India's longest 300-meter single lane steel cable suspension bridge was inaugurated by

Arunachal Pradesh Chief Minister Pema Khandu. The bridge across the Siang river at Yingkiong in Upper

Siang district has been named Byorung Bridge. It is built at a cost of ₹48.43-crore provided by the DONER

(Department for Development of North Eastern Region) Ministry. The Bridge will reduce the distance from

Yinkiong to Tuting by almost 40 km.

13)Which among the company join hands with Bharti AXA General Insurance recently?

Ans:-IRCTC

Explanation:-Indian Railway Catering and Tourism Corporation (IRCTC), the Railways' subsidiary, will

offer insurance of up to INR50 lakh for free for air passengers who book their tickets through its portal.

IRCTC has tied up with Bharti-Axa for the offer. IRCTC's service charge of INR59 is the cheapest

compared with travel websites. IRCTC has tied up with Galileo, a computer reservation system, for

powering the flight search engine. At present, over 30 lakh users log in through its mobile app and 3 lakh

ticket bookings happen daily through the app.

14)World Hindi Day is celebrated on which date?

Ans:-January 10

Explanation:-World Hindi Day is celebrated on January 10 every year, marking the anniversary of first

World Hindi Conference which was held in 1975. The first World Hindi Conference was inaugurated by the

then prime minister Indira Gandhi. World Hindi Day was first observed on January 10, 2006. World Hindi

Page 38: Monthly Current Affairs of SEPTEMBER - SSCE

SSCE 8981426494/8296260082 Downloaded from :- www.onlinessce.com Page 38

Day and National Hindi Diwas are completely different. National Hindi Diwas is celebrated every year on

September 14. On that day in 1949, the constituent assembly adopted Hindi, written in Devanagari script, as

the official language of the Union. While the focus of the World Hindi Day is to promote the language at the

global stage.

15)The Union Government has launched the Web- Wonder Women campaign in collaboration with

which human rights organization?s

Ans:-Breakthrough India

Explanation:-The Union Ministry of Women and Child Development (WCD) has launched an online

campaign, '#www : Web- WonderWomen'. The purpose of the campaign is to discover and celebrate the

exceptional achievements of women, who have been driving positive agenda of social change via social

media. The Ministry in collaboration with Twitter India and Breakthrough India (a human rights

organization) has started this campaign. It aims to recognize the fortitude of Indian women stalwarts from

across the globe who have used the power of social media to run positive & niche campaigns to steer a

change in society. This Campaign will recognize and acknowledge the efforts of these meritorious Women.

The Campaign invites entries from across the world till 31st of January 2019.

16)The First 'National Consultation on Child Protection' for 2019 was held in __________.

Ans:-New Delhi

Explanation:-The First 'National Consultation on Child Protection' for 2019 was held at India Habitat

Centre, New Delhi under the Chairmanship of the Secretary, Ministry of Women and Child Development.

The meeting was organized to follow up on the exercise of monitoring of the CCIs by States/UTs, to

deliberate upon various aspects of child protection and to take stock of the efforts made by States/UTs

regarding improvements in the Child Care Institutions.

17)Union Minister Nitin Gadkari gave an approval to Iran's Pasargad Bank to open a branch in

__________.

Ans:-Mumbai

Explanation:-Union Minister Nitin Gadkari has announced that India has allowed an Iranian bank, Pasargad

Bank, to open a branch in Mumbai. The move is aimed at circumventing US trade and investment sanctions

as India builds a strategic port in the West Asian country.

18)Who has been appointed as Director General, Fire Services, Civil Defence and Home Guards?

Ans:-Alok Verma

Explanation:-The Cabinet Committee on Appointments has now posted Alok Verma as Director-General,

Fire Services, Civil Defence and Home Guards, for the remaining period of his present term that ends on

January 31. He has been removed as CBI Director and the Additional Director M. Nageswara Rao has again

been appointed interim CBI Director. The CVC was of the view that Mr.Verma's conduct in the case was

suspicious, and there was a prima facie case against him. It also felt that the entire truth would come out if a

criminal investigation was ordered.

Page 39: Monthly Current Affairs of SEPTEMBER - SSCE

SSCE 8981426494/8296260082 Downloaded from :- www.onlinessce.com Page 39

Daily Current Affairs 12th January, 2019

1) Recently, Ministry of External Affairs and Observer Research Foundation organised the 4th

Edition of Raisina Dialogue in New Delhi with the theme of;

Ans:-A World Reorder: New Geometries; Fluid Partnerships; Uncertain Outcomes

Explanation:-The 4th Edition of Raisina Dialogue was organized by Ministry of External Affairs in

partnership with Observer Research Foundation in New Delhi. The Theme of the Dialogue was "A World

Reorder: New Geometries; Fluid Partnerships; Uncertain Outcomes". The Raisina Dialogue whose first

edition was held in 2015 is India's flagship annual geopolitical and geostrategic conference. The Aim of the

Discussion was to address the issues arising from ongoing global transitions and changes to the world order

triggered by unique leadership, innovative partnerships and new technologies. The Inaugural address was

delivered by the Prime Minister of Norway H E Ms Erna Solberg in the presence of India's Prime Minister,

Narendra Modi and External Affairs Minister, Sushma Swaraj. More than 600 delegates from 93 countries

of the world including influential political leaders, strategic thinkers, policy practitioners, technology

innovators and business representatives shared their Vision in the Raisina Dialogue.

2)Polavaram project of which state entered the Guinness book of world record by pouring 32,315.5

cubic meters of concrete non-stop in 24 hours.

Ans:-Andhra Pradesh

Explanation:-The Polavaram project of Andhra Pradesh got its place in the Guinness book of world

record.This project entered the guinness by pouring 32,315.5 cubic meters of concrete non-stop in 24

hours.This project surpassed the leading record of 21,580 cubic metres which was achieved by Abdul

Wahed Bin Shabib project. The state government invested Rs 15,380.97 crore on the project ; the central

government is to release Rs3,517.84 crore. Polavaram irrigation project,is a multipurpose terminal reservoir

will irrigate 38.78 Lac acres of land. It will also set up of 960 mw hydro power plant which will provide

drinking water to 540 villages with a population of 28.5 Lac.

3)The Union Cabinet approved the setting up of 3 AIIMS in which of the following states?

Ans:-J&K and Gujarat

Explanation:- The Union Cabinet approved the setting up of three All India Institutes of Medical Sciences

in Jammu and Kashmir and Gujarat. The institutes will be set up under the Pradhan Mantri Swasthya

Suraksha Yojana. These will be in Samba in Jammu at a cost of Rs. 1,661 crore, in Pulwama in Kashmir at a

cost of Rs. 1,828 crore, and in Rajkot, Gujarat, at a cost of Rs. 1,195 crore. The objective is to establish the

new AIIMS as Institutions of National Importance for providing quality tertiary healthcare, medical

education, nursing education and research in the region. The Cabinet approval also involves the creation of a

post of director with a basic pay of Rs. 2,25,000 for each of the three AIIMS. The cost of construction,

operation, and maintenance of the three AIIMS will be fully borne by the Central government.

4)Which of the following allowed tokenization of debit, credit and prepaid card transactions to

enhance the safety of digital payments ecosystem in India?

Ans:-RBI

Explanation:-The Reserve Bank of India has allowed tokenization of debit, credit and prepaid card

transactions to enhance the safety of the digital payments ecosystem in the country. The central bank has

allowed card schemes like Visa, Mastercard and others to start issuing tokens for third party payment

Page 40: Monthly Current Affairs of SEPTEMBER - SSCE

SSCE 8981426494/8296260082 Downloaded from :- www.onlinessce.com Page 40

applications as well. "Tokenisation and de-tokenisation" shall be performed only by the authorized card

network and recovery of original Permanent Account Number (PAN) should be feasible for the authorized

card network only. Tokenisation acts as an additional layer of security as it masks sensitive card data such as

the 16-digit Account Number, expiration date and security code.

5)Su Tseng-chang was appointed as the Prime Minister of which country?

Ans:-Taiwan

Explanation:-Taiwan President Tsai Ing-wen has appointed Su Tseng-chang as Prime Minister, during a

Cabinet reshuffle following the ruling Democratic Progressive Party's heavy losses in local elections. Su is a

former premier appointed in 2006 by then-President Chen Shui-bian and was a chairman of Tsai's

Democratic Progressive Party for two terms. His appointment follows the widely-expected resignation of

William Lai, the second premier to quit since Tsai took office in 2016.

6)Where was the 25th edition of Partnership Summit 2019?

Ans:-Mumbai

Explanation:-The 25th edition of Partnership Summit 2019, a global platform for dialogue, debate,

deliberation and engagement among Indian and global leaders on economic policy and growth trends in

India was held in Mumbai. The two-day event was inaugurated by the Vice President of India, M Venkaiah

Naidu. The Summit was organized by the Department of Industrial Policy and Promotion, Ministry of

Commerce & Industry, Government of India, State Government of Maharashtra and Confederation of Indian

Industry.

7)Who among the following was conferred with the ICSI Lifetime Award?

Ans:-Adi Godrej

Explanation:-Industrialist and Godrej Group Chairman Adi Godrej was conferred the ICSIs Lifetime

Achievement Award for 'Translating Excellence in Corporate Governance into Reality'.

8)Who has been named as India's Chef de Mission for the 2020 Tokyo Olympics?

Ans:-Birendra Prasad Baishya

Explanation:-President of the Indian Weightlifting Federation (IWF), Birendra Prasad Baishya has been

named as India's Chef de Mission for the 2020 Tokyo Olympics. it is for the first time that the sport of

Weightlifting has been accorded with the honour of Chef de Mission for the Olympics. Baishya is also one

of the Indian Olympic Association (IOA) Vice Presidents. He is a former Union minster and a former Rajya

Sabha Member from Assam. The Japanese capital -Tokyo - will host the Games for the second time in 2020

after having organised it 56 years back in 1964.

9)Name the person who was appointed as President of Alliance for Media Freedom (AFMF).

Ans:-Narasimhan Ram

Explanation:-Narasimhan Ram, Chairman, the Publishing Private Limited, has been appointed President of

the Alliance for Media Freedom (AFMF), a cross-media advocacy and action group of media organization,

promoters, and journalists. While appointing him as the president of the Alliance, his role in thwarting the

Page 41: Monthly Current Affairs of SEPTEMBER - SSCE

SSCE 8981426494/8296260082 Downloaded from :- www.onlinessce.com Page 41

efforts of governments and authoritarian elements to impede the independent functioning of the media was

commended.

10) What is the threshold limit of GST registration according to 32nd Meeting of GST Council?

Ans:-40 lakh

Explanation:-The 32nd Meeting of GST Council was concluded in New Delhi. The Limit of annual

turnover for composition scheme increased to 1.5 crore rupees effective from April 1, 2019. The Council has

decided to increase the GST registration threshold limit to 40 lakh rupees from 20 lakh. The Taxpayers who

are giving Services of up to Rs 50 lacs per annum will be taxed under 6% under Composition Scheme. The

council has decided to simplify the compliance under the composition scheme.

11)Name the bill passed by the Lok Sabha for regulation of the use and application of

Deoxyribonucleic Acid (DNA) technology.

Ans:-The DNA Technology (Use and Application) Regulation Bill, 2019

Explanation:-The Lok Sabha passed "The DNA Technology (Use and Application) Regulation Bill, 2019"

by voice vote after a brief discussion. The Bill has been formulated recognizing the need for regulation of

the use and application of Deoxyribonucleic Acid (DNA) technology, for establishing identity of missing

persons, victims, offenders, under trials and unknown deceased persons. The bill was introduced by Science

and Technology Minister Harsh Vardhan. The Atal Bihari Vajpayee government had set up a DNA profiling

advisory committee. The key components of this Bill include establishment of a DNA Regulatory Board and

accreditation of DNA laboratories undertaking DNA testing, analysing. Establishment of the National and

Regional DNA Data Banks, as envisaged in the Bill, will assist in forensic investigations. The Bill will add

value in empowering the criminal justice delivery system by enabling the application of DNA evidence,

which is considered the gold standard in crime investigations.

12) For which state, Union Cabinet approved Scheduled Tribe (ST) status to 6 communities?

Ans:-Assam

Explanation:-Union Cabinet has approved Scheduled Tribe (ST) status to six communities of Assam.The

communities that are proposed to be extended ST status include, Koch Rajbongshi, Tai Ahom, Chutiya,

Matak, Moran and the tea tribes. The move came before the centre introduced the Citizenship Amendment

Bill, 2019 in Lok Sabha. This amendment bill will give a status to the various tribal people.

13)Which IIT has decided to launch 6-month Artificial Intelligence course recently?

Ans:-IIT Kharagpur

Explanation:-IIT Kharagpur will launch a six-month course on Artificial Intelligence (AI) and Machine

Learning for working professionals and engineering students. The aim of the certified programme is to

strengthen India's talent pool in Machine Learning and AI. The course will begin from March 2019 & will

be offered at institute units in Kharagpur, Bangalore and Kolkata and possibly in Hyderabad. The course

will comprise 16 one-credit modules and one capstone project. The course modules have been carefully

designed to cover mathematical and algorithmic foundations, artificial intelligence fundamentals and several

modules on statistical and machine learning methods.

14) India's 1st Manned Mission Gaganyaan will be launched by __________.

Page 42: Monthly Current Affairs of SEPTEMBER - SSCE

SSCE 8981426494/8296260082 Downloaded from :- www.onlinessce.com Page 42

Ans:- December 2021

Explanation:- ISRO Chairman Dr. K Sivan has announced that the organization is well prepared to meet

the targets of launching Gaganyaan manned mission to outer space by December 2021. Under Gaganyaan

project, India is planning to send three astronauts to outer space for seven days and bring them back. 332

launches are planned this year. Important among them are Chandrayaaan -2 mission to the moon in mid-

April. GSAT-20 Satellite for high bandwidth connectivity to be launched in September-October

15) What is the GDP rate of India for the year 2018-19 as per the report of World Bank?

Ans:-7.3%

Explanation:- The World Bank projected India' GDP, expected to grow at 7.3% in the fiscal year 2018-19,

and 7.5% in the following two years. The Bank said India will continue to be the fastest growing major

economy in the world. In India, the growth has accelerated, driven by an upswing in consumption, and

investment growth has firmed as the effects of temporary factors wane, the World Bank said in its latest

report. Domestic demand has strengthened as the benefits of structural reforms such as Goods and services

Tax (GST) harmonization and bank recapitalization take effect. In 2018,China's Projection is 6.5 and 6.2%

each in 2019 and 2020 and 6% in 2021, according to the January 2019 Global Economic Prospects report

released by the World Bank. Strong domestic demand is envisioned to widen the current account deficit to

2.6 per cent of GDP next year. The World Bank's estimate suggest that India's potential growth rate is

around 7%, and is expected to remain around 7%.

16) Name the Chairman of NSE who resigned his post recently.

Ans:- Ashok Chawla

Explanation:- Ashok Chawla resigned as chairman of the National Stock Exchange of India Ltd (NSE) with

immediate effect "in light of recent legal developments". Chawla's resignation came on a day the Central

Bureau of Investigation (CBI) told a special court in Delhi that it has obtained the sanction of the ministries

concerned to file charges against him and four other former bureaucrats in the Aircel-Maxis case.

17)Bill passed by Lok Sabha that aims to remove leprosy as a ground for divorce in five personal laws.

Ans:- Personal Laws (Amendment) Bill, 2018

Explanation:-The Lok Sabha has passed the Personal Laws (Amendment) Bill, 2018. The bill was moved

by Minister of State for Law PP Chaudhary. The bill was introduced as various recommendations were

made to remove leprosy as a ground for divorce. The bill seeks to remove leprosy as a ground for divorce in

five personal laws Hindu Marriage Act(1955), Dissolution of Muslim Marriages Act(1939),Divorce

Act(1869), Special Marriage Act(1954) and the Hindu Adoptions and Maintenance Act(1956). In 2010 the

United Nations General Assembly adopted a Resolution on the 'Elimination of discrimination against

persons affected by leprosy and their family members', which was signed and ratified by India. The Law

Commission of India in its 256th Report recommended the elimination of discrimination against people

affected by the disease. Leprosy is a chronic infectious bacterial disease caused by Mycobacterium leprae.

Leprosy mainly affects the skin, the peripheral nerves, mucosal surfaces of the upper respiratory tract and

the eyes.

18) This country tops in the EIU Democracy Index 2018.

Ans:- Norway

Page 43: Monthly Current Affairs of SEPTEMBER - SSCE

SSCE 8981426494/8296260082 Downloaded from :- www.onlinessce.com Page 43

Explanation:-The 11th edition of EIU Democracy Index 2018 was released by the Economist Intelligent

Unit compiling the state of democracies in 167 countries in the world. In the list, Norway topped and India

has slipped to 42nd place on the Economist Intelligence Unit's annual Global Democracy Index.

19)How many states signed an agreement for Renukaji Dam Multipurpose Project?

Ans:-6

Explanation:-An agreement for Renukaji Dam Multipurpose Project was signed in New Delhi among the

States of Uttar Pradesh, Haryana, Himachal Pradesh, Delhi, Rajasthan and Uttarakhand in the presence of

Water Resources, River Development and Ganga Rejuvenation Minister Nitin Gadkari. All the six Chief

Ministers were present on the occasion. Under the agreement, three storage projects are proposed to be

constructed on the river Yamuna and two of its tributaries - Tons and Giri in the hilly regions of Uttarakhand

and Himachal Pradesh. In addition to this, a concession agreement was also signed for Namami Gange

projects for the city of Prayagraj under Hybrid Annuity Mode and One City One Operator Concept.

20) For how many Indian cities, the Environment Ministry launched the National Clean Air

Programme?

Ans:- 102

Explanation:- The Environment Ministry launched the National Clean Air Programme (NCAP) aimed at

reducing toxic particulate matter in 102 Indian cities by 2024. The NCAP will be a mid-term, five-year plan

with 2019 as the first year, while 2017 will be the base year for comparison of concentration. An amount of

INR300 crore has been sanctioned for this purpose.

Daily Current Affairs 13th January, 2019

1)With which country, the Union cabinet approved an MoU on the Development of Advanced Model

Single Window?

Ans:-Japan

Explanation:Cabinet approves MoU between India and Japan on Development of Advanced Model Single

Window on January 10, 2019. The cabinet also approves MoU on Maritime issues between India and

Denmark and approves MoU between India and France on technical bilateral cooperation in the field of new

and renewable energy and pproves Signing of the TOR provide tax assistance to Swaziland.

2)This state will host the two-day International Dam Safety conference starting from 13 February

2019.

Ans:-Odisha

Explanation:-Odisha will host the two-day International Dam Safety conference starting from 13 February

2019. The conference will be organised by Union Ministry of Water Resources in collaboration with the

Odisha government and the World Bank.

3)Which city is the venue of the 25th edition of Partnership Summit 2019?

Ans:-Mumbai

Page 44: Monthly Current Affairs of SEPTEMBER - SSCE

SSCE 8981426494/8296260082 Downloaded from :- www.onlinessce.com Page 44

Explanation:-Vice President M. Venkaiah Naidu inaugurated the 25th edition of Partnership Summit 2019

in Mumbai. This edition of the Summit will showcase India in the present landscape of an emerging "New

India", the "New Global Economic Address". The 2-day summit is expected to build new partnerships and

explore investment opportunities in sectors like Artificial Intelligence, big data, food processing, defense,

aeronautics, renewable energy and tourism. It will see New India in active engagement and collaboration

within the country and with the world. The summit is a global platform for dialogue, debate, deliberation

and engagement among Indian and global leaders on economic policy and growth trends in India.

4)ISRO aims to send human astronauts to space by the year of?

Ans:-2021

Explanation:-The year 2021 will be a busy year for the Indian Space Research Organisation (ISRO) as it

has set itself the target to send astronauts to space. The Gaganyaan project by the space agency will make

India the fourth nation to independently send humans into space announced ISRO chief Dr. K Sivan. The

Gaganyaan project was announced by PM Modi on Independence Day last year and union minister Ravi

Shankar Prasad last month had said that three Indian astronauts will be sent to space for up to seven days by

2022 as part of the Gaganyaan.

5)Which municipal corporation has teamed up with NGO under the concept 'Feed the need' to feed

the hugry?

Ans:-Telangana

Explanation:-The Greater Hyderabad Municipal Corporation (GHMC) has teamed up with an NGO, Apple

Home for Orphan Kids at Neknampur of Manikonda, a voluntary organization in its west zone to bridge the

gap between the hungry and food wastage. As per the details, the NGO come up with a proposal to partner

with the civic body for installation of refrigerators at various places with the concept 'Feed the Need'. The

corporation is planning to facilitate installation of refrigerators at vantage points across the zone, so that

food leftover from weddings and other ceremonies may be stored for the poor and urchins to consume. A

proposal towards this was cleared by the GHMC Standing Committee, at a meeting convened in Hyderabad,

Telangana. The refrigerators will be of 530-litre capacity, and installed at 10 locations in Madhapur, Banjara

Hills, Jubilee Hills, Erragadda, Red Hills, Chanda Nagar, and Gachibowli.

6)With which university did the Telangana-based SR Innovation Exchange (SRiX) company sign an

MoU?

Ans:-Cranfield University

Explanation:-SR Innovation Exchange (SRiX), a Telangana-based technology business incubator signed a

collaborative agreement Cranfield University (U.K.). It aims at providing British and Indian entrepreneurs

access to new businesses and academic expertise. As per the agreement, a 10-week immersion programme is

being developed with Indian start-ups mentored in the U.K. by leading Cranfield academics. The partnership

facilitates knowledge exchange between experts from Cranfield visiting SRiX to conduct learning sessions

focused on manufacturing excellence, whereas Cranfield would support SRiX in designing innovation

programmes for corporate clients in the manufacturing sector.

7)Virat Kohli and __________ has recently received the honorary membership of the Sydney Cricket

Ground (SCG).

Ans:-Ravi Shastri

Page 45: Monthly Current Affairs of SEPTEMBER - SSCE

SSCE 8981426494/8296260082 Downloaded from :- www.onlinessce.com Page 45

Explanation:-Indian skipper Virat Kohli and coach Ravi Shastri has received the honorary membership of

the Sydney Cricket Ground (SCG) for their contribution to the sport of cricket. Recently, The Indian team

defeated Australia 2-1 in the four-match Test series, ending the country's 71-year-old wait for a rare series

win Down Under. Apart from Kohli and Shastri, the only other international cricketers to be granted the

honorary membership of the SCG are Sachin Tendulkar and Brain Lara of the West Indies.

8)Name the state that quit from the Ayushman Bharat-Pradhan Mantri Jan Arogya Yojna.

Ans:-West Bengal

Explanation:-West Bengal chief minister Mamata Banerjee announced exit of West Bengal from the

Ayushman Bharat-Pradhan Mantri Jan Arogya Yojna due to the use of PM's picture in the official poster of

the centre-state joint scheme. According to the announcement, the states are supposed to bear 40% of the

project cost, while the Centre will foot 60% of the bill. Furthermore, the centre had unilaterally changed the

scheme to Ayushman Bharat- Pradhan Mantri Jan Aarogya Yojana in spite of agreed efforts to retain the

state sponsored health scheme name: Swasthyasathi which was launched in 2016. Thus, West Bengal joined

the league of states of Telangana, Kerala, Odisha, and Delhi which have opted out of the scheme for varied

reasons.

9)ndia's longest single lane steel cable suspension bridge was named __________.

Ans:-Byorung Bridge

Explanation:-India's longest 300-metre single lane steel cable suspension bridge was inaugurated by

Arunachal Pradesh Chief Minister Pema Khandu. The bridge across the Siang river at Yingkiong in Upper

Siang district has been named Byorung Bridge. The bridge, built at a cost of ?48.43-crore provided by the

DONER (Department for Development of North Eastern Region) Ministry, will reduce the distance from

Yinkiong to Tuting by almost 40 km.

10)Andhra Pradeshs seventh airport inaugurated at __________.

Ans:-Orvakallu

Explanation:-Andhra Pradesh's seventh airport at Orvakallu near Kurnool city was inaugurated by chief

minister N Chandrababu Naidu. The airport has been categorized as '3C' and is capable of handling

turboprop aircraft like ATR-72 and Bombardier Q-400. While Visakhapatnam, Rajamahendravaram,

Vijayawada and Tirupati are major airports in Andhra Pradesh, Kadapa, Puttaparthi (not a commercial

airport yet) and Orvakallu in Kurnool district are the smaller ones.

11)Which Indian journalist has been appointed president of the Alliance for Media Freedom

(AFMF)?

Ans:-Narasimhan Ram

Explanation:-Narasimhan Ram, chairman of THG Publishing Private Limited, has been appointed president

of the Alliance for Media Freedom (AFMF), a cross-media advocacy and action group of media

organisations, promoters and journalists. The Alliance is dedicated to work for the cause of media freedom,

media independence and the safety of the last journalist. It has adopted a charter that underlines the

importance of creating enabling conditions to safeguard free speech guaranteed by the Indian Constitution.

Page 46: Monthly Current Affairs of SEPTEMBER - SSCE

SSCE 8981426494/8296260082 Downloaded from :- www.onlinessce.com Page 46

Daily Current Affairs 14th January, 2019

1)Name of the Indian CM who launched the 'One Family One Job' scheme in the state.

Ans:-Pawan Kumar Chamling

Explanation:-Sikkim Chief Minister Pawan Kumar Chamling launched the 'One Family One Job' scheme,

under which for every family one government job has been allotted, in Gangtok. New recruits were taken for

Group C and Group D posts in 12 departments. Thus, Sikkim became the first state in the country to carry

out such a programme for the people entitling them to state government employee benefits. He also declared

that all loan debts in the farming and agriculture sector would be revoked.

2)RBI slapped Rs 3 crore on this bank for non-compliance in criteria for directors of the bank.

Ans:-Citibank India

Explanation:-The Reserve Bank of India (RBI), under the relevant provisions of the Banking Regulation

Act, 1949, imposed a penalty of Rs 3 crore on Citibank India for non-compliance in criteria for directors of

the bank. Citibank has been accused of deficiencies in regulatory compliance and of directions regarding the

'fit-and-proper criteria' for directors of the bank. The US-based Citibank has been operating in India for over

115 years

3)This state tops in the Khelo India Youth Games with 91 medals?

Ans:-Maharashtra

Explanation:-Maharashtra topped the medal tally with 37 golds at the Khelo India Youth Games in Pune.

Delhi was at second position with 35 gold and Haryana too maintains its third position with 29 gold. 10-year

old Abhinav Shaw became the youngest gold medallist in the shooting. He teamed up with Mehuli Ghosh to

help West Bengal win the 10m air rifle mixed team event.

4)Name of the Indian writer has won the Sushila Devi Literature Award for the novel "Things to

Leave Behind".

Ans:-Namita Gokhale

Explanation:-Writer Namita Gokhale has won the Sushila Devi Literature Award for her novel "Things to

Leave Behind". She was awarded in the 'Best Book of Fiction Written by a Woman Author' category at the

inaugural edition of Bhopal Literature and Art Festival (BLF).

5)Who will be the next Chief Information Officer and spokesperson of the CBI replacing Abhishek

Dayal?

Ans:-Nitin Wakankar

Explanation:-The senior Indian Information Service officer Nitin Wakankar appointed the new Chief

Information Officer and spokesperson of the CBI replacing Abhishek Dayal who has been transferred to the

Publication Division. The decision was taken after the removal of Alok Verma from the post of the director

of the Central Bureau of Investigation by a high powered selection committee. Nitin Wakankar has

previously served in Rashtrapati Bhawan and Defense Ministry.

Page 47: Monthly Current Affairs of SEPTEMBER - SSCE

SSCE 8981426494/8296260082 Downloaded from :- www.onlinessce.com Page 47

6)Who launched National Youth Parliament Festival 2019 in New Delhi?

Ans:-Rajyavardhan Rathore

Explanation:-Minister of Youth Affairs and Sports, Col Rajyavardhan Rathore launched the National Youth

Parliament Festival 2019 at a function in New Delhi. The festival will continue till 24th of Feburary

2019.The theme of the festival is- "Be the Voice of New India" and "Find solutions and contribute to

policy". Ministry of Youth Affairs and Sports propose to take the Youth Festival to every nook and corner of

the country and celebrate it as the "National Youth Parliament Festival". Organising District Youth

Parliaments and taking the festival to the doorsteps of the youth would provide chance to youth of the

country to participate and share their about how to make a New India.

7)Which is the first state to implement the 10% reservation for economically weaker sections of the

general category?

Ans:-Gujarat

Explanation:-Gujarat has become the first state to implement the 10% reservation for economically weaker

sections of the general category. This comes a day after President Ram Nath Kovind gave his assent to the

bill. Gujarat CM Vijay Rupani stated that it will also be applicable to jobs for which announcement has been

made but the procedure is yet to commence.

8)For which bank, Brahm Dutt was apponted as non-executive part-time chairman?

Ans:-Yes Bank

Explanation:-Yes Bank, India's fourth-largest private sector bank, announced the appointment of Brahm

Dutt as non-executive part-time chairman up till July 4, 2020. He is currently also the chairman of the

Nomination and Remuneration Committee. Brahm Dutt has been on Yes Bank's board since July 2013 as an

independent director, and contributed to almost all the sub-committees of the board over the past five and a

half years.

9)Who received the Philip Kotler Presidential award?

Ans:-Narendra Modi

Explanation:-The Prime Minister, Shri Narendra Modi, received the first-ever Philip Kotler Presidential

award, in New Delhi. The Award focuses on the triple-bottom-line of People, Profit, and Planet. It will be

offered annually to the leader of a Nation. The Citation mentions initiatives such as Make in India, Startup

India, Digital India, and Swachh Bharat.

10)Which team won the first ever Premier Badminton League?

Ans:-Bengaluru Raptors

Explanation:-Bengaluru Raptors have won their first ever Premier Badminton League, PBL title. In the title

clash played at Bengaluru, Bengaluru edged out Mumbai Rockets, with a 4-3 win. Star player Kidambi

Srikanth, Vu Ji Trang and the Men's Doubles pair of Mohammad Ahsan and Hendra Satiawan won their

matches for Bengaluru guiding them to victory.

11)This company will launches the Ten Iridium communications satellites?

Page 48: Monthly Current Affairs of SEPTEMBER - SSCE

SSCE 8981426494/8296260082 Downloaded from :- www.onlinessce.com Page 48

Ans:-SpaceX

Explanation:-The SpaceX Falcon 9 launched Iridium Communications satellite comprises of 10 voice and

data satellites,which is an alternative for entire fleet with a new generation of orbiters from the Vandenberg

Air Force station in California. The Iridium constellation was originally developed by Iridium SSC, with

first-generation satellites deployed between 1997 to 2002. The constellation's name comes from the

chemical element Iridium, which has atomic number 77. The Iridium constellation was originally developed

by Iridium SSC, with first-generation satellites deployed between 1997 to 2002. The constellation's name

comes from the chemical element Iridium, which has atomic number 77.

12)The youngest Indian to win junior world title.

Ans:-Kartik Singh

Explanation:-Gurgaon's 8-year-old Kartik Singh won the under-8 category Junior world title at the Kids'

Golf World Championship organised by US kids Golf in Malaysia to become the youngest Indian to win the

title. Kartik is a member of the junior programme at the DLF Golf Academy. The DLF Golf Academy which

was opened in 1999, is the first professional golf school in the country and Karan Bindra is the Director of

Instruction.

13)Global Innovation Policy Center (GIPC) launched a new innovation strategy "Fair Value for

Innovation" at Raisina Dialogue, 2019 in New Delhi. GIPC belongs to which country?

Ans:-USA

Explanation:-The Global Innovation Policy Center (GIPC) of United States Chamber of Commerce

launched a new innovation strategy at Raisina Dialogue, 2019 in New Delhi. The Initiative is titled as "Fair

Value for Innovation". The Initiative will examine economic underpinnings in order to enable breakthrough

innovation and explore the opportunities to harness innovation capital in India and around the globe through

research, advocacy, partnerships, and programs. India is the first market where GIPC is launching this new

innovation initiative.

14)Who will head the 7-member committee on GST Revenue Shortfall faced by the states?

Ans:-Sushil Kumar Modi

Explanation:-Bihar Deputy Chief Minister Sushil Modi will head the 7-member committee to look into

revenue shortfall being faced by the states after the GST roll-out and suggest steps for augmenting

collections. The GST Council, headed by Finance Minister Arun Jaitley and comprising state ministers, had

on December 22, 2018, decided to set up a group of ministers (GoM) to analyze reasons for the shortfall in

revenue collections by the states since July 2017. It would also look into structural patterns of major sectors

of the economy impacting revenue collection, including the services sector.

15)Which company appointed Don Callahan as an additional and independent director?

Ans:-TCS

Explanation:-India's largest software exporter Tata Consultancy Services has appointed Don Callahan as an

additional and independent director for a period of 5 years, based on the recommendations of the

Nomination and Remuneration Committee. He was the Chief Administrative Officer and Head of

Operations and Technology at Citigroup and was a member of Citi's Operating Committee earlierDaily

Current Affairs 17th January, 2019

Page 49: Monthly Current Affairs of SEPTEMBER - SSCE

SSCE 8981426494/8296260082 Downloaded from :- www.onlinessce.com Page 49

1)Who introduced a road safety robot named ROADEO?

Ans:-Chennai Traffic Police

Explanation:-The Chennai Traffic Police introduced a road safety robot named ROADEO, to help in traffic

management and assist citizens. With this, Chennai became the second city after Mumbai to induct robots in

traffic management. The robots would be integrated with traffic signals and could be managed manually via

Bluetooth devices. The robot has been developed by schoolchildren of a private robotics lab.

2)With which country India signed a MoU for the establishment of the Joint Task Force on Blue

Economy?

Ans:-Norway

Explanation:-Norway Prime Minister Erna Solberg arrived in New Delhi for a three-day state visit to India

at the invitation of the Prime Minister of India, Shri Narendra Modi. This made her the first foreign leader

and head of the government to visit New Delhi in 2019. On 8th January 2019, India and Norway signed a

Memorandum of Understanding (MoU) for the establishment of the Joint Task Force on Blue Economy, in

order to promote multi-sectoral cooperation in various aspects of Blue Economy. The two sides recognised

the importance of sustainable use of the oceans, including for food security, energy sources, mineral

exploration and climate friendly maritime transport. The decision assumes significance in view of the fact

that at least 15 per cent of Indian population resides in the coastal areas while 70 per cent of the Norwegian

economy is dependent on the maritime industry of the country. The blue economy is an emerging concept

which encourages better management of ocean or 'blue' resources.

3)Prime Minister Narendra Modi inaugurated a spiritual circuit development project in which of the

following state?

Ans:-Kerala

Explanation:-Prime Minister Narendra Modi inaugurated the project "Development of Spiritual Circuit:

Sree Padmanabha Swamy Temple - Aranmula-Sabarimala" at the cost of Rs. 92.22 crores under Swadesh

Darshan Scheme of Tourism Ministry in Kerala. He also dedicated the 13-km Kollam bypass on NH 66 in

Kerala to the nation. The Sree Padmanaaswamy Temple is one of the 108 Divyadesams of Lord Vishnu and

majority of the work under this project has been carried out at this temple. Under the Project, the ministry of

tourism has developed and rejuvenated the temple along with its surroundings keeping in mind the factors of

heritage and socio-cultural aspects.

4)Who won the Gandhi Peace Prize for the year 2018?

Ans:-Shri Yohei Sasakawa

Explanation:-Shri Yohei Sasakawa won the Gandhi Peace Prize for the year 2018 for his contribution in

Leprosy Eradication in India and across the world.

5)Name the mountaineer who became the first Indian and youngest man to climb the 7 highest peaks and

volcanic summits across all continents.

Ans:-Satyarup Siddhanta

Explanation:-Mountaineer Satyarup Siddhanta conquered Antartica's highest point, Mount Sidley. With this

feat, he became the first Indian and youngest man to climb the 7 highest peaks and volcanic summits across

all continents. He achieved the feat at the age of 35 years and 262 days. Satyarup hails from South Kolkata

and is a software engineer by profession.

5)Recently, Private sector lender IDFC Bank changed its name as __________.

Ans:-IDFC First Bank Ltd

Explanation:-Private sector lender IDFC Bank changed its name to IDFC First Bank Ltd owing to the

merge of the bank with Non-Banking Finance Company (NBFC) Capital First on December 2018. The

Page 50: Monthly Current Affairs of SEPTEMBER - SSCE

SSCE 8981426494/8296260082 Downloaded from :- www.onlinessce.com Page 50

change was effective from January 12, 2019 by virtue of 'Certificate of Incorporation pursuant to change of

name' issued by the Registrar of Companies, Chennai. The board of IDFC Bank also approved the

appointment of V Vaidyanathan, founder and chairman of Capital First Ltd, as managing director and chief

executive officer of the merged entity.

6)Name of India's first Commonwealth Games gold medalist who has appointed as the chief coach of

Women's Team.

Ans:-Mohammed Ali Qamar

Explanation:-Mohammed Ali Qamar, the man who won India's first Commonwealth Games gold medal in

boxing, has taken over as the chief coach for the country's female boxers. He has replaced veteran Shiv

Singh. Qamar, who is set to turn 38, is also the youngest person to land the job. Qamar's CWG gold came in

the 2002 Manchester edition in the light flyweight category.

7)Which country to buy $5 billion oil, gas from USA?

Ans:-India

Explanation:-India committed to purchase USD 5 billion worth of oil and gas from the US per annum and

USD 18 billion worth of defence equipment that are under implementation, a top Indian diplomat here said,

highlighting the growing bilateral trade cooperation

8)January 14 was celebrated as;

Ans:-Veterans Day

Explanation:-The Armed Force is celebrated the Veterans Day to honour the legends of the Army, Navy

and Air Force. The main celebration in Delhi will take place at the New Delhi. Minister of State(Defence)

Dr Subhash Bhambre inaugurated the main event. This Day is commemorated in the memory of first Indian

Commander-in-Chief of the Armed Forces Field Marshal K M Cariappa becoming a veteran in 1953. To

mark the occasion, veteran rallies will be held country wide at Armed Forces stations. Defence Minster

Nirmala Sitharaman along with former Army chief General Shankar Roy Chowdhury, Air Chief Marshal

(retd) Arup Raha,Lieutenant General Abhay Krishna, General Officer Commanding-in-Chief (GOC-in-C),

Eastern Command paid homage to those who made supreme sacrifice in the line of duty.

9)Madhya Pradesh Chief Minister Kamal Nath launched his government's __________ crore farm

debt-waiver scheme named 'Jai Kisan Rin Mukti Yojana'.

Ans:-Rs 50,000

Explanation:-Madhya Pradesh Chief Minister Kamal Nath launched his government's Rs 50,000-crore farm

debt-waiver scheme named 'Jai Kisan Rin Mukti Yojana'. It is expected to benefit 55 lakh small and

marginal farmers. The eligibility cut-off date has been extended to December 12, 2018, from March 31,

2018. Only those farmers who have not registered themselves to pay GST and income tax can avail benefits

under this scheme.

10)Which of the following Indian city is the world's most dynamic city according to the 6th City

Momentum Index?

Ans:-Bengaluru

Explanation:-According to the 6th City Momentum Index, released by property consultant JLL, Bengaluru

has emerged as the world's most dynamic city with a robust technology and innovation ecosystem in place.

The report is titled: 'JLL City Momentum: The World's Top 20 Most Dynamic Cities'. The key theme in this

year's index was cities having a robust technology and innovation ecosystem. Among other Indian cities,

Hyderabad ranked second with Delhi at fourth, Pune at fifth, Chennai at seventh and Kolkata at fifteenth

position featuring in top 20 cities. The other cities include Hanoi at the third position, Nairobi at 6th, Ho Chi

Minh City at 8th Xi'an at 9th Guangzhou at 10th.

Page 51: Monthly Current Affairs of SEPTEMBER - SSCE

SSCE 8981426494/8296260082 Downloaded from :- www.onlinessce.com Page 51

11)Ministry of External Affairs organised the First Annual Disarmament and International Security

Affairs Fellowship Programme in which of the following city?

Ans:-New Delhi

Explanation:-The First Annual Disarmament and International Security Affairs Fellowship Programme was

organised by Ministry of External Affairs in New Delhi. The event will be concluded on 1st February 2019.

The event was inaugurated by the United Nations Under Secretary General and High Representative for

Disarmament, Ms. Izumi Nakamitsu, and Foreign Secretary, Shri Vijay Keshav Gokhale. The Fellowship

Programme, which is being held at the Foreign Services Institute, New Delhi will be attended by Young

Diplomats from 27 countries.

12)India and this country signed an MoU on the safety in Mines, Testing and Research Station on Jan

16.

Ans:-Australia

Explanation:-The Union Cabinet chaired by Prime Minister Narendra Modi has apporoved an MoU

between India and Australia on the safety in Mines, Testing and Research Station. It also egularisation of

Pay Scales of below Board Level Executives in NHPC Ltd., North East Electric Power Corporation, THDC

India Ltd. and SJVN Ltd. w.e.f. 1.1.97. And Expenditure sanction for Integrated E-filing and Centralized

Processing Centre 2.0 Project.

13)Who took oath as Member of Union Public Service Commission (UPSC)?

Ans:-Dr. T.C.A Anant

Explanation:-Dr. T.C.A Anant, former Chief Statistician of India & Secretary, Ministry of Statistics and

Programme Implementation, took the Oath of Office and Secrecy as Member, Union Public Service

Commission (UPSC) .Oath was administered by Shri Arvind Saxena, Chairman, UPSC. The commission

has a strength of 10 members, besides a chairperson.Despite Anant's appointment, the commission had one

more vacancy .Former Delhi Police Commissioner Bhim Sain Bassi, Prof. Pradeep Kumar Joshi, Air

Marshal A S Bhonsle (retired), Sujata Mehta, Manoj Soni, Smita Nagaraj, M Sathiyavathy and Bharat

Bhushan Vyas are members of the UPSC. A member of the UPSC have a maximum of six-year tenure or till

an incumbent attains the age of 65.

14)Noted mathematician Michael Atiyah was passed away recently. He belongs to the country of;

Ans:-United Kingdom

Explanation:-Prof. Sir Michael Atiyah (89), the noted British mathematician has passed away in United

Kingdom on 11th January 2019. He was best known for his co-development of a branch of mathematics

called topological K-theory and the Atiyah-Singer index theorem. His research also involved deep insights

relating to mathematical concepts known as "vector bundles". Sir Michael was also a recipient of the Abel

Prize (2004) and Fields Medal (1966), and was the former president of the Royal Society in London. He was

one of the foremost mathematicians whose work encompassed topology and geometry, and worked with

several Indian mathematicians such as Indian-American mathematician Harish-Chandra and Vijay Kumar

Patodi, during his career.

15)Name the ordinance promulgated by President Ram Nath Kovind to protect the rights of married

Muslim women and criminalise the practice of triple talaq or "Talaq-e-Biddat", on 12th January

2019?

Ans:-Muslim Women (Protection of Rights on Marriage) Ordinance, 2019

Explanation:-Ministry of Law and Justice announced that President Kovind has promulgated the Muslim

Women (Protection of Rights on Marriage) Ordinance, 2019. The ordinance is to protect the rights of

married Muslim women and criminalise the practice of triple talaq or Talaq-e-Biddat" .On this

occassion,The President also re-promulgated Companies Amendment Ordinance 2019 and Indian Medical

Page 52: Monthly Current Affairs of SEPTEMBER - SSCE

SSCE 8981426494/8296260082 Downloaded from :- www.onlinessce.com Page 52

Council Amendment Ordinance 2019. Instant "Triple" or "Talaq-e-Biddat"is an Islamic practice that allows

men to divorce their wives immediately by uttering the word "talaq" (divorce) thrice. The pronouncement

can be oral or written, or, in recent times, delivered by electronic means - telephone, SMS, email or social

media.The Modi Government formulated a bill named The Muslim Women (Protection Of Rights on

Marriage) bill 2017 and was passed in Lok Sabha on December 28th 2017 to protect the rights of married

muslim women.

Daily Current Affairs 18th January, 2019

1)Minister of State in Department of Space Dr. Jitendra Singh inaugurated the 1st batch of UNNATI

programme of ISRO in which city?

Ans:-Bengaluru

Explanation:-Minister of State in Department of Space Dr. Jitendra Singh inaugurated the 1st batch of

UNNATI programme of Indian Space Research Organisation (ISRO) in Bengaluru, Karnataka. UNNATI:

Unispace Nanosatellite Assembly and Training programme. It has been launched by ISRO as part of

initiative by United Nations Office for Outer Space Affairs to commemorate the 50th anniversary of the first

United Nations Conference on the Exploration and Peaceful Uses of Outer Space (UNISPACE-50).

2)RBI imposed a Rs 1-crore penalty to this bank for non-compliance of Know Your Customer (KYC)

guidelines and fraud-classification norms.

Ans:-Bank of Maharashtra

Explanation:-Reserve Bank of India (RBI),imposed a Rs 1-crore penalty on state-owned Bank of

Maharashtra (BoM) for non-compliance of Know Your Customer (KYC)guidelines and fraud-classification

norms. The is penalty has been imposed on failure of the bank to follow the rules and regulation issued by

the RBI. In the year 2018, the RBI had imposed a penalty of Rs 1 crore on BoM on account of delay on the

part of the bank to detect and report fraud in accounts.

3)President Ram Nath Kovind appoints whom as Additional Solicitor General in Supreme court?

Ans:-Sanjay Jain

Explanation:-President Ram Nath Kovind appointed senior advocate Sanjay Jain as Additional Solicitor

General of India for the Supreme Court.He will be on the post till June 30,2020.With the appointment of

Sanjay Jain, the Central government now has seven Additional Solicitor General in Supreme Court.

Additional Solicitor General, appointed under Law Officers Act 1987, is the third largest law officerof the

Central Government. It is a duty of the Additional Solicitor Genral to assist theSolicitor General. The

President of India Ramnath Kovind also appointed Senior Advocate KM Nataraj as the Additional Solicitor

General (ASG) in the Supreme Court of India. His term will also end on June 30, 2020.He also holded the

post as the Additional Advocate-General of Karnataka for four years from 2009 and also as the Additional

Solicitor General of India for Southern Zone for a period of 03 years.

4)The world's first television channel dedicated to human rights was launched in __________.

Ans:-London

Page 53: Monthly Current Affairs of SEPTEMBER - SSCE

SSCE 8981426494/8296260082 Downloaded from :- www.onlinessce.com Page 53

Explanation:-The world's first television channel dedicated to human rights was launched in London on 16

January 2019. The channel was launched by the International Observatory of Human Rights (IOHR). The

web-based channel would bring human rights issues to audiences in over 20 countries across Europe, Latin

America and the Middle East.

5)ISRO will launch a student satellite called 'Kalamsat' and an imaging satellite known as 'Microsat-

R' on __________.

Ans:-24 January 2019.

Explanation:-ISRO will launch a student satellite called 'Kalamsat' and an imaging satellite known as

'Microsat-R' on 24 January 2019. Both the satellite will be carried by 'PSLV C-44' rocket. PSLV is a four-

stage launch vehicle. In PSLV-C44, the fourth stage (PS4) of the vehicle would be moved to a ?higher

circular orbit so as to establish an orbital platform for carrying out experiments.

6)IMBEX 2018-19 a bilateral army exercise of India-Myanmar begins in __________.

Ans:-Chandimandir

Explanation:-Second edition of the India-Myanmar bilateral army exercise, IMBEX 2018-19, commenced

at Chandimandir Military Station, that houses the headquarters of the Western Command, Chandigarh. The

aim of this joint training programme, spread over six days, is to train the Myanmar delegation for

participation in United Nations peacekeeping operations under the UN flag.

7)Name of the first Indian athlete who is nominated for the prestigious Laureus World Comeback of

the Year Award.

Ans:-Vinesh Phogat

Explanation:-Indian star wrestler Vinesh Phogat became the first Indian athlete to be nominated for the

prestigious Laureus World Comeback of the Year Award. Vinesh has been nominated in "Laureus World

Sporting Comeback" category for the year 2019 and will be competing for the award with some of the

world's greatest sportsmen including golfer Tiger Woods, Canadian snowboarder Mark McMorris,

American alpine ski racer Lindsey Vonn, Japanese figure skater Yuzuru Manyu and Dutch para-

snowboarding star Bibian Mentel-Spee.

8)A project named SCATFORM to improve forests in the targeted catchment areas was launched in

the state of;

Ans:-Tripura

Explanation:-A Rs 1,000 crore 'Sustainable Catchment Forest Management' (SCATFORM) project was

launched in Tripura on 16 January 2020. The project will help improve forests in the targeted catchment

areas. It would also create livelihood of the people living in and around the forest areas. The Japan

International Cooperation Agency (JICA) will bear 80% cost of the project.

9)Who was named as interim chief executive officer (CEO) of Accenture?

Ans:-David Rowland

Explanation:-Accenture,a Dublin based consultant firm's CEO Pierre Nanterme steps down from his post

due to his health issues. Financial Officer of the company David Rowland was named interim CEO. Though

Page 54: Monthly Current Affairs of SEPTEMBER - SSCE

SSCE 8981426494/8296260082 Downloaded from :- www.onlinessce.com Page 54

resigned as CEO of the company,Pierre Nanterme will continue at Accenture as advisor to the CEO. The

lead independent director Marge Magner was named non-executive chair.

10)How many childrens were shortlisted for the award 'Pradhan Mantri Rashtriya Bal Puraskar

2019'?

Ans:-26

Explanation:-The National Selection Committee under the Women and Child Development Ministry, have

finalised the names of 26 children for 'Pradhan Mantri Rashtriya Bal Puraskar 2019'. Maneka Gandhi is the

Chairperson of the National Selection Committee. The award is conferred upon children for their bravery,

social service, and exceptional achievement in the fields of innovation, sports, and arts.

11)Name the German chemical company which has signed a MoU with the Adani Group to invest Rs.

16,000 crore in the acrylics value chain?

Ans:-BASF

Explanation:-BASF SE (BASF), German chemical company and the largest chemical producer in the

world, has signed a Memorandum of Understanding (MoU) with the Adani Group to invest 2 billion Euros

or Rs. 16,000 crore in the acrylics value chain. This deal is BASF's largest investment in India to date. The

designated site will be located at Mundra port in Gujarat. BASF would hold the majority stake in the joint

venture (JV). It is expected that this partnership with BASF is a big step forward in enabling 'Make in India'

programme, as this partnership allows to produce in Mundra several of the chemicals along the C3 chemical

value chain that is currently being imported.

12)Who honoured with 2019 Japan Prize for his contributions in Biological Production, Ecology?

Ans:-Dr. Rattan Lal

Explanation:-Japan Prize Foundation announced India born professor Dr. Rattan Lal as the recipient of the

2019 Japan Prize in the field of "Biological Production, Ecology". Dr. Yoshio Okamoto was announced to

be the recipient of the 2019 Japan Prize on of "Materials and Production" field. Each laureate will receive a

certificate of recognition and a commemorative gold medal. About the Award: The Japan Prize is highly

competitive: the nomination process ends in February, and, every year from March to November, the

Foundation considers the nominations of 15,000 prominent scientists and researchers from around the world.

The 2020 Japan Prize will focus on fields of the fields of "Electronics, Information and Communication"

and "Life Science".

13)Who has won the Franco-German Human Rights Award?

Ans:-Yu Wensheng

Explanation:-Detained Chinese lawyer, Yu Wensheng has won the Franco-German Human Rights Award

for his exceptional contribution to the protection and promotion of human rights. His wife, Madam Xu Yan,

received the Franco-German Prize for Human Rights and the Rule of Law on her husband's behalf from the

French and German ambassadors to China at an event organised by the German embassy in Beijing on

January 14, 2019. Wensheng is best known for suing the Beijing government over the city's once chronic

pollution - was detained in January 2018 and charged with "inciting subversion of state power". Prior to his

arrest, he had circulated an open letter calling for five reforms to China's constitution, including the

institution of multi-candidate presidential elections.

Page 55: Monthly Current Affairs of SEPTEMBER - SSCE

SSCE 8981426494/8296260082 Downloaded from :- www.onlinessce.com Page 55

14)Who inaugurated the two-day Gandhian Resurgence Summit in Prayagraj on 17th Jan 2019?

Ans:-Shri Ramnath Nath Kovind

Explanation:-The President of India Shri. Ramnath Nath Kovind inaugurated the two-day Gandhian

Resurgence Summit on the occasion of Father of the Nation Mahatma Gandhi's 150 birth anniversary at

Prayagraj in Uttar Pradesh. The summit was organised by the Parmarath Niketan, the Harijan Sevak Sangh

and the Global Interfaith WASH Alliance. The President also unveiled the statue of Maharishi Bhardwaj in

the town.

15)Where was the 1st National EMRS National Sports Meet 2019 held?

Ans:-Hyderabad

Explanation:-The 1st National Level Sports Meet for the students of Eklavya Model Residential Schools

(EMRS) was held at G.M.C Balayogi Stadium in Gachibowli, Hyderabad. The event has a participation of

1777 students comprising of 975 boys and 802 girls representing 20 States of the country. The striking

feature of Sports meet has been the platform made available to tribal students studying in EMRS from the

remote corners of North East India to Himachal Pradesh in North to Kerala in South and Gujarat in West to

exhibit their latent talent in sports which would otherwise not be possible in normal course. The three day

event had competitions in 13 different disciplines of track and field that includes Hockey, Wrestling,

Football, Archery, Volley Ball, Kabaddi, Handball, Kho Kho and Athletics.

16)Who was appointed as the new chief executive officer (CEO) of Myntra and Jabong?

Ans:-Amar Nagaram

Explantion:-Ananth Narayanan, the chief executive officer of Myntra and Jabong, a unit of Walmart-owned

Flipkart, quit the company to pursue external opportunities. Ananth Narayanan is expected to join as the

chief executive officer of video streaming service Hotstar. Flipkart executive Amar Nagaram who

previously headed consumer shopping experience has been appointed as the new head of Myntra and Jabong

and will report to Flipkart Group CEO Kalyan Krishnamurthy. Earlier in November Flipkart's co-founder &

group CEO, Binny Bansal, had also stepped down from the company following a probe into serious personal

misconduct.

17)The Election Commission of India appointed __________ as the Chief Electoral Officer of Andhra

Pradesh.

Ans:-Gopal Krishna Dwivedi

Explanation:-The Election Commission of India (ECI) appointed Gopal Krishna Dwivedi as has Chief

Electoral Officer of Andhra Pradesh with immediate effect. Mr.Dwivedi will replace R. P. Sisodia. The ECI

termed the CEO's replacement as an administrative decision. Gopal Krishna Dwivedi, a 1993-batch Uttar

Pradesh cadre IAS officer, was serving as Principal Secretary, Animal Husbandry, Dairy Development and

Fisheries. As per G.O RT. No. 131, through which the ECI's order has been republished by the A.P.

government, the ECI nominated Mr. Dwivedi as the CEO in exercise of the powers conferred by Section

13A (1) of the Representation of People Act, 1950 in place of Mr. Sisodia. It was mentioned that the ECI

consulted the State government for affecting the reshuffle.

18)How much did RBI through the purchase of government securities?

Ans:-10,000 cr

Page 56: Monthly Current Affairs of SEPTEMBER - SSCE

SSCE 8981426494/8296260082 Downloaded from :- www.onlinessce.com Page 56

Explanation:-Reserve Bank of India announced that it will inject 10,000 crore rupees into the system to

increase liquidity through open market operations (OMOs). The offers have been instructed to be submitted

in electronic format on the RBI Core Banking Solution (E-Kuber) system. It also plans to inject liquidity

under OMOs for 50,000 crore rupees in January 2019.

19)The 10th edition of the India Rubber Expo (IRE-2019) has started in which city?

Ans:-The 10th edition of the India Rubber Expo (IRE-2019) has been inaugurated by Union Minister Suresh

Prabhu at the Bombay Convention & Exhibition Centre in Mumbai on January 17. The IRE will showcase

the latest technological developments in rubber industry and also talk about the current market status,

facilities and services on R&D and quality control. It provides a platform for the Indian companies to meet

and collaborate with overseas companies. The 3-day event offers a unique opportunity for Indian companies

to meet and collaborate with overseas companies. The India Rubber Expo is Asia's largest rubber expo.

Daily Current Affairs 19th January, 2019

1)Who has been appointed as the Chief Justice of Pakistan?

Ans:-Asif Khosa

Explanation:-Justice Asif Saeed Khosa, known for adding lyrical flair in his judgments, sworn in as the

Chief Justice of Pakistan in a ceremony at Islamabad. It was attended by foreign legal luminaries, including

former Indian SC judge Madan Bhimarao Lokur. Justice Khosa, 64, will retire on December 21. He is the

26th Chief Justice of Pakistan. The Chief Justice is nominated by Imran khan, the PM of Pakistan and

appointed by Arif Alvi, the President of Pakistan. Prior to this Mr.Khosa served as judge of the Lahore High

Court. He succeeded Mian Saqib Nisar.

2)The US-Ireland Alliance has announced that the veteran actress ________ will receive the 'Oscar

Wilde Award' on 21 February 2019.

Ans:-Glenn Close

Explanation:-The US-Ireland Alliance has announced that the veteran actress Glenn Close will receive the

'Oscar Wilde Award' on 21 February 2019. She will receive the award at the 14th Annual Oscar Wilde

Awards. She also won the 2018 Golden Globe award for Best Actress in a Motion Picture Drama for the

film, 'The Wife'.

3)Which country signed a powerful MoU with India at International chamber of media and

entertainment industry?

Ans:-Mongolia

Explantion:-The Embassy of Mongolia and International Chamber of Media and Entertainment Industry

(ICMEI), India have further strengthened the relationship between India and Mongolia by signing a MoU in

the field of art and culture. The MoU was signed in the presence of H.E. Gonchig Ganbold Ambassador of

Mongolia to India and Dr. Sandeep Marwah, President ICMEI. The MoU will enable the two countries to

work together on different segments including print, radio, television, cinema, animation, Performing and

Fine Arts, tourism, hospitality, education, environment, solar energy, skill development, PR events and

advertising, Fashion and Design, and Journalism. During the event, the deserving students from Mongolia

were handed over the first scholarship of a short-term course of Asian Academy of Film and Television

which also includes boarding and lodging for three months.

Page 57: Monthly Current Affairs of SEPTEMBER - SSCE

SSCE 8981426494/8296260082 Downloaded from :- www.onlinessce.com Page 57

4)Ministry of Petroleum and Natural Gas launched Saksham 2019 in which city?

Ans:-New Delhi

Explanation:-'Saksham', an annual high intensity one-month long people-centric mega campaign of

Petroleum Conservation Research Association (PCRA) was launched by the Ministry of Petroleum and

Natural Gas in New Delhi. 'Saksham' Cycle Day in 200 cities, Cyclothons, Workshops for drivers of

commercial vehicles, Nationwide campaign through Radio, TV, Digital Cinemas, Outdoor media etc. will be

conducted, with a focus on reaching out to various segments of fuel users.

5)Libcoin and this plans to build India's 1st Lithium Ion Giga Factory.

Ans:-BHEL

Explanation:-Bharat Heavy Electricals Limited (BHEL) and Libcoin are in dialogue to form a world-class

consortium to initially build 1GWh lithium-ion battery plant in India. Its capacity will be scaled up to

30GWh in due course. With this, India has finally taken steps into its energy security and clean energy

commitment to the world. BHEL will be sending a team of senior officers for the study of the facilities,

R&D infrastructure and other techno-commercial issues soon.

6)With which company Reliance Jio Infocomm launch a new e-commerce platform in the country?

Ans:-Reliance Retail

Explanation:-Reliance Industries Ltd Chairman and managing director Mukesh Ambani announced that

group companies Reliance Retail and Reliance Jio Infocomm would jointly launch a new e-commerce

platform in the country. Gujarat would be the first state to get it. The announcement was made at the

inauguration of the three-day Vibrant Gujarat summit. The new e-commerce project would empower 1.2

million shopkeepers in Gujarat.

7)Who inaugurated the National Museum of Indian Cinema (NMIC) in Mumbai?

Ans:-Narendra Modi

Explanation:-Prime Minister Narendra Modi inaugurated the National Museum of Indian Cinema (NMIC)

in Mumbai. The development of the Museum was guided by the Museum Advisory Committee headed by

Shyam Benegal. An innovation Committee headed by Prasoon Joshi provided an upgrade to the NMIC.

Built at a cost of Rs 140.61 crore, the museum depicts the journey of over a century of Indian cinema in a

story telling mode with the help of visuals, graphics, artifacts, interactive exhibits and multimedia

expositions.

8)Which country launched a Satellite for World's 1st Artificial Meteor Shower?

Ans:-Japan

Explanation:-Japan Aerospace Exploration Agency has launched Tokyo-based startup Star-ALE's mini-

satellite aimed to deliver the world's first artificial meteor shower. The satellite carries 400 tiny balls, will be

enough for 20-30 events, that will glow as they rush down the atmosphere on being released. ALE plans to

deliver its first show in Hiroshima in early 2020.

9)India's 2nd Defence Innovation Hub will come up in __________.

Page 58: Monthly Current Affairs of SEPTEMBER - SSCE

SSCE 8981426494/8296260082 Downloaded from :- www.onlinessce.com Page 58

Ans:-Nashik

Explanation:Nashik in Maharashtra will be the site of the country's second defence innovation hub after

Coimbatore in Tamil Nadu. Union Minister of State for Defence Subhash Bhamre stated that the defence

innovation hub at Nashik will help local industries and entrepreneurs in the country's defence sector. He

added, the plan is to achieve annual arms exports of Rs 35,000 crore.

10)Which country will host the 14th session of the Conference of Parties to UNCCD to address the

issue of land degradation and desertification?

Ans:-India

Explanation:-India will host the 14th session of the Conference of Parties to the United Nations Convention

to Combat Desertification (UNCCD) to address the issue of land degradation and desertification which will

have participation from over 100 countries. It is likely to be held in October 2019. The UNCCD, established

in 1994, has 197 parties working towards maintaining and restoring land and soil productivity and mitigating

the effects of drought. The permanent secretariat of the Convention is located in Bonn, Germany since

January 1999.

11)India will need how many airports and an investment of $40-50 billion to handle at least 1.1 billion

passengers flying to, from and within the country, as per Vision 2040 document released at the Global

Aviation Summit (GAS) 2019?

Ans:-200

Explanation:-Union Minister for Commerce & Industry and Civil Aviation, Shri Suresh Prabhu unveiled

the Vision 2040 document for aviation sector in the Global Aviation Summit (GAS) 2019, in Mumbai. The

document is prepared by consultancy firm KPMG and industry body Federation of Indian Chambers of

Commerce and Industry (FICCI) in consultation with the government.The Vision 2040 document aims to

have 1.12 billion air passengers in 2040 from 187 million air passengers in 2018. This comprises around 821

million domestic passengers and around 303 million international passengers (to and from India). It further

states that India will need 200 airports and an investment of $40-50 billion to handle at least 1.1 billion

passengers flying to, from and within the country.

12)Senior IPS Officer Prabhat Singh was appointed as Director General for which Department?

Ans:-NHRC

Explanation:-The Department of Personnel & Training, Government of India has appointed Senior IPS

Officer Prabhat Singh as Director General (Investigation) in the National Human Rights Commission.

Singh, a 1985 batch IPS Officer of AGMUT cadre is at present Special Director, Central Reserve Police

Force (CRPF). He has been appointed to the post for a period up to April 30, 2020.

13)With which country India has signed a long-term contract supply of uranium ore concentrates to

widen its source base for the nuclear fuel?

Ans:-Uzbekistan

Explanation:- India has signed a long-term contract with Uzbekistan for supply of uranium ore concentrates

to widen its source base for the nuclear fuel. The contract was exchanged in the presence of Prime Minister

Narendra Modi and Uzbekistan President Shavkat Mirziyoyev, who were at the Vibrant Gujarat summit in

Gandhinagar. Uzbekistan is the seventh largest exporter of uranium in the world. The Exim Bank signed an

Page 59: Monthly Current Affairs of SEPTEMBER - SSCE

SSCE 8981426494/8296260082 Downloaded from :- www.onlinessce.com Page 59

agreement with Uzbekistan to extend a 200-million US dollar line of credit for financing housing and social

infrastructure projects.

14)HDFC Standard Life Insurance has renamed as;

Ans:HDFC Life Insurance

Explanation:- HDFC Standard Life Insurance has changed its name to HDFC Life Insurance following the

receipt of relevant approvals from regulatory authorities. The change in name is effective right off the bat

and the company will henceforth operate under the name HDFC Life Insurance Company Ltd. it stated.

15)How many Indian institutions made it to the Emerging Economies University Rankings 2019

report, released by Times Higher Education (THE)?

Ans:-49

Explanation:-The London-based, Times Higher Education (THE) released the Emerging Economies

University Rankings 2019 report which shows that India has improved its ranking and secured a higher

position with 49 institutions making it to the list. According to this year's report 25 Indian institutions figure

in the top 200 institutes, a rise from 17 last year. The report stated that the Indian Institute of Science

(IISc),Bangaluru retains the highest spot for India (14th), followed by the Indian Institute of Technology

Bombay (27th). Last year India had 42 institutions in the overall list. China remains the most represented

nation with 72 institutions in total and claiming four of the table's top five positions with Tsinghua

University topping the chart. Nearly 450 universities from 43 countries, across four continents were

compared this year.

16)Which state government announces Shaurya Award for Students?

Ans:Haryana

Explanation:-Chief Minister of Haryana, Shri Manohar Lal Khattar announced "Shaurya Award" to

students in the name of 71 state police force martyrs who laid down their lives while maintaining law and

order and protecting society since the formation of the state in 1966. Prize money of Rs. 11,000 will be

given to students for courageous deeds every year in a village block where the school is located under this

award. The announcement was made by Manohar Lal Khattar at the first state-level function at National

Police Memorial in New Delhi.

17)Name the Indian who has been featured in the Top "Global Thinkers 2019".

Ans:-Mukesh Ambani

Explanation:-Chairman and Managing Director of Reliance Industries, Mukesh Ambani who is also the

Richest Indian, has been featured in the Top "Global Thinkers 2019" ranking released by the prestigious

Foreign Policy publishing. Alibaba's founder Jack Ma, Amazon's CEO Jeff Bezos and IMF Head Christine

Lagarde also featured in the list of few names released by Foreign Policy Publishing. The Full list of 'Global

Thinkers 2019' will be published on 22nd January 2019 at the 10th Anniversary of Global Thinkers listing.

Mukesh Ambani has replaced Alibaba's founder Jack Ma to become the Asia's Richest person with a fortune

of 44.3 billion dollar.

Daily Current Affairs 20th January, 2019

Page 60: Monthly Current Affairs of SEPTEMBER - SSCE

SSCE 8981426494/8296260082 Downloaded from :- www.onlinessce.com Page 60

1)The Ministry of Finance constituted a Group of Ministers (GoM) for boosting the Real Estate Sector

under the GST regime. It consists of how many members?

Ans:-7

Explanation:-The Ministry of Finance on January 15, 2019 constituted a 7-Member Group of Ministers

(GoM) for boosting the Real Estate Sector under the GST regime. The GoM was constituted in pursuance of

the decision of GST Council taken in the 32nd meeting held on January 10, 2019 at New Delhi. The group

has been tasked to provide a Composition Scheme for Residential Construction Units. The Secretary of the

GoM will be Manish Sinha, Joint Secretary (TRU-II), Central Board of Indirect taxes and Customs (CBIC).

2)The telecom company which is to launch AI-powered chatbot in India.

Ans:-BESCOM

Explanation:-The Bangalore Electricity Supply Company (Bescom) is preparing to launch Artificial

Intelligence (AI) powered chatbots to deal with the innumerable calls and messages that its multiple

helplines receive when there are power cuts. The power utility has called for an Expression of Interest from

IT service providers to implement the idea on its social media pages, helpline, and app. The chatbots are

expected to be utilized in multiple interfaces, such as WhatsApp, social media, and the website.

3)Who has won the 'Laureus Sportsman of the Year award' the maximum number of times?

Ans:-Roger Federer

Explanation:-The Laureus World Sports Awards is an annual awards ceremony, which honours individuals

and teams from the world of sports along with sporting achievements throughout the year. Swiss tennis

player Roger Federer with 6 awards holds the record for having won the 'Laureus Sportsman of the Year

award' maximum number of times.

4)PM Modi inaugurates Vibrant Gujarat Global Summit 2019 on January 18, 2019. The theme of the

summit is;

Ans:-Youth Connect 2019: Shaping A New India - The Story of Billion Dreams

Explanation:-Prime Minister Narendra Modi inaugurated the ninth edition of the Vibrant Gujarat Global

Summit 2019 on January 18, 2019. The summit is being held at the Mahatma Mandir, Gandhinagar, Gujarat.

The theme of the Vibrant Gujarat Global Summit 2019 was "Youth Connect 2019: Shaping A New India -

The Story of Billion Dreams". The heads of five countries including Uzbekistan President Shavkat

Mirziyoyev, Rwanda President Paul Kagame, Prime Minister of Denmark Lars Lokke Rasmussen, Prime

Minister of the Czech Republic Andrej Babis and Prime Minister of Malta Joseph Muscat are expected to

attend the summit.

5)The Ministry of Finance has started which series on Twitter to educate general public about the

budgetary process?

Ans:-Know Your Budget

Explanation:-The Union Ministry of Finance has started 'Know Your Budget' fortnight series on its Twitter

handle to educate general public about the budgetary process. The series would explain the importance of

Union Budget and its making, and would continue for about a fortnight. In the first series of tweets, the

Ministry explained the terms - Union Budget and Vote on Account. These are as follows: The Budget is the

Page 61: Monthly Current Affairs of SEPTEMBER - SSCE

SSCE 8981426494/8296260082 Downloaded from :- www.onlinessce.com Page 61

most comprehensive report of the government's finances in which revenues from all sources and outlays for

all activities are consolidated. The Budget also contains estimates of the government's accounts for the next

fiscal year called Budget estimates. The Vote on Account is defined as a grant made in advance by

Parliament for expenditure for a part of the next financial year.

6)Which station name was changed to Pandit Deendayal Upadhyay Station?

Ans:-Mughalsarai junction railway station

Explanation:-The Uttar Pradesh Cabinet approved the proposal to rename the Mughalsarai tehsil after

Pandit Deendayal Upadhyay, co-founder of the Bharatiya Jana Sangh. The proposal for renaming the

Mughalsarai railway station to Pandit Deendayal Upadhyay junction was initiated in August 2018. Many

places in Uttar Pradesh have got new names under the government of Chief Minister Yogi Adityanath. In

2018, the state cabinet decided to rename Allahabad city Prayagraj, and Faizabad district was renamed

Ayodhya district. The Indian Air Force airport in Gorakhpur was renamed after Maha Yogi Gorakhnath.

7)How many lawmakers voted against the Brexit deal in the UK Parliament?

Ans:-432

Explanation:-UK lawmakers voted 432 to 202 against May's plan for taking Britain out of the European

Union, the biggest parliamentary defeat for a government in modern British political history.

8)The Union Home Minister has approved a task force report on the use of space technology in which

of the following area?

Ans:-Border management

Explanation:-Union Home Minister Rajnath Singh has approved report of task force created by MHA to

identify areas for use of space technology in improving Border Management. The key aim is to strengthen

island and border security and facilitate development of infrastructure in border and island areas.

9)Which institute has announced the launch of a full-fledged bachelor's programme in Artificial

Intelligence technology?

Ans:-IIT Hyderabad

Explanation:-The Indian Institute of Technology-Hyderabad has announced the launch of a full-fledged

bachelor's programme in Artificial Intelligence technology from the new academic session, which is a first

for the country and only the third globally.

10)Who were awarded honorary membership of Sydney Cricket Ground?

Ans:-Ravi Shastri, Virat Kohli

Explanation:-Indian national cricket team's captain Virat Kohli and coach Ravi Shastri on January 11, 2019

received the honorary membership of the Sydney Cricket Ground (SCG) for their contribution to the sport of

cricket.The Indian team defeated Australia 2-1 in the four-match Test series, ending the country's 71-year-

old wait for a rare series win Down Under.

Page 62: Monthly Current Affairs of SEPTEMBER - SSCE

SSCE 8981426494/8296260082 Downloaded from :- www.onlinessce.com Page 62

11)The Constitution (103rd Amendment) Act, 2019, the Act providing __________ % reservation in

government jobs and educational institutions to Economically Weaker Sections (EWS) of General

Category.

Ans:-10%

Explanation:-The Constitution (103rd Amendment) Act, 2019, the Act providing 10 percent reservation in

government jobs and educational institutions to Economically Weaker Sections (EWS) of General Category,

came into effect on January 14, 2019. The move came after the Union Government exercised its powers

conferred by sub-section (2) of section 1 of the Constitution (One Hundred and Third Amendment) Act,

2019 and appointed January 14, 2019 as the date on which the provisions of the said Act shall come into

force.

12)Indian Naval Air Station Shibpur will be commissioned as this __________.

Ans:-INS Kohasa

Explanation:-Naval Air Station Shibpur will be commissioned as INS Kohasa by Chief of Naval Staff

Admiral Sunil Lanba on January 24, 2019. The move is being undertaken to enhance the operational

capability of Andaman and Nicobar Command.

13)In which State do the Airports Authority of India (AAI) plan to test remote Air Traffic Control

(ATC) tower system?

Ans:-Gujarat

Explanation:- The Airports Authority of India (AAI) is to run a pilot project that will in all likelihood see

Gujarat's Ahmedabad and Bhavnagar as the first to do away with manned towers for providing Air Traffic

Control (ATC) services at their airports. Earlier, AAI and SAAB signed Memorandum of Understanding

(MoU) that the airports would run a pilot deployment of a Remote ATC Tower system where air traffic

controllers and the control tower would be replaced by a bank of 16 high-definition, remotely operated,

infrared cameras. SAAB India, which is into defence and aviation, will lead the pilot project that is expected

to take a year before it becomes operational. The system is projected as a measure that can help AAI save

costs. AAI, which manages more than 120 airports, is expected to operationalize more airfields in the

coming months, especially with new routes being added under the regional connectivity scheme.

14)Which ministry will organise the event 'Shehri Samridhi Utsav' from February 1, 2019?

Ans:-Ministry of Housing and Urban Affairs

Explanation:-'Shehri Samridhi Utsav' organised by the Ministry of Housing and Urban Affairs, will focus

on urban livelihoods and extending the outreach of National Urban Livelihoods Mission (DAY-NULM) to

the poorest of the poor and the most vulnerable, and facilitate access of SHG members to the other

government schemes. The Deen Dayal Antyodaya Yojana - National Urban Livelihoods Mission extends

coverage to all the 4041 statutory cities and towns, there by covering almost the entire urban population. The

mission aims to provide the shelter equipped with essential services to the urban homeless in a phased

manner to reduce poverty and vulnerability of the urban poor households.

15)Who to lead Indian womens hockey team for Spain tour?

Ans:-Rani Rampal

Page 63: Monthly Current Affairs of SEPTEMBER - SSCE

SSCE 8981426494/8296260082 Downloaded from :- www.onlinessce.com Page 63

Explanation:-Ace striker Rani Rampal will captain an 18-strong Indian women's team, while experienced

goalkeeper Savita will be her deputy for the season opener in Spain starting Jan 26, Hockey India announced

Friday. The Indian Hockey team is set to play four matches with Spain and two matches against the

Women's World Cup runners-up, Ireland. The team will leave on January 24 for the Spain tour from

Bengaluru, where they have been based for the past two weeks for the National Camp.

Daily Current Affairs 21st January, 2019

1)Who has been elected as the new Prime Minister of Sweden?

Ans:-Stefan Lofven

Explanation:-In Sweden, Prime Minister Stefan Lofven has been elected for a second term on 18th January

2019 after an agreement was struck between the Social Democrats, Greens, Liberals, and Centre Party, and

after the Left Party agreed to abstain from voting against Lofven. He is expected to formally present his new

government and its full programme on 21st January 2019. It will be one of the weakest administrations in

Sweden in 70 years, with just 32.7% of voters having cast ballots for the two parties. Sweden's inconclusive

legislative elections in September 2018 had not left either main parliamentary bloc with a majority.

2)Which state became the third state after Jharkhand and Gujarat to approve the 10 % reservation

for economically backward among upper castes in jobs and educational institutions?

Ans:-Uttar Pradesh

Explanation:-Uttar Pradesh became the third state after Jharkhand and Gujarat to approve the 10 per cent

reservation for economically backward among upper castes in jobs and educational institutions. The

Approval was given in a meeting headed by Uttar Pradesh Chief Minister, Yogi Adityanath. The

Reservation for the poor among upper castes came into effect on 14th January 2019, when Parliament

passed "The Constitution (124 Amendment) Bill 2019".

3)Who was appointed as Additional Solicitor General of India for the Supreme Court?

Ans:-Sanjay Jain

Explanation:-President Ram Nath Kovind appointed senior advocate Sanjay Jain as Additional Solicitor

General of India for the Supreme Court.He will be on the post till June 30,2020.With the appointment of

Sanjay Jain, the Central government now has seven Additional Solicitor General in Supreme Court.

Additional Solicitor General, appointed under Law Officers Act 1987, is the third largest law officerof the

Central Government. It is a duty of the Additional Solicitor Genral to assist theSolicitor General. The

President of India Ramnath Kovind also appointed Senior Advocate KM Nataraj as the Additional Solicitor

General (ASG) in the Supreme Court of India. His term will also end on June 30, 2020.He also holded the

post as the Additional Advocate-General of Karnataka for four years from 2009 and also as the Additional

Solicitor General of India for Southern Zone for a period of 03 years.

4)Who won Rosa Park Trailblazer award?

Ans:-Gurinder Singh Khalsa

Explanation:-An Indian-American philanthropist and entrepreneur, Gurinder Singh Khalsa(45), has been

presented with the prestigious Rosa Park Trailblazer award for his campaign that forced US authorities

change their policy towards headgear of the Sikh community.

Page 64: Monthly Current Affairs of SEPTEMBER - SSCE

SSCE 8981426494/8296260082 Downloaded from :- www.onlinessce.com Page 64

5)Airports Authority of India (AAI) plan to test remote Air Traffic Control (ATC) tower system for

which of the following state?

Ans:-Gujarat

Explanation:-The Airports Authority of India (AAI) is to run a pilot project that will in all likelihood see

Gujarat's Ahmedabad and Bhavnagar as the first to do away with manned towers for providing Air Traffic

Control (ATC) services at their airports. Earlier, AAI and SAAB signed Memorandum of Understanding

(MoU) that the airports would run a pilot deployment of a Remote ATC Tower system where air traffic

controllers and the control tower would be replaced by a bank of 16 high-definition, remotely operated,

infrared cameras. SAAB India, which is into defence and aviation, will lead the pilot project that is expected

to take a year before it becomes operational. The system is projected as a measure that can help AAI save

costs. AAI, which manages more than 120 airports, is expected to operationalize more airfields in the

coming months, especially with new routes being added under the regional connectivity scheme.

6)Reserve Bank of India (RBI) approved the appointment of V Vaidyanathan as MD and CEO of

which bank for a period of three years?

Asn:-IDFC First Bank

Explanation:-Reserve Bank of India (RBI) approved the appointment of V Vaidyanathan as MD and CEO

of IDFC First Bank for a period of three years. The Tenure of V Vaidyanathan started from 19th December

2019. V Vaidyanathan is the founder and chairman of Capital First. IDFC Bank and non-banking financial

company Capital First completed their merger in December 2018 and created a loan asset book of 1.03 Lakh

crore for the merged entity IDFC First bank.

7)Which city is the venue of the 7th ASEAN-India Tourism Ministers 2019?

Ans:-Ha Long

Explanation:-The 7th ASEAN-India Tourism Ministers was held in Ha Long City, Viet Nam on 18th

January 2019. Here, Union Tourism Minister of India, K J Alphons co-chaired the Tourism Ministers'

meeting with the Vietnamese minister, Mr. Nguyen Ngoc Thien. As a follow up to the Summit, the

Ministers launched the ASEAN-India Tourism Cooperation Year 2019, and looked forward to enhancing

cooperation with a view to increasing two-way tourist visits and promoting greater people-to-people

exchanges between ASEAN and India. The Ministers' meeting agreed to further enhance ASEAN-India

cooperation in tourism under the framework of the '2012 Memorandum of Understanding between ASEAN

and India on strengthening tourism cooperation with intensified efforts and activities. The Tourism Ministers

of Brunei Darussalam, Cambodia, Indonesia, Lao PDR, Malaysia, Myanmar, Philippines, Singapore and

Thailand also attended the meeting.

8)25th International conference and exhibition on terrestrial and satellite broadcasting, BES EXPO

2019 held in __________.

Ans:-New Delhi

Explanation:-25th International conference and exhibition on terrestrial and satellite broadcasting, BES

EXPO 2019 was inaugurated by the Information and Broadcasting Secretary Amit Khare and TRAI

Chairman RS Sharma in New Delhi. The theme of the exhibition is "Next Gen Broadcasting in the IT

World". The Exhibition was concluded on 19th January 2019. More than 300 companies from 25 countries

from around the world have displayed their product at the BES EXPO 2019. The conference was organized

for the 25th consecutive year and will be attended by more than 1000 delegates. BES EXPO 2019 was

Page 65: Monthly Current Affairs of SEPTEMBER - SSCE

SSCE 8981426494/8296260082 Downloaded from :- www.onlinessce.com Page 65

organized by Broadcast Engineering Society (India), endorsed by DRM U.K. and supported by Prasar

Bharati, ABU Malaysia, AES India, Ministry of Communications & IT and Society of Broadcast Engineers,

U.S.A.

9)2019 Arab Economic and Social Development Summit held in which of the following place?

Ans:-Beirut

Explanation:-The Arab Economic and Social Development Summit held in Lebanon's capital Beirut. The

leaders of many Arab countries took part in the summit aimed to make a joint statement on a 29-item agenda

that ranges from discussions on an Arab free trade zone and the economic effect of Syrian refugees on host

countries. The economic meeting is a prelude to the actual Arab League summit taking place in Tunisia in

March.

10)India's first PolicyHack for Teachers was launched by;

Ans:-Dell

Explanation:-India's first PolicyHack for Teachers has been launched by Dell in partnership with the

UNESCO Mahatma Gandhi Institute of Education for Peace and Sustainable Development (MGIEP) and

Ignus. As a part of its three-city PolicyHack series, Dell hosted an interactive hackathon where teachers

focused on their learning journey. It's first-of-its-kind hackathon for teachers which will create a platform

where teachers, school representatives and a cross-sectional group experts related to education can interact

with each other.

11)Defence Minister Nirmala Sitharaman has inaugurated the Diffo Bridge recently. It was located in

the state of;

Ans:Arunachal Pradesh

Explanation:-In Arunachal Pradesh, Defence Minister Nirmala Sitharaman has inaugurated the Diffo

Bridge over Diffo river in Lower Dibang Valley district on 18th January 2019. The bridge is 426 mts long

Pre Stressed Concrete Box Girder type bridge on Roing-Koron-Paya road in the state. It would provide

uninterrupted access between Dibang valley and Lohit valley region of Eastern Arunachal Pradesh and an

all-weather Road to the troops deployed on the China Border. The construction work was done by Project

Udayak, Border Roads Organisation (BRO) in this region.

12)The 2-day Agri Vision 2019 Conference was held in __________.

Ans:-Hyderabad

Explanation:-The 2-day Agri Vision 2019 Conference on envisioning Agro solutions for Smart and

Sustainable Agriculture was held in Hyderabad. It was inaugurated by Vice-President M Venkaiah Naidu.

The conference was jointly organized by Professor Jayashankar Telangana State Agricultural University

(PJTSAU), Confederation of Indian Industry(CII) and Cornell University, USA.

13)Which city was named as the World Capital of Architecture for 2020 by UNESCO?

Ans:-Rio de Janeiro

Explanation:-The United Nations Educational, Scientific, and Cultural Organization (UNESCO) has named

the Brazilian city of Rio de Janeiro as the World Capital of Architecture for 2020. Rio will be the first city to

Page 66: Monthly Current Affairs of SEPTEMBER - SSCE

SSCE 8981426494/8296260082 Downloaded from :- www.onlinessce.com Page 66

receive the title under a programme launched together by UNESCO and the International Union of

Architects (UIA) in November 2018. The city beat Paris and Melbourne to get the position. As the first

World Capital of Architecture, Rio de Janeiro will hold a series of events under the theme "All the worlds.

Just one world," and promote the internationally agreed 2030 Agenda for Sustainable Development's 11th

Goal: "Make cities and human settlements inclusive, safe, resilient and sustainable".

14)Where was the two Fast Interceptor Craft (FIC) induced to boost coastal security by the Indian

Navy?

Ans:- Nagapattinam, Tamil Nadu

Explanation:-The Indian Navy inducted two Fast Interceptor Craft (FIC) at Nagapattinam, Tamil Nadu, to

boost coastal security. The vessels were acquired for Sagar Prahari Bal, a unit of the Navy. The boats can

carry a variety of armament from heavy machine guns to grenade launchers with the upper deck canopies

bullet-proof.

15)The Government of India (GoI) has approved setting up of three new Naval Air Squadrons in

which two states?

Ans:-Gujarat and Tamil Nadu

Explanation:-The Government of India (GoI) has approved setting up of three new naval air squadrons in

Gujarat and Tamil Nadu as part of efforts to bolster the Indian Navy's overall strength. Besides this, the

Central Government has also sanctioned manpower towards manning additional aircraft in existing Dornier

Surveillance Squadrons in Kerala and Andaman Islands. A contract for procurement of 12 Dornier aircraft

was signed with Hindustan Aeronautics Limited (HAL) two years back and their delivery is commencing

soon. The new Dornier 228 maritime surveillance aircraft to be delivered to the Navy are fitted with

improved 'state of the art' sensors and equipment which includes glass cockpit, advanced surveillance radar,

optical sensors and networking features. The aircraft will enhance maritime domain awareness of Indian

Navy through round the clock sensor based surveillance and provide targeting data in areas of operation to

ward off terror and other threats from sea.The addition will further strengthen Indian Navy's efforts at

Coastal Security of India's vast coastline of over 7 thousand kilometres.

16)Who was named as the event ambassador for the 16th Tata Mumbai Marathon which will be held

on January 20, 2019?

Ans:-M C Mary Kom

Explanation:-Six-time women's world boxing champion and Olympic medallist M C Mary Kom was

named as the event ambassador for the16th Tata Mumbai Marathon which will be held in January 20, 2019.

It would be organized by Procam International. The marathon will have a prize money of USD 405,000 and

will see the participation of over 46,000 runners.

17)The Embassy of this country and International Chamber of Media and Entertainment Industry

(ICMEI), India have signed a MoU in the field of art and culture.

Ans:-Mongolia

Explanation:-The Embassy of Mongolia and International Chamber of Media and Entertainment Industry

(ICMEI), India have further strengthened the relationship between India and Mongolia by signing a MoU in

the field of art and culture. The MoU was signed in the presence of H.E. Gonchig Ganbold Ambassador of

Mongolia to India and Dr. Sandeep Marwah, President ICMEI. The MoU will enable the two countries to

Page 67: Monthly Current Affairs of SEPTEMBER - SSCE

SSCE 8981426494/8296260082 Downloaded from :- www.onlinessce.com Page 67

work together on different segments including print, radio, television, cinema, animation, Performing and

Fine Arts, tourism, hospitality, education, environment, solar energy, skill development, PR events and

advertising, Fashion and Design and Journalism. During the event, the deserving students from Mongolia

were handed over the first scholarship of short-term course of Asian Academy of Film and Television which

also includes boarding and lodging for three months. Besides, decisions regarding the visit of media team to

Mongolia, festival of Films from Mongolia, joint venture in feature film and short film with Mongolian film

makers, promotion of Mongolia as a film location, Tourism in Mongolia, etc. were also taken at a joint

meeting.

18)According to a report by global consultancy firm PwC, what is the position of India in the world's

largest economy rankings in 2019?

Ans:-5th

Explanation:-India is likely to surpass the United Kingdom in the world's largest economy rankings in

2019, to become the world's fifth largest economy, according to a report by global consultancy firm PwC.

PwC's Global Economy Watch report projects real GDP growth of 1.6% for the UK, 1.7% for France and

7.6% for India in 2019.

19)India's first Light combat helicopter (LCH) was developed by;

Ans:-HAL

Explanation:-Light combat helicopter (LCH),has successfully carried out air-to-air missile firing on a

moving aerial target at the integrated test range in Chandipur on Odisha coast.This LCH is first in the

country,which is designed and developed by Rotary Wing Research & Design Centre (RWRDC),Hindustan

Aeronautics Limited (HAL). The helicopter is equipped with helmet mounted sight and a forward looking

infrared sighting system, allowing its pilots to detect and destroy any target on ground or in the air. Other

weapons on the LCH include a 20mm Turret gun and 70 mm Rockets,that had completed the test last year.

20)President of India Shri Ram Nath Kovind appointed Justice Lingappa Narayana Swamy as the

Acting Chief Justice of which high court?

Ans:- Karnataka

Explanation:-President of India Shri Ram Nath Kovind using the power conferred to him under Article 223

of the Constitution appointed Justice Lingappa Narayana Swamy as the Acting Chief Justice of the

Karnataka High Court. He was appointed Acting Chief Justice of the Karnataka High Court following the

elevation of the incumbent Chief Justice Dinesh Maheshwari to the Supreme Court. Justice Swamy who

began his career as an Advocate in 1987 and practiced in the Karnataka High Court is presently the senior

most Judge of the Karnataka High Court.

Daily Current Affairs 22nd January, 2019

1)Flipkart promotes whom to head Myntra-Jabongs fashion & lifestyle categories?

Ans:-Rishi Vasudev

Explanation:-Walmart backed E-Commerce giant Flipkart promoted its Fashion department head Rishi

Vasudev to head Myntra - Jabong's Fashion and lifestyle categories as an additional responsibility. Rishi

Vasudev will report to Flipkart CEO Kalyan Krishnamurthy after the formal announcement which is

expected in next 10 days.

Page 68: Monthly Current Affairs of SEPTEMBER - SSCE

SSCE 8981426494/8296260082 Downloaded from :- www.onlinessce.com Page 68

2)Which state CM hiked Rs.200 per month under Madhu Babu Pension Yojana scheme?

Ans:-Odisha

Explanation:-Odisha's Chief Minister Naveen Patnaik announced in a video conference on "Ama Gaon,

Ama Bikash" a hike of Rs 200 per month in the social security pension under Madhu Babu Pension Yojana

(MBPY). The state government launched Madhu Babu Pension Yojana on January 1, 2008, by merging two

pension schemes - Old Age Pension and Odisha Disability Pension schemes. According to the new

change,the elderly persons aged over 60 years would get a revised old age pension of Rs 500 per month as

against Rs 300 and those who are more than 80 years old, the pension would be Rs 700 per month instead of

Rs 500 per month and the enhanced pension will come into effect from February 15, 2019.

3)Name the country which has signed an MoU to provide modern infrastructural facilities to Swami

Vipulananda Institute of Aesthetic Studies of Eastern University in Batticaloa district of Sri Lanka.

Ans:-India

Explanation:-India and Sri Lanka signed an MoU to provide modern infrastructural facilities to Swami

Vipulananda Institute of Aesthetic Studies of Eastern University in Batticaloa district of Sri Lanka. The

project involves refurbishment of the auditorium and construction of a modern building complex with

recording cum editing facilities and supply vehicles to the institute through a grant of over 27 crore Sri

Lankan rupees from the Government of India.

4)The annual meet of World Economic Forum (WEF) begins in __________.

Ans:-Switzerland

Explanation:-The annual meet of World Economic Forum (WEF) began on January 22, 2019 in Davos,

Switzerland. The theme of the World Economic Forum 2019 would be 'Globalisation 4.0: Shaping a Global

Architecture in the Age of the Fourth Industrial Revolution'. Overall, around 3000 participants from 115

economies would be present during the annual meeting. Those expected to attend the meet include German

Chancellor Angela Merkel, Swiss President Ueli Maurer, Japan's Shinzo Abe, Italy's Giuseppe Conte,

Brazilian President Jair Bolsonaro and Israel's Benjamin Netanyahu, among 30 heads of states and

governments. The US President Donald Trump will not be attending the meet this year.

5)Virat Kohli has been confered with how many honors in the 2018 ICC Awards?

Ans:- 3

Explanation:- Virat Kohli has become the first player in history to win the Sir Garfield Sobers Trophy for

ICC Cricketer of the Year, the ICC Men's Test Player of the Year and the ICC ODI Player of the Year

awards following an extraordinary 2018. The International Cricket Council also announced the men's Test

and ODI teams of the year with Virat Kohli named captain of both sides. Kohli's 1,322 runs at an average of

55.08 with five centuries in 13 Tests and 1,202 runs at an astounding average of 133.55 with six hundreds in

14 ODIs saw him selected to both sides by every voting member of the Academy with the majority of them

also opting to name him at the helm of each.

6)Nirmala Sitharaman launched Tamil Nadu defence industrial Corridor in __________.

Ans:-Tiruchirppalli

Page 69: Monthly Current Affairs of SEPTEMBER - SSCE

SSCE 8981426494/8296260082 Downloaded from :- www.onlinessce.com Page 69

Explanation:-Defence Minister Nirmala Sitharaman launched a series of projects as a part of the Tamil

Nadu Defence Industrial Corridor in Tiruchirppalli.This is considered to be the second corridor succeeding

the one launched in Aligarh,Uttar Pradesh. This is considered to be the mega corridor that would link the

remaining miltary manufacturing units present in South India,it Is tuned at the cost of 3038cr. The Tamil

Nadu corridor has six nodes at Trichy, Salem, Hosur, Kovai, Madurai and Chennai. It is expected to be a

specialised facility hub in aero component manufacturing. Sitharaman also launched the "CODISSIA

Defence Innovation and Incubation Centre" to support the MSMEs, start-ups and existing industries in their

endeavour to expand and contribute significantly for defence indigenisation.

7)Who inaugurated the 15th Pravasi Bharatiya Diwas Convention at Varanasi, Uttar Pradesh?

Ans:-Narendra Modi

Explanation:-The Prime Minister, Narendra Modi inaugurated the 15th Pravasi Bharatiya Diwas

Convention at Varanasi, Uttar Pradesh. For the first time, the three day long convention is being organized

in Varanasi from 21-23 January, 2019. The theme of PBD Convention 2019 is "Role of Indian Diaspora in

building New India".

8)Who will head an 11-member working group constituted by the IBBI to study the concept of 'Group

Insolvency' and suggest a suitable framework?

Ans:-U.K. Sinha

Explanation:-Former Chairman of Securities and Exchange Board of India (SEBI), U.K. Sinha will head an

11-member working group constituted by the Insolvency and Bankruptcy board of India (IBBI) to study the

concept of 'Group Insolvency' and suggest a suitable framework. The Working Group have to submit the

report within two months recommending a regulatory framework to facilitate insolvency resolutions and

liquidation of corporate debtors in a Group. It was noted that Corporate groups falls apart in insolvency and

thereby thwart the chances of their restructuring. Group Insolvency will prevent this situation. Group

Insolvency will be very beneficial in a situation where two or more applications are pending in the same

court against debtors in the same group. Managing Director of State Bank of India, Anshula Kant and

Executive Director and CFO of Tata Steel, Koushik Chatterjee are members this working group.

9)Which company launched an e-platform under Project ReWeave to help Handloom weavers?

Ans:-Microsoft

Explanation:-Microsoft India launched a new e-commerce platform re-weave.in for handloom weavers

under its Project ReWeave. his e-commerce platform will help connect artisans to buyers directly, enabling

them to expand to newer customers and markets. Microsoft in association with the National Institute of

Fashion Technology (NIFT) also curated a special curriculum in 'CAD and Color for Handloom Weaving' to

provide digital training in handloom design. The company first entered the Indian market in 1990 and has

since worked closely with the Indian government, the IT industry, academia and the local developer

community to usher in some of the early successes in the IT market. Microsoft currently offices in the 9

cities of Ahmedabad, Bangalore, Chennai, Hyderabad, Kochi, Kolkata, Mumbai, the NCR (New Delhi and

Gurgaon) and Pune.

10)Who has been chosen for the first Sheikh Saud International Prize for Materials Research?

Ans:- C. N. R. Rao

Page 70: Monthly Current Affairs of SEPTEMBER - SSCE

SSCE 8981426494/8296260082 Downloaded from :- www.onlinessce.com Page 70

Explanation:-Jawaharlal Nehru Centre for Advanced Scientific Research announced that the Centre for

Advanced Materials of the United Arab Emirates (UAE) chose Eminent Scientist and Bharat Ratna recipient

CNR Rao for the first Sheikh Saud International Prize for Materials Research. The Award containing a

plaque, a medal and a Cash Prize of USD 100000 will be presented by Sheikh Saud to Professor CNR Rao.

CNR Rao was awarded the highest Civilian award, Bharat Ratna in 2014 and presently he is working as the

Honorary President of Jawaharlal Nehru Centre for Advanced Scientific Research.

11)Which country tops in the Global Trust Index as per the 2019 Edelman Trust Barometer report?

Ans:-China

Explanation:-The 2019 Edelman Trust Barometer report, noted that the Global Trust Index witnessed a

marginal increase of 3 points to 52. China topped the Trust Index among both the informed public and the

general population segments. India is among the most trusted nations globally when it comes to government,

business, NGOs and media. India was at the 2nd place in the informed public category and 3rd place in the

general population category. In terms of trust in companies headquartered in each market, the most trusted

are those from Switzerland, Germany, and Canada.

12)Who has been sworn in as the president of Madagascar recently?

Ans:-Andry Rajoelina

Explanation:-Madagascar president, Andry Rajoelina, has sworn into office after taking the oath in

Antananarivo before nine judges of the High Constitutional Court. He won nearly 56% to defeat another

former president, Marc Ravalomanana.

13)The central bank of which country banned the use of Indian currency notes with denominations

above Rs.100?

Ans:-Nepal

Explanation:- The Nepal Rastra Bank (NRB), the central bank of Nepal, has recently announced the ban of

Indian currency notes with denominations above Rs.100. Under the new rule, Nepali citizens cannot carry

these denominations to other countries except India. Similarly, Nepalis are also not allowed to bring such

notes from other countries. The central bank has issued a circular prohibiting Nepali traveller, banks and

financial institutions from holding or carrying and trading Indian bank notes higher than Rs. 100. The central

bank said in its circular that Indian denominations of Rs 2,00, Rs 500 and Rs 2000 cannot be carried and

used for trading. The decision to ban these notes has come at a time when the NRB has requested the

Reserve Bank of India (RBI) to allow Nepal to use currency notes with all denominations. At present, only

the notes with a denomination of Rs. 100 and below are freely exchangeable in Nepal.

14)Indians over 65 and under 15 years can now use Aadhaar Card to visit which country?

Ans:-Nepal

Explanation:-The Union Ministry of Home Affairs on January 20, 2019 announced that Aadhaar Cards are

now valid travel documents for Indians under 15 and over 65 years travelling to Nepal and Bhutan.

However, Indians belonging to the age bracket of 15 to 65 years will not be able to use Aadhaar to travel to

the two neighbouring countries.

15)Which state government has recently roll out Startup and Innovation policy '2018-2023'?

Page 71: Monthly Current Affairs of SEPTEMBER - SSCE

SSCE 8981426494/8296260082 Downloaded from :- www.onlinessce.com Page 71

Ans:- Tamil Nadu

Explanation:-The Tamil Nadu government roll out 'Startup & Innovation policy 2018-2023' with a mission

of providing an enabling and innovative ecosystem for startups registered in the State and to make Tamil

Nadu a 'Global Innovation hub for startups' by 2023. The mission shall aim for a minimum of 1,00,000 high

skilled job creation, direct and indirect, in the Startup ecosystem. For implementing the policy, the State will

set up Tamil Nadu Startup Fund with a corpus of Rs 250 crore. The fund will be invested in other SEBI

registered Alternative Investment Funds for investment in startups and MSMEs established in Tamil Nadu.

16)Israel opens new international airport near;

Ans:-Red Sea

Explanation:-Israel inaugurated a new international airport Monday in its desert south, meant to boost

tourism to the nearby Red Sea and serve as an emergency alternative to Tel Aviv's Ben Gurion Airport in

times of conflict. Initially Ramon Airport will handle only domestic flights, operated by Israeli carriers

Arkia and Israir.

17)Who won the "Woman's Voice Award" at the Apeejay Kolkata Literary Festival (AKLF)?

Ans:-Ranjani Murali

Explanation:-US Based Indian poet Ranjani Murali was awarded the "Woman's Voice Award" at the

Apeejay Kolkata Literary Festival (AKLF). The Objective of the award is to recognize and encourage

creative writing by women in India. The Award carried a cash prize of Rs 1 Lakh along with a citation.

Ranjani Murali's second book "Clearly you are ESL" won the Great Indian Poetry Collectives (GIPCs)

Editor's Choice award. Her first book "Blind Screens" was published in July 2017.

18)Name of the Indian MP who has been bestowed with Sansad Ratna Award under Jury Committee

Special Award category.

Ans:-Anurag Thakur

Explanation:-Himachal Pradesh MP Anurag Thakur has been bestowed with Sansad Ratna Award for his

distinguished performance as a parliamentarian under Jury Committee Special Award category. Thakur

became the first BJP MP to receive the Jury Committee award. 12 parliamentarians were honored with the

Sansad Ratna award, which was set up in 2010 on the suggestion of ex-President APJ Abdul Kalam.

19) India Steel 2019 Exhibition and Conference has started in which of the city?

Ans:-Mumbai

Explanation:-India Steel 2019 Exhibition and Conference had begun in Mumbai. The three-day event

organized by the Ministry of Steel helped chart out the future growth path of the steel industry. The event

provided an opportunity for all the stakeholders to deliberate and identify ways to contribute towards

building a robust roadmap for the further growth of the steel sector. More than 10 thousand business visitors

both from India and abroad are expected to visit.

20)Which state will host the 2nd World Integrated Medicine Forum 2019?

Ans:-Goa

Page 72: Monthly Current Affairs of SEPTEMBER - SSCE

SSCE 8981426494/8296260082 Downloaded from :- www.onlinessce.com Page 72

Explanation:-The 2nd World Integrated Medicine Forum 2019 on the Regulation of Homeopathic Medical

Products will be held in Goa from January 23, 2019. 20 countries will participate in the event. This forum

will strengthen regulatory framework worldwide, to assure that the users of Homoeopathy could have wider

access to quality-assured homeopathic medicines.

Daily Current Affairs 23rd January, 2019

1)Who is the head of the committee which is formed by Madras High Court to weed out invasive plant

species from Western Ghats?

Ans:-Cherukuri Raghavendra Babu

Explanation:-The Madras High Court has expressed serious concern over the exotic invasive species that

have stifled the growth of the native shola forests and grasslands on the Western Ghats. The committee will

be headed by Cherukuri Raghavendra Babu, the chairman of the expert committee on invasive species in

Chennai National Biodiversity Authority. A division bench of justices M M Sundaresh and N Sathish passed

the interim order on a batch of Public Interest Litigation (PIL) petitions seeking to restrain the Tamil Nadu

government from cultivating eucalyptus trees in Western Ghats for commercial purpose.

2)With which Institute Omega Healthcare Management Services has signed an MoU for advanced use

of the latest technology in providing healthcare solutions?

Ans:-IIIT Bangalore

Explanation:-Omega Healthcare Management Services has signed a pact with IIIT-Bangalore for advanced

use of the latest technology in providing healthcare solutions. The MoU is aimed at building industry-grade

AI engines that can be embedded as part of Omega's business solutions to solve healthcare business

challenges. The pact also entails research programmes in artificial intelligence (AI) and Data Science,

aiming to focus on specific business-related processes along with expected outcomes, with the programme

being collectively decided by both organisations.

3)Where did Prime Minister Narendra Modi unveil a statue of Dr Vikram Sarabhai, the father of

Indian space programme?

Ans:-Ahmedabad

Explanation:- The Prime Minister Shri Narendra Modi unveiled a statue of Dr Vikram Sarabhai, the father

of Indian space programme at Sabarmati Riverfront in Ahmedabad, Gujarat. The statue was commissioned

by Amdavad Municipal Corporation (AMC) and was closely supervised by Kartikeya Sarabhai and Mallika

Sarabhai, the son and daughter of Dr. Sarabhai. The statue was installed to mark the innovator's birth

centenary year, and to instill a scientific temperament among Gujarat's children. The statue which has been

sculpted by Ahmedabad-based artist Dhruv Shilpi is 6-ft-tall, made up of bronze statue and weighs 1,050 kg.

It shows Dr Sarabhai seated on a chair and work on his table. A public gallery was also set up adjacent to the

statue featuring Vikram Sarabhai's life and work.

4)Arjuna Award winner Raghbir Singh Bhola passed away recently. He belongs to which sport?

Ans:-Hockey

Explanation:-Former Indian hockey player Raghbir Singh Bhola, a two-time Olympic medallist, has passed

away aged 92. In 2000, he was honored with the Arjuna Award for his achievements in hockey. He served in

Page 73: Monthly Current Affairs of SEPTEMBER - SSCE

SSCE 8981426494/8296260082 Downloaded from :- www.onlinessce.com Page 73

diverse roles such as the member of the IHF selection committee, FIH international umpire, manager of the

Indian hockey team, TV commentator and government observer at the Olympic Games.

5)Which country has taken over the Chair of the Group of 77 (G-77) for 2019?

Ans:-Palestine

Explanation:-The State of Palestine has launched the historical phase by presiding Group of 77 (G-77) for

2019 on January 15. Palestinian President Mahmoud Abbas succeeded Egyptian President Abdel Fattah al-

Sisi with a promise to confront "assaults" on multilateralism and a pledge to seek a peaceful two-state

solution of the Israeli-Palestinian conflict. The decision to elect Palestine as the 2019 Chair of the G77 was

taken in September 2018 by the foreign ministers of the Groups' member states. For this, the UN General

Assembly (UNGA) in 2017 adopted a resolution granting the observer-state of Palestine additional rights to

act as chair of the G-77. The bloc - which through an expanding membership is now comprised of 134

countries - rotates chairmanship duties each year to different members. The G-77 is the global body's largest

intergovernmental organisation of developing countries including India. It is established in 1964 by 77

developing countries in Geneva. It claims to provide the means for the countries of the South to articulate

and promote their collective economic interests and enhance their joint negotiating capacity on all major

international economic issues within the UN system and promote South-South cooperation for development.

6)Which country is the biggest supporters of international aid as per the survey of the World

Economic Forum?

Ans:-India

Explanation:-The World Economic Forum (WEF) released a survey ahead of its annual meeting which

states that Indians are biggest supporters of international aid. 95 per cent of Indians supported the

international aid and expect India to help other nations. World Economic Forum worked with Qualtrics to

poll over 10000 from different parts of the world. South Asian Countries like India, Pakistan and

Bangladesh exhibit widespread support for international aid leaving behind European countries.

7)Which state government to give 10% quota to economically weaker sections in general category?

Ans:-Assam

Explanation:-Assam Cabinet headed by Chief Minister Sarbananda Sonowal has approved 10 percent quota

for the economically weaker sections of the general category in government employment and government

education institutes. The cabinet also approved the guidelines for disbursement of Rs. 5,000 each to five

lakh farmers of the state under theMukhya Mantri Krishi Sa Sajuli Yozana for the purchase of farm tools

and implements. In another decision, the Cabinet approved a loan amount of ₹95 crore under the Rural

Infrastructure Development Fund (RIDF) from NABARD, in-order to construct and strengthen

embankments, dykes and other infrastructure across the state. Furthermore, the cabinet also approved

incentives for government doctors, nurses and other healthcare workers under Ayushman Bharat & Atal

Amrit Abhiyan.

8)The only Indian port which listed amongst the top 30 container ports globally as per Lloyds Report.

Ans:-Jawaharlal Nehru Port Trust

Explanation:-The Jawaharlal Nehru Port Trust (JNPT), India's premier container port got listed amongst the

top 30 container ports globally, as per the latest Lloyds Report. JNPT notched up five spots, to be 28th on

Page 74: Monthly Current Affairs of SEPTEMBER - SSCE

SSCE 8981426494/8296260082 Downloaded from :- www.onlinessce.com Page 74

the list, compared to its previous ranking. This is a validation of all the efforts and strategic initiatives being

implemented at JNPT in order to enhance overall port efficiency.

9)Pakke Paga Hornbill Festival (PPHF) was as the "State Festival" of __________.

Ans:-Arunachal Pradesh

Explanation:-Arunachal Pradesh Chief Minister, Pema Khandu declared Pakke Paga Hornbill Festival

(PPHF) as the "State Festival" at Seijosa in East Kameng district, Arunachal Pradesh. Pakke Paga Hornbill

Festival (PPHF) is only conservation festival of Arunachal Pradesh. The PPHF festival was celebrated first

time in 2015 to recognize the role played by the resident Nyishi tribe in conserving hornbills in the Pakke

Tiger Reserve (PTR).

10)The cabinet of this state has decided to cater 5% reservation each for Kapu community and the

economically backward classes (EBCs).

Ans:-Andhra Pradesh

Explanation:-Andhra Pradesh cabinet has decided to cater 5% reservation each for Kapu community and

the economically backward classes (EBCs) among the forward castes. The cabinet approved the decision of

doubling the welfare pensions given under the NTR Bharosa Scheme. Pensions of Rs 1000 and Rs 1500 are

doubled to Rs 2000 and Rs 3000 respectively. It is believed that 54.61 lakh pensioners will be benefited

from this decision. The cabinet decided to give exemption of life tax for autos and quarterly tax for tractors.

It has also approved the proposal of motor vehicle tax arrears. In total 9.79 lakh vehicle owners will be

benefited to the extent of Rs 66.50 crores.

11)How many MoU's signed between India and Afghanistan, recently?

Ans:-11

Explanation:-India signed 11 Memorandum of Understandings (MoUs) with Afghanistan for 26 projects

worth 9.5 million dollars in the field of infrastructure, public services such as health clinics and governance-

related projects. The MoUs signed will involve the construction of classrooms, orphanages, health clinics,

canal protection walls and enhancing economic opportunities for women. The projects will be developed in

seven provinces of Afghanistan- Balkh, Ghor, Herat, Kabul, Bamyan, Badghis and Kapisa. It will help local

communities in education, employment and livelihood generation and capacity building. These projects

which are part of 577 community-based Small Development Projects (SDP) are being funded by India from

2005-2021 and worth $120 million.

12)This company has decided to invest $ 400 million dollars for over 40 % stake in BrainBees

Solutions.

Ans:-SoftBank

Explanation:--SoftBank has decided to invest $400 million for over 40 per cent stake in BrainBees

Solutions, which owns and operates omni-channel baby and mother care product retailer FirstCry. SoftBank

is valuing 8 years old Pune based venture, FirstCry at $600-700 million and planning to pick over 40 per

cent stake whereas FirstCry Founder, Supam Maheshwari and Amitava Saha will retain 12-14 per cent

stake. As per SoftBank officials the money will come in tranches linked to financial and business

performance milestones. Morgan Stanley acted as the Financial advisor for this big deal.

Page 75: Monthly Current Affairs of SEPTEMBER - SSCE

SSCE 8981426494/8296260082 Downloaded from :- www.onlinessce.com Page 75

13)Which company will create an independent Institute for Ethics in Artificial intelligence (AI) with

an investment of $7.5 million?

Ans:-Facebook

Explanation:-Social media giant Facebook announced that it will create an independent Institute for Ethics

in Artificial intelligence (AI) with an initial investment of $7.5 million over a period of five years. Technical

University of Munich (TUM) in Germany will collaborate with this project which aims to explore

fundamental issues affecting the use and impact of AI. The Institute will address issues that affecting the use

and impact of AI such as Safety, Privacy, Fairness, and transparency. TUM was ranked 6th in the world for

AI research by the Times Higher Education magazine in 2018.

14)World's oldest man who died due to natural causes in Japan at the age of 113?

Ans:-Masazo Nonaka

Explanation:-World's oldest man Masazo Nonaka died due to natural causes in the Japan's northern most

island of Hokkaido at the age of 113. Masazo Nonaka was born on 25 July 1905 and have lived with his four

generation family. Japan has one of the world's highest life expectancies. The Longest living man on record

Jiroemon Kimura who died after his 116th birthday in June 2013 was also from Japan.

15)Sitanshu Yashashchandra has been awarded Saraswati Samman, 2017, for his poetry collection

"Vakhar". He belongs to the state of;

Ans:-Gujarat

Explanation:-Noted Gujarati poet Sitanshu Yashashchandra has been awarded Saraswati Samman, 2017,

for his poetry collection in Gujarati named "Vakhar". This is the 27th edition of the prestigious award

instituted by K.K.Birla foundation given every year to an outstanding work of an Indian citizen published

during the last 10 years in any of the languages mentioned in Schedule 8 of the constitution. The award

comprises of a citation and a plaque apart from award money of 15 lakh rupees. Noted film director and

author Gulzar gave away the award at a solemn function at National museum auditorium in New Delhi. Mr.

Yashashchandra, recipient of the Padmashri in 2006, is known for seamlessly blending history with poetry.

16)India ranks __________ in the global talent competitive index.

Ans:-80

Explanation:-India has moved up one position to rank 80th on the global talent competitive index, but

remains a laggard among the BRICS nations. Switzerland continues to top the list, released on the first day

of the World Economic Forum (WEF) Annual Meeting at Davos by INSEAD business school in partnership

with Tata Communications and Adecco Group.

17)The government has constituted a high power committee to monitor the pricing of medicines and

healthcare products. It will be headed by;

Ans:-NITI Aayog

Explanation:-The government has constituted a high power committee to monitor the pricing of medicines

and healthcare products. The committee will be headed by a Niti Aayog member (Health). It will be tasked

with recommending price controls and monitoring the prices of specific drugs and health products to ensure

their affordability. The Standing Committee on Affordable Medicines and Health Products (SCAMHP) will

Page 76: Monthly Current Affairs of SEPTEMBER - SSCE

SSCE 8981426494/8296260082 Downloaded from :- www.onlinessce.com Page 76

be the recommending body to the National Pharmaceutical Pricing Authority (NPPA). The committee will

also include the chief economic adviser and other policymakers.

18)Who conferred the Pradhan Mantri Rashtriya Bal Puraskar 2019.

Ans:-Shri Ram Nath Kovind

Explanation:- The President of India, Ram Nath Kovind conferred the Pradhan Mantri Rashtriya Bal

Puraskar 2019. The awards were presented to 26 shortlisted awardees including one joint award for National

Child Award (now renamed as Bal Shakti Puraskar) under the category of innovation, scholastic, sports, art

and culture, social service and bravery. Under National Child Welfare Awards category, 2 individuals and 3

institutions were awarded. The names of awardees were finalized by the National Selection Committee

under the Chairpersonship of Minister of Women and Child Development, Maneka Sanjay Gandhi.

19)Who has become the first woman IPS DIG and Indo-Tibetan Border Police (ITBP) officer to

successfully complete the South Pole expedition?

Ans:-Aparna Kumar

Explanation:-Aparna Kumar has become the first woman IPS DIG and Indo-Tibetan Border Police (ITBP)

officer to successfully complete the South Pole expedition. Ms Kumar has successfully reached the South

Pole after covering 111 kilometres of walking on the snow. She also carried equipment weighing 35

Kilograms along with her. She has already scaled the top six mountain peaks of the six continents of the

globe. Ms Kumar reached the South Pole on January 13 where she unfurled the National and ITBP flags. Ms

Kumar is a 2002 batch UP Cadre IPS officer, who has been posted at the Northern Frontier Headquarters of

ITBP in Dehradun.

20)Who was named as ICC's Emerging Player Of The Year?

Ans:-Rishabh Pant

Explanation:-Rishabh Pant was named the International Cricket Council's (ICC) Emerging Player Of The

Year following an impressive show in India's first-ever Test series win in Australia. The 21-year-old

wicketkeeper-batsman scored 350 runs in the four-match Test series that India won 2-1 to script history in

71 years and 11 attempts. With an unbeaten 159-run knock in the 4th Test in Sydney, Rishabh Pant became

the first Indian wicket-keeper to score a Test century in Australia. Earlier, he equalled the record for the

most catches taken in a Test with 11 against Australia in Adelaide in December.

Daily Current Affairs 24th January, 2019

1)Where did Insolvency and Bankruptcy Board of India organise the Insolvency and Bankruptcy

Awareness Programme?

Ans:-Vadodara, Gujarat

Explanation:-The Insolvency and Bankruptcy Board of India organised an Insolvency and Bankruptcy

Awareness Programme at Vadodara, Gujarat.It is organised in collabration with three Insolvency

Professional Agencies, namely, the Indian Institute of Insolvency Professionals of ICAI (lead partner), the

ICSI Institute of Insolvency Professionals, and the Insolvency Professional Agency of Institute of Cost

Accountants of India.

Page 77: Monthly Current Affairs of SEPTEMBER - SSCE

SSCE 8981426494/8296260082 Downloaded from :- www.onlinessce.com Page 77

2)Vice President M. Venkaiah Naidu launched the book 'Universal Brotherhood Through Yoga' in

which of the following city?

Ans:-New Delhi

Explanation:-Vice President M. Venkaiah Naidu launched the book 'Universal Brotherhood Through Yoga'

in New Delhi. The book is compiled by Bharatiya Sanskrit Pitham.

3)How many Indians won the prestigious Pravasi Bharatiya Samman Award?

Ans:-3

Explanation:-Three Indian expats from the UAE are amongst those who won the prestigious Pravasi

Bharatiya Samman Award the highest honor conferred on Indian diaspora. President Ram Nath Kovind

conferred the award on Girish Pant, Surender Singh Kandhari and Dr. Zulekha Daud and other awardees at

the Pravasi Bharatiya Divas (PBD) Convention in Varanasi.

4)Which country successfully tested the Arrow 3 missile system?

Ans:-Israel

Explanation:-Israel successfully tested the Arrow 3 missile system from Central Israel,it is done in

collaboration with American Missile Defense Agency. Arrow 3 is considered to be the next step in the

defence system,which would help the Israel Miltary Force to attack their enemies from a greater distance

and greater height. It is a long range anti-ballastic missile.

5)Who inaugurated the first inter-state bridge over Ravi River in J&K?

Ans:-Nitin Gadkari

Explanation:-Nitin Gadkari inaugurated first inter-state 1210-meter span-bridge over Ravi River in Keerian

Gandial in Kathua district,jammu and kashmir. It was built at a cost of Rs 158.84 crore in a span of three and

a half years linking areas of Katthua district with Punjab. The bridge also called as Keediyan-Gandiyal

bridge will benefit over 2,20,000 people living on the two sides - Kathua in Jammu, and Pathankot in

Punjab. It will reduce the distance between the two cities from 45 kms to 8.6 kms.

6)Which ministry constituted a committee to study the existing Special Economical Zone(SEZ) policy

of India?

Ans:-Ministry of Commerce and Industry

Explanation:-Baba Kalyani led committee constituted by the Ministry of Commerce and Industry to study

the existing Special Economical Zone(SEZ) policy of India submitted its report to Union Minister for

Commerce & Industry and Civil Aviation Suresh Prabhu in New Delhi. The reports of the committee were

made public on 22nd January and government is seeking suggestions or comments on the recommendations

of the committee till 30th January 2019. Objectives of the Baba Kalyani led committee which was

constituted in June 2018 were to evaluate the SEZ policy and make it World Trade Organization (WTO)

compatible, suggest measures to maximize utilization of vacant land in SEZs. The report says that if India is

going to become a USD 5 trillion economy by 2025 then the current environment of manufacturing

competitiveness and services has to undergo a basic paradigm shift.

Page 78: Monthly Current Affairs of SEPTEMBER - SSCE

SSCE 8981426494/8296260082 Downloaded from :- www.onlinessce.com Page 78

7)Who has conducted the massive coastal defence exercise 'Sea Vigil-2019' to check 26/11-style

attacks?

Asn:-Indian Navy

Explanation:-The Indian Navy, in coordination with the Indian Coast Guard (ICG) and other stakeholders,

conducted the massive coastal defence exercise codenamed 'SEA VIGIL-2019'off the Indian coast on

January 22-23, 2019. The exercise, a first of its kind, covered the entire 7516.6 km coastline and Exclusive

Economic Zone of India and involves all the 13 coastal States and Union Territories along with all maritime

stakeholders, including the fishing and coastal communities. The main goal of the large-scale exercise is to

comprehensively and holistically validate the efficacy of the measures taken since '26/11'. The exercise also

aims to test India's preparedness to thwart any attempt by anti-national elements to carry out an attack on its

territory or against its citizens by infiltrating through the sea route.

8)Uttar Pradesh government announced a monthly pension of __________ rupees to the destitute

women in Varanasi.

Ans:-Rs 500

Explanation:-UP government announced to give a monthly pension of Rs.500 to the destitute women in

Varanasi. This measure came immediately after the implementation of 10 per cent reservation for

economically weak in the general category in the upcoming educational year. The Parliament passed the

Constitution (124th Amendment) Bill, 2019 in Parliament to grant 10 reservation in education and

government jobs to economically weaker individuals belonging to the general category, across religions.The

reservation is meant for individuals whose annual earning is below Rs 8 lakh and who possess less than 5

acres of agriculture land. Yogi Adityanath also announced pension scheme to Hindu seers who are above the

age of 60. The scheme is also likely to benefit the seers in Uttar Pradesh. The pension will be provided to

Hindu sadhus who are above the age of 60.

9)Manipur, Meghalaya and Tripura celebrate their 42nd statehood day on __________.

Ans:-January 21

Explanation:-Manipur, Meghalaya and Tripura are celebrating their statehood day. It is their 42nd

statehood day. In the year 1972, all the three states became full-fledged states under the North Eastern

Region (Reorganisation) Act, 1971. The erstwhile princely states of Tripura and Manipur were merged with

India in 1949 and became full-fledged states in 1972. Meghalaya was a part of Assam before it was carved

out as a full-fledged state under the North Eastern Region (Re-Organisation) Act, 1971.

10)ISRO will launch Microsat-R a military imaging satellite into space on 24th January 2019. The

total weight of the staellite is;

Ans:-130 kg

Explanation:- Indian Space Research Organisation's (ISRO) first mission of 2019, Microsat-R, is to be

launched into space on 24th January 2019. The 130-kg Microsat-R is a military imaging satellite. Microsat-

R and its payload come assembled from a handful of laboratories of the Defence Research and Development

Organisation (DRDO) and is meant for military use. The satellite was assembled outside and ISRO only

interfaced it with its own systems and the launch vehicle, just as it treats any customer satellite. PSLV-C44

will be launched from the older First Launch Pad at the Satish Dhawan Space Centre, Sriharikota in Andhra

Pradesh. For its part, ISRO is experimenting on two aspects of the vehicle. One is to reuse a waste stage.

Page 79: Monthly Current Affairs of SEPTEMBER - SSCE

SSCE 8981426494/8296260082 Downloaded from :- www.onlinessce.com Page 79

11)Indian Africa Field Training Exercise (IAFTX)-2019 was conducted in which city?

Ans:-Pune

Explanation:-The final planning conference to work out the modalities for Indian Africa Field Training

Exercise (IAFTX)-2019 was conducted at Pune, Maharashtra. Delegates from Egypt, Ghana, Nigeria,

Senegal, Sudan, South Africa, Tanzania, Namibia, Mozambique, Uganda, Niger & Zambia attended the

conference. The joint training exercise is being conducted with more than a dozen African countries & India.

It aims at synergysing humanitarian mine action and joint peace operations. The initial planning conference

for the exercise was held in December 2018.

12)India placed __________ in the world's most innovative countries according to the Bloomberg 2019

Innovative Index.

Ans:-54th

Explanation:-India ranked at the 54th Position with a score of 47.93 out of 100 among the world's most

innovative countries according to the Bloomberg 2019 Innovative Index. South Korea retained its top spot

with a score of 87.38, Germany ranked second with a score of 87.30 whereas Finland has been ranked at the

third spot with a score of 85.70. The Annual Bloomberg Innovation Index ranks countries by analyzing their

performances using seven metrics including Research and Development spending, Manufacturing

Categories and Concentration of high-tech public computers.

13)Tamil Nadu's "Jallikattu" held in __________, has entered the world record for the maximum

number of bulls released into the sport arena.

Ans:-Pudukottai

Explanation:-Jallikattu as a symbol of the valour and courage of the Tamil people. A grand "jallikattu"

(bull-taming) event held in Pudukottai and this event entered the world record for the maximum number of

bulls released into the sport arena.This event had the participation of 1,354 bulls and 424 tamers.

14)The Netaji Subhash Chandra Bose museum has inagurated in which city?

Ans:-New Delhi

Explanation:-Prime Minister Narendra Modi inaugurated the Subhash Chandra Bose museum at the iconic

Red Fort in New Delhi to mark the leader's 122nd birth anniversary. The Museum on Netaji Subhash

Chandra Bose and Indian National Army provides a detailed account of Subhash Chandra Bose and the

history of Indian National Army (INA). It also showcases various artefacts related to Subhash Chandra Bose

and INA. The artefacts include wooden chair and sword used by Netaji, medals, badges, uniforms and other

artefacts related to INA. Beside this, Prime Minister also inaugurated the Yaad-e-Jallian Museum on the

Jallianwala Bagh and World War I, the Museum on 1857- India's first war of Independence and

Drishyakala-Museum on Indian Art within the Red Fort complex.

15)With which country the Union Cabinet has approved MoU for cooperation on the recruitment of

domestic workers?

Ans:-Kuwait

Explanation:-The Union Cabinet has approved recently the signing of Memorandum of Understanding

(MoU) between India and Kuwait for cooperation on the recruitment of domestic workers. The MOU

Page 80: Monthly Current Affairs of SEPTEMBER - SSCE

SSCE 8981426494/8296260082 Downloaded from :- www.onlinessce.com Page 80

provides a structured framework for cooperation on domestic workers related matters and provides

strengthened safeguards for Indian domestic workers including female workers deployed in Kuwait. The

MOU is initially valid for a period of 5 years and incorporates provision for automatic renewal. Around

three lakh Indian domestic workers deployed in Kuwait will benefit from the decision. The MOU will

promote bilateral cooperation in domestic workers related matters between the two countries.

16)The theme of National Girl Child Day 2019 was;

Ans:-Empowering Girls for a Brighter Tomorrow

Explanation:-National Girl Child Day is celebrated in India on 24 January. Women and Child Development

Ministry celebrated the National Girl Child Day 2019 at Pravasi Bharti Kendra, New Delhi. The programme

also observed the anniversary of Beti Bachao Beti Padhao (BBBP) Scheme. The theme of National Girl

Child Day 2019 was "Empowering Girls for a Brighter Tomorrow" and celebrating with objectives of

generating awareness on the issue of declining Child Sex Ratio (CSR) and create a positive environment

around valuing the girl child. A booklet on "Innovations under BBBP" will also be released on the occasion.

17)Where was the 2nd World Integrated Medicine Forum 2019 held?

Ans:-Goa

Explanation:-Union Minister of State (Independent Charge) for AYUSH, Shripad Naik inaugurated 2nd

World Integrated Medicine Forum 2019 in Goa. International drug regulators who are dealing with

homeopathic or traditional medicines from various countries participated in the forum. It is organized by the

Central Council for Research in Homeopathy under the Ministry of AYUSH along with World Integrated

Medicine Forum. The forum is expected to witness participation from 20 countries. The theme for the forum

is ' Regulation of Homeopathic Medical Products; Advancing global collaboration'.

18)Indian and Japanese Coast Guards undertake disaster control exercise in which city?

Ans:- Yokohama

Explanation:-Indian and Japanese Coast Guards started disaster, search and rescue exercises off the cost of

Yokohama in Japan. ICGS Shaunak is the main Indian Coast Guard Ship taking part in the drills. Rajendra

Singh, Chief of Indian Coast Guard and his Japanese counterpart Shuichi Iwanami reviewed the drills. The

Coast guards of India and Japan was working together after the establishment of deeper ties between the two

countries. Japan Coast guard visited India last year whereas in this year Indian coast guard has visited Japan.

19)Which high court released a movie "72 Hours - Martyr Who Never Died" a movie about Sino-

Indian war hero Rifleman Jaswant Sing Rawat?

Ans:-Delhi High Court

Explanation:-The Delhi High Court allowed the release of the movie '72 Hours - Martyr Who Never died'

which is based on the life of the 1962 Sino-Indian war hero Rifleman Jaswant Sing Rawat who was

posthumously awarded the Maha Vir Chakra for gallantry during 1962 war. Rifleman Jaswant Singh Rawat

was a Sino-Indian war hero who was serving as an Indian army soldier in Garhwal Rifles. He was serving

the 4th battalion during the battle of Nuranang of the Nort- East Frontier Agency in the war of 1962, and

now it's in Arunachal Pradesh. He was honored by building a memorial by the Chinese people's Liberation

army and it was named as Jaswant Garh.

Page 81: Monthly Current Affairs of SEPTEMBER - SSCE

SSCE 8981426494/8296260082 Downloaded from :- www.onlinessce.com Page 81

20)Name the Indian tennis player who won the International Tennis Federation's (ITF) 25,000-dollar

Women's tournament in Singapore.

Ans:-Ankita Raina

Explanation:-India's Ankita Raina won the International Tennis Federation's (ITF) 25,000-dollar Women's

tournament in Singapore by defeating top seed and world number 122 Arantxa Rus of the Netherlands, by 6-

3, 6-2. Besides being the first title of the season, it is Ankita's fourth title at the USD 25k level, and eighth

overall. Ankita has earned 50 points from this win and has achieved a jump of 35 places to a career-high 168

in the latest WTA singles rankings issued on 21st January 2019. She will next play the Fed Cup, starting in

the first week of next month at Astana, Kazakhstan.

Daily Current Affairs 25th January, 2019

1)Every 25 January is celebrated as National Voters Day. The Theme of 2019 National Voters Day is;

Ans:-No Voter to be left behind

Explanation:-25 January is celebrated every year as National Voters Day to spread awareness about the

necessity of voting and to encourage young generation towards the voting rights. National Voters Day

Theme 2019 is "No Voter to be left behind". This is the ninth year of celebrating National Voters Day. 25th

January is the foundation day of the Election Commission of India (ECI) which came into existence in 1950.

This day was first celebrated in 2011 to encourage as much as young voters to take part in the electoral

process. Earlier the eligibility age of the voter was 21 years but in 1988 it was lowered to 18 years.

2)Who inaugurates National Declamations Contest on Patriotism and nation building?

Ans:-Shri Rajyavardhan Rathore

Explanation:-Minister of state of Youth affairs and Sports (Independent Charge) Rajyavardhan Rathore

inaugurates National declamation contest on Patriotism and nation building. It was organized by Nehru

Yuva Kendra Sangathan (NYKC) along with Department of Youth affairs. The contest has been conducting

since 2015-16. Minister said that India is a country of diversities and therefore symbols like National Flag

and National Another are necessary to inculcate feeling of belongings and Patriotism. It aims to further

strengthen the spirit of nationalism in youth and whole hearted participation in nation building. It will enable

the government to take lead in understanding the functions and policies of the government towards nation

building.

3)Recently, DRDO Launched Long Range Surface to Air Missile (LR-SAM). In which coast it was

launched?

Ans:-Odisha Coast

Explanation:-The Defence Research and Development Organisation (DRDO) has successfully test fired

Long Range Surface to Air Missile (LR-SAM) from naval warship INS Chennai. The test was conducted off

the coast of Odisha. The LR-SAM hit a low flying aerial target during the test.

4)When was the International Day of Education is observed?

Ans:-24th January

Page 82: Monthly Current Affairs of SEPTEMBER - SSCE

SSCE 8981426494/8296260082 Downloaded from :- www.onlinessce.com Page 82

Explanation:-International Day of Education is observed on 24 January. The aim of the day is to celebrate

the role of education for peace & development. The United Nations General Assembly on December 3

adopted a resolution declaring January 24 as International Day of Education. In India, National Education

Day is observed on 11 November to commemorate the birth anniversary of Maulana Abul Kalam Azad.

5)Who will be conferred with the Ashoka Chakra?

Ans:-Lance Naik Nazir Ahmad Wani

Explanation:-Lance Naik Nazir Ahmad Wani will be conferred with the Ashoka Chakra. He was a militant

but later joined the Indian Army in 2004. He was killed during an anti-terror operation in November 2018.

He was also awarded the Sena Medal. Ashok Chakra is the highest peacetime military decoration for valour,

courageous action or self-sacrifice away from the battlefield.

6)With which country, The Union Cabinet of India approved MoC in the field of food processing

sector?

Ans:-Japan

Explanation:-The Union Cabinet approved the Memorandum of Cooperation (MoC) between India and

Japan in the field of food processing sector. The MoC will help in increasing food processing in India by

getting access to best practices and better markets The MoC will lead to the betterment of the food

processing sector in India by introducing innovative techniques and processes.

7)Himachal Pradesh has decided to host a global investor summit in which city?

Ans:-Dharamsala

Explanation:-Himachal Pradesh Chief Minister Jai Ram Thakur announced that the state will be holding a

global investor summit in Dharamsala on June 10 and 11, 2019. He launched the website and mobile app of

the global investor summit. The state has created a land bank for allocation to industries.

8)Worlds longest 3D printed concrete bridge opened in which country?

Ans:-China

Explanation:-World's longest 3D printed concrete bridge was opened in China. It is located on the

Wenzaobang River in Shanghai. The Bridge is designed by Professor Xy Weiguo from Tsinghua University

in China. The bridge is of 26.3 metre-long and 3.6-metre-wide and is made out of 44 hollowed-out 3D

printed concrete blocks

9)Who has been appointed as the new MD & CEO of Yes Bank after Rana Kapoor?

Ans:-Ravneet Singh Gill

Explanation:-Yes Bank has appointed veteran banker Ravneet Singh Gill as its new MD & CEO and the

lender has also received the RBI's approval for the same.He will join the bank on or before March 1, 2019.

Mr Gill is currently the chief Deutsche Bank's India operations. Gill joined Deutsche Bank in 1991 and has

worked across different businesses including corporate banking, capital markets and wealth management.

10)World Health Organization declared the year 2019 as;

Page 83: Monthly Current Affairs of SEPTEMBER - SSCE

SSCE 8981426494/8296260082 Downloaded from :- www.onlinessce.com Page 83

Ans:-Year of action on preparedness for health emergencies

Explanation:-The World Health Organization released a list of top ten threats to global health in 2019 and

considered vaccination to be the effective weapon to prevent the same. Air pollution and climate change: it

is considered as a major risk factor to human as the micro pollutants from various things affect every single

part of the human cell. Burning Fossils is the major contributor. Noncommunicable

diseases:Noncommunicable diseases, such as diabetes, cancer and heart disease, are collectively responsible

for over 70% of all deaths worldwide. This disease are caused due to the inactivity of the body so WHO

decided in collaboration with government help them achieved the global target of reducing physical

inactivity by 15% by 2030. Global influenza pandemic:WHO has set up a unique partnership which will

ensure effective and equitable access to vaccines in developing countries. Vulnerable environment:WHO

will continue to work in developing countries to strengthen health systems so that they are able to respond to

outbreak of certain viruses.

11)Famous cricket player Johan Botha has announced his immediate retirement from all forms of

Cricket. He belongs to which country?

Ans:-South Africa

Explanation:-Former South Africa all-rounder Johan Botha has announced his immediate retirement from

all forms of Cricket. Botha played for South Africa in 5 test matches, 78 one-day internationals, and 40 T20

Internationals from 2005 to 2012. Botha captained the Proteas in 10 ODIs and took the team to the No. 1

ranking after they defeated Australia 4-1 in Australia in 2009.

12)Who has been appointed as the interim finance and corporate affairs minister by the President of

India?

Ans:-Shri Piyush Goyal

Explanation:-President of India appointed Piyush Goyal as the interim finance and corporate affairs

minister because of Arun Jaitely's health issues. Piyush Goyal's tenure as an interim Finance Minister will be

till August 2019. Jaitley will be designated as a minister without a portfolio during his period of

indisposition. Being the interim finance and corporate affair minister, he will present the budget on

February.

13)Union Minister Nitin Gadkari laid the foundation stone for 6 Namami Ganga projects in Agra and

___________ in Uttar Pradesh.

Ans:-Mathura

Explanation:-Nitin Gadkari, Minister for Road Transport & Highways, Shipping and Water Resources,

River Development & Ganga Rejuvenation laid foundation stone for six Namami Ganga project in Agra and

Mathura in Uttar Pradesh. There were four projects in Mathura that cost of Rs 511.74 crore. Out of the

four,two were sewage projects. The first one was a integrated sewerage infrastructure in the city on Hybrid

Annuity Mode with the motive of 'One-City-One-Operator'. There is also a project for cleaning of 27 Ghats

at a cost of Rs 3.60 Crore and to erdicte of Industrial Pollution, there is also a project for upgradation of

infrastructure of Textile Printing units at the Mathura Industrial Area at a cost of Rs 13.87 Crore. The

projects in Agra included Sewage Scheme at 857.26cr and laying sewer house connections under the

AMURT scheme at a cost of 353.57cr.

14)The Railway Protection Force (RPF) has launched a dedicated RPF Women Squad named as

'Joymati Bahini' at which city?

Page 84: Monthly Current Affairs of SEPTEMBER - SSCE

SSCE 8981426494/8296260082 Downloaded from :- www.onlinessce.com Page 84

Ans:-Guwahati

Explanation:-The Railway Protection Force (RPF) has launched a dedicated RPF Women Squad named as

'Joymati Bahini' at Kamkhaya Railway Station in Guwahati on 23 January 2019. The squad comprises

women sub-inspectors and constables of the Railway Protection Force. The main aim of the 'Joymati Bahini'

is to strengthen women security around Guwahati region within railway premises and trains.

15)Ronojoy Dutta was appointed as the new CEO of which of the following company?

Ans:-IndiGo

Explanation:-IndiGo owner InterGlobe Aviation has appointed Ronojoy Dutta as the new Chief Executive

Officer (CEO) of the company with immediate effect. The company has also approved the appointment of

Meleveetil Damodaran, the Independent Non-Executive Director, as the Chairman of the Board of Directors

of the Company.

16)Name the defence company who signed a MoU with the Airports Authority of India to explore Air

Traffic Management (ATM) solutions for airports operating under the UDAN Regional Connectivity

Scheme.

Ans:-SAAB

Explanation:- Swedish aerospace and defence company SAAB signed a MoU with the Airports Authority

of India (AAI) on 23 January 2019. The MoU was signed to explore Air Traffic Management (ATM)

solutions for airports operating under the UDAN Regional Connectivity Scheme. SAAB's ATM solutions

are now deployed in Ahmedabad, Amritsar, Guwahati, Jaipur, Lucknow, Chennai, Kolkata, Mumbai, New

Delhi, and Cochin.

17)According to the report of United Nations' World Economic Situation and Prospects (WESP) 2019,

what will be the growth rate of India for in 2019-20?

Ans:-7.6%

Explanation:-With robust private consumption and 7.4% GDP growth rate in the current fiscal, the Indian

economy is expected to accelerate at a rate of 7.6% in 2019-20, according to a report by the United Nations

titled United Nation's World Economic Situation and Prospects (WESP) 2019. As per the report, the world

trade growth moderated over the course of 2018 to 3.8% from growth of 5.3% in 2017.

18)Where was the new Naval Air Base 'INS Kohassa' commissioned?

Ans:-Andaman and Nicobar Islands

Explanation:-Navy Chief Admiral Sunil Lanba commissioned the new Naval Air Base 'INS Kohassa' in

Andaman and Nicobar Islands on 24 January 2019. The airbase will be used to handle operations of

helicopters and small aircraft like the Dornier. INS Kohassa will be India's fourth air base and the third naval

air facility in the Andamans that overlooks key sea lanes of communication and strategic points.

19)Who has decided to form a 'State Tiger Protection Force' to save the Big Cat population in the

State?

Ans:-Telangana

Page 85: Monthly Current Affairs of SEPTEMBER - SSCE

SSCE 8981426494/8296260082 Downloaded from :- www.onlinessce.com Page 85

Explanation:-The Telangana government has decided to form a 'State Tiger Protection Force' to save the

Big Cat population in the State. The State Forest Protection Committee headed by Chief Secretary S K Joshi

took the decision to form the STPF. The 112-member-armed STPF will be headed by an Assistant

Conservator of Forests to protect the tiger population in Amarabad and Kawal Tiger Reserves. The State and

Central government will share the cost of the force on 40:60 basis. Fund amount of 2.25 Cr was also

sanctioned to protect the tigers and to prevent forest fire accidents. The Committee decided to act tough

against those felling forest trees and committing other related crimes.

20)8th Battalion of National Disaster Response Force has been selected for the Award "Subhash

Chandra Bose Aapda Prabandhan Puraskar". It was located in;

Ans:-Ghaziabad

Explanation:- The government has instituted an annual award titled Subhash Chandra Bose Aapda

Prabandhan Puraskar. It will be announced every year on 23rd January, the birth anniversary of Netaji

Subhash Chandra Bose. The Award carries a certificate and a cash prize of 51 lakh rupees. This year, 8th

Battalion of National Disaster Response Force located at Ghaziabad has been selected for the Award for its

commendable work in disaster management. The award has been instituted to acknowledge the contribution

and selfless service of organizations and individuals to humanity in the aftermath of any disaster.

Daily Current Affairs 26th January, 2019

1)Who will be the Chief Guest of India's Republic Day Parade?

Ans:-Cyril Ramaphosa

Explanation:- South Africa President Cyril Ramaphosa will be the Chief Guest at the Republic Day Parade.

He is the second President of South Africa after Nelson Mandela to be the Chief Guest at the parade.

President Ramaphosa will be accorded ceremonial welcome at Rashtrapati Bhawan Friday morning. Prime

Minister Narendra Modi and President Ramaphosa will hold delegation level talks on bilateral, regional and

global issues of mutual interest. Both the leaders will also address the India-South Africa Business Forum

today with the objective to grow business ties between the two countries. AIR correspondent reports,

relations between India and South Africa are age old. Father of the Nation Mahatma Gandhi is revered in the

African nation as he had an important role in the non-violent movement in South Africa. India was at the

forefront of the international community in its support to the anti-apartheid movement there. India is among

the top five trading partners of South Africa.

2)How many Technology Mission Centre was launched by the Union Minister Harsh Vardhan?

Ans:-3

Explanation:-Union Minister Harsh Vardhan launched three important centre's set up by the Department of

Science and Technology (DST) at IIT Madras on 25 January 2019. The first one is the 'DST -IITM Solar

Energy Harnessing Centre'. The second centre is the 'DST-IITM Water -IC for SUTRAM of EASY

WATER'. The third centre is the 'Testbed on Solar thermal desalination solutions'.

3)Which IIT has launched an app 'Roshni' to help blind identify notes?

Ans:-IIT Ropar

Explanation:-IIT Ropar has launched an android app 'Roshni' to help the blind in recognizing the

denomination of the currency notes, using image processing and analytics. At present, notes of Rs 10, 20,

Page 86: Monthly Current Affairs of SEPTEMBER - SSCE

SSCE 8981426494/8296260082 Downloaded from :- www.onlinessce.com Page 86

50, 100, 200, 500 and 2,000 are in circulation, which the blind people find it difficult to recognize due to

their size and the tactile marks. The app can recognize both old and new notes and gives audio information

as to which note you are holding in your hands. The user has to bring the currency note in front of phone

camera and the app would provide audio notification intimating the currency notes denomination to the user.

The app uses the patterns and features embedded on the notes to differentiate and determine the currency

denomination.

4)Which country will organise a Bhagwad Gita Mahotsav in partnership with the Haryana

Government?

Ans:-Mauritius

Explanation:-Mauritius Prime Minister, Mr. Pravind Jugnauth announced in his address as the Chief Guest

at inauguration of the 15th Pravasi Bharatiya Divas at Varanasi, Uttar Pradesh that Mauritius will organize

the first Bhojpuri Festival in 2020. It was also announced by Mr. Pravind Jaugnauth that Mauritius will hold

a Bhagwad Gita Mahotsav in partnership with the Haryana Government in February 2019.

5)Which insurance company has overtaken Life Insurance Corporation of India, recently?

Ans:- Max Life Insurance

Explanation:-Private life insurer Max Life Insurance has overtaken Life Insurance Corporation of India

(LIC) to the best track record in terms of claims settlement for individual deaths in FY18. According to

IRDAI's Annual Report 2017-18, Max Life settled 98.26 percent of individual death claims while LIC

settled 98.04 percent.

6)Centre launched Apollo Proton Cancer Centre (APCC) in which of the following city?

Ans:-Chennai

Explanation:-India launched Apollo Proton Cancer Centre (APCC) in Chennai, Tamil Nadu. With this

launch, India has become the 16th country in the world to offer proton therapy for cancer. The Centre offers

proton therapy with pencil-beam technology that provides a high degree of precision. APCC is a 150-bed

integrated cancer care Centre. It has the Intensity Modulated Proton Therapy and Image-Guided

Radiotherapy component. At present, there are 72-odd facilities operational across the globe. Around 60 to

70% of cancer patients need radiation. Through proton beam therapy, which is a form of radiation, zero dose

beyond the tumour is achievable.

7)Which novel has been adjudged as the best novel of 2017 at the Kerala Sahitya Akademi Awards?

Ans:-Nireeswaran

Explanation:-The Kerala Sahitya Akademi Awards for the year 2017 have been announced on 23rd January

2019. In it, 'Nireeswaran' by V.J. James has been adjudged the best novel of 2017, while Aymanam John's

Ethara Characharangalude Charitra Pusthakam has secured the award for the best short story. Mindaprani,

written by Veerankutty, is the best poetry. The award is given each year by the Kerala Sahitya Akademi

(Kerala Literary Academy) to Malayalam writers for their outstanding books of literary merit.

8)Sultan Abdullah was elected as the King of which of the following country?

Ans:-Malaysia

Page 87: Monthly Current Affairs of SEPTEMBER - SSCE

SSCE 8981426494/8296260082 Downloaded from :- www.onlinessce.com Page 87

Explanation:-Sultan Abdullah was elected as Malaysia's new King on 24 January 2019, following the

abdication of Sultan Muhammad V in January 2019. He is the ruler of the Malaysia state of Pahang. He will

have a five-year term. He is the current President of the Asian Hockey Federation and a council member of

FIFA, the world football body.

9)India's first woman IPS officer who completes South Pole expedition.

Ans:-Aparna Kumar

Explanation:-Aparna Kumar has become the India's first woman Indian Police Service (IPS) DIG and Indo-

Tibetan Border Police (ITBP) officer to successfully complete the South Pole expedition. She unfurled the

National and the ITBP flags at the South Pole on 13th January 2019 after covering 111Km of walking on the

snow. Aparna will now attempt to conquer the North Pole this April. She has covered six of the highest

peaks across continents including the Mount Everest. The one left for her to scale is the Mount Denali in

Alaska.The ITBP is a Central Armed Police Force which is primarily deployed to secure the icy Himalayan

borders of the nation since its Inception in 1962.

10)Ministry of Agriculture & Farmers Welfare organized a National Conference on Agriculture for

Zaid Campaign-2019 in __________.

Ans:-New Delhi

Explanation:- National Conference on Agriculture for Zaid Campaign-2019 was organised by the Ministry

of Agriculture & Farmers Welfare in New Delhi on 24 January 2019. Special sessions were held to discuss

strategy for potential crops and irrigation potential for zaid season. It was also said that the total Foodgrain

production in the country is estimated at 284.83 million tonnes in 2017-18.

11)Who has been awarded as 'Best Electoral Practices Award' in Andhra Praadesh?

Ans:-B.Lakshmikantham

Explanation:-Vijayawada's Krishna District Collector B. Lakshmikantham has been selected for the 'Best

Electoral Practices Award' to be presented by Governor E.S.L. Narasimhan during national voters'day

celebrations in the city. The district registered 2,54, 576, a record number of new voters in September and

October last year following various voter awareness campaigns under the Systematic Voter's Education and

Electoral Participation initiative. As on September 1, the number of voters was 30.51 lakh. Krishna was the

only district in the country to achieve this in short span and has now over 33.03 lakh voters enrolled with the

Election Commission of India against 46 lakh population. The district also crossed the ideal electoral

population ratio.

12)What was the theme of the 15th Pravasi Bharatiya Divas?

Ans:-Role of Indian Diaspora in building New India

Explanation:-The celebration of 15th Pravasi Bharatiya Divas was commenced in Varanasi, Uttar Pradesh.

Prime Minister of Mauritius, Mr. Pravind Jugnauth was the chief Guest of Convention. The theme of this

year's Pravasi Bharatiya Diwas (PBD) convention was "Role of Indian Diaspora in building New India". The

Celebration of 15th PBD was concluded on 23rd January 2019.

13)Which state to provide land to service sector industries?

Ans:-Bihar

Page 88: Monthly Current Affairs of SEPTEMBER - SSCE

SSCE 8981426494/8296260082 Downloaded from :- www.onlinessce.com Page 88

Explanation:-The department decided to provide land to the service sector entities as well to boost

investments in the state. The event was organized by Bihar Foundation, in association with state industries

department and Confederation of Indian Industry (CII). Over 200 crore of investment has already been made

in the food processing sector in the state. Business mechanisms and international policies of different

countries can be used to enrich the entrepreneurship structure in Bihar. Members of 10 different chapters of

the foundation from abroad and eight chapters from across the country participated in the event.

14)Which state government bagged two awards for BBBP scheme at the National Girl Child Day

celebrations in New Delhi?

Ans:-Tamil Nadu

Explanation:-The Tamil Nadu State government bagged two awards for its performance pertaining to the

Union government's Beti Bachao Beti Padhao (BBBP) scheme at the National Girl Child Day celebrations in

New Delhi. Maneka Gandhi, Minister for Women and Child Development, presented the awards. While one

award was for the State's performance, the other was for Tiruvannamalai district.

15)Which bank recommended Unconditional Cash Transfer to farmers to alleviate the agrarian

distress instead of Universal Basic Income (UBI) scheme?

Ans:-SBI

Explanation:-State Bank of India recommended Unconditional Cash Transfer to farmers to alleviate the

agrarian distress instead of Universal Basic Income (UBI) scheme in its Ecowrap report. Initiating Rythu

Bandhu Scheme on national level may not be feasible presently because land data in several states including

Bihar, Jharkhand, Gujarat and Tamil Nadu is not yet digitized. The Report suggested that Government

should consider Unconditional Cash Transfer scheme until the problems are ironed out in terms of proper

tenancy laws because it will be more equitable (on per farmer basis) with meaning full impact. As per the

report, The Government had estimated Agriculture Subsidy (Plus Support) at Rs 98,100 crore in the 2019-20

budget which includes 13,000 crore for Crop Insurance, 15,000 crore for Interest Subsidy and Rs. 70,100

crore for Fertilizer subsidy.

Daily Current Affairs 26th January, 2019

1)Who led the all-women Assam Rifles contingent, which has created history by participating for the

first time in a Republic Day parade?

Ans:-Khushboo Kanwar

Explanation:-On 70th Republic Day parade 2019, women power displayed at Rajpath, with contingents of

the Indian Navy, India Army Service Corps and a unit of Corps of Signals (transportable satellite terminal)

were all led by women officers and a woman officer exhibiting stunts on bike. In it, Major Khushboo

Kanwar led all-women Assam Rifles contingent has created history by participating for the first time in a

Republic Day parade. Capt Shikha Surabhi from the Corps of Signals became the first woman to perform

bike stunts alongside her male teammates as part of daredevils, a major attraction every Republic Day. For

the first time, a lady officer, Lt. Bhavana Kasturi led a contingent of the India Army Service Corps and Capt

Bhavna Syal, a third-generation officer in the armed forces, led the transportable satellite terminal's

contingent.

2)The Centre for Military Airworthiness and Certification cleared the use of which product in the

military aircraft, recently?

Page 89: Monthly Current Affairs of SEPTEMBER - SSCE

SSCE 8981426494/8296260082 Downloaded from :- www.onlinessce.com Page 89

Ans:-Biofuel

Explanation:-After months of ground and flight trials, the Centre for Military Airworthiness and

Certification (CEMILAC), a premier military certification agency, on January 22, 2019 cleared the use of

indigenously produced bio-fuel for use in the military aircraft.

3)Which organisation hosted a two-day Technology Conclave in New Delhi?

Ans:- NIC

Explanation:-Technical Advisory Group (TAG) of National Informatics Centre (NIC) inaugurated a two-

day Technology Conclave in New Delhi.The conclave was inaugurated by Shri Ajay Sawhney, Secretary,

Ministry of Electronics and Information Technology.The theme of the conclave was "Technologies for

NextGen Governance".It concluded on 23rd January,2019. This Technology Conclave is considered to be

the first move through a decade of digital technology and setting our path wide toward the New Digital

India. The technologies covered during this conclave were Big Data and Advanced Analytics, Artificial

Intelligence, Machine Learning, Deep Learning Methodologies, Cloud Native Scalable Applications, Micro-

services, Software Defined Infrastructure and Cyber Security.

4)The researchers of this country developed a system that can produce electricity and hydrogen fuel

from carbon dioxide.

Ans:-South Korea

Explanation:-Scientists from Ulsan National Institute of Science and Technology (UNIST) in South Korea

have developed a system that can produce electricity and hydrogen fuel from carbon dioxide. The Hybrid

Na-CO2 system can continuously produce electrical energy and hydrogen through efficient carbon dioxide

(CO2) conversion with stable operation for over 1,000 hours. The key to this technology is the easy

conversion of chemically stable CO2 molecules to other materials. Much of human CO2 emissions are

absorbed by the ocean and turned into acidity. The researchers focused on this phenomenon and came up

with the idea of melting CO2 into water to induce an electrochemical reaction.If acidity increases, the

number of protons increases, which in turn increases the power to attract electrons. If a battery system is

created based on this phenomenon, electricity can be produced by removing CO2. In particular, this system

has shown stability to the point of operating for more than 1,000 hours without damage to electrodes. The

system can be applied to remove CO2 by inducing voluntary chemical reactions.

5)BSNL and this french company join hands to offer data services through SMS.

Ans:-Be-Bound

Explanation:-Indian state-owned telecommunications BSNL and French company Be-Bound had signed an

agreement to become partners to provide data connectivity through SMS in areas where there is no Internet

connectivity or signal breaks. BSNL Chairman and Managing Director Anupam Shrivastava informed that

Be-Bound has been given patent and already developed this technology which will be embedded in mobile

apps. This app used to send command to Be-Bound server if it does not get data connectivity for SMS-based

connectivity. Be-Bound server will facilitate SMS connectivity after getting command from the app. The

app will also send SMS from user's account because for connectivity .The limit of 100 SMS per day per

subscriber which is sufficient for this service.

6)Amul Camel Milk was launched in the state of;

Ans:-Gujarat

Page 90: Monthly Current Affairs of SEPTEMBER - SSCE

SSCE 8981426494/8296260082 Downloaded from :- www.onlinessce.com Page 90

Explanation:-The Gujarat Co-operative Milk Marketing Federation Limited (GCMMF) has launched Amul

Camel Milk in selected markets of Gandhinagar, Ahmedabad and Kutch. The camel milk offers a wide

range of benefits such as: Camel milk assists healthy bacteria growth in the gut making it easy to digest.

Camel milk assists healthy bacteria growth in the gut making it easy to digest and it also improves

gastrointestinal health and systemic immunity. Rich in Vitamin B1 (Thiamin) Camel Milk can provide 70%

of the recommended daily value per serving. Camel Milk is rich in calcium which helps to build.

7)IRDAI sets a panel to identify systematically important insurers. It was headed by;

Ans:-Pravin Kutumbe

Explanation:-A panel is all set by Insurance Regulatory Development Authority of India (IRDAI) headed

by Pravin Kutumbe, to identify domestically systematically important insurers (SIIs) and an augmented

regulatory framework is logically established in this regard.

8)Which DRDO's missile is used to destroy enemy radars and communication facilities?

Ans:- Anti-radiation missile

Explanation:- India tested the new generation anti-radiation missile(NGRAM) which would destroy all the

surveillance and radar systems.The missile is launched from Balasore. The Defence-Research and

Development Organization (DRDO) has created this missile as a surface to air missile and is screened by

Sukhoi-30 MKI fighter jet. The new-generation anti-radiation missile (NGRAM) has the Strike range of 100

kilometers,this is the first indigenous air-to-ground missile developed by the DRDO, after the supersonic

BrahMos cruise missile developed jointly with Russia.

9) How many MoU's were signed at the 2nd edition of Tamil Nadu government's Global Investors

Meet (GIM) 2019?

Ans:-304

Explanation:-The second edition of Tamil Nadu government's ambitious two-day Global Investors Meet

(GIM) 2019 which aimed at showcasing the state's potential business opportunities and wooing investment

was inaugurated at Chennai Trade Centre, Chennai, Tamil Nadu. The event was attended by over 250

companies from 10 countries. The Investors Meet Concluded on 24th January 2019. The event was

inaugurated by Tamil Nadu Chief Minister Edapadi Palaniswami in the presence of Union Defence Minister

Nirmala Sitharaman, Deputy Chief Minister of Tamil Nadu O. Pannerselvam and Industries Minister of

Tamil Nadu M.C. Sampath. This year focus will be given on key 12 sectors including automobile, textiles,

aerospace, agro and food processing, pharmaceutical and chemicals, tourism, housing and construction,

Electronics and hardware, Skill Development, Heavy Engineering, MSMEs and Information and

Technology. The objective of the event is to position Tamil Nadu as the investor's choice besides

showcasing its potential business opportunities. Rs.75 crore had been allocated by the state government to

conduct the two-day event. The event had managed to exceed the target investment of Rs 2 lakh crore and

has witnessed the signing of 304 Memorandums of Understanding (MoUs) worth Rs 3.4 lakh crore with

promise of job opportunities for about 10.5 lakh people in the state.

10)Who won the women's singles 2019 Australian Open Tennis Tournament?

Ans:-Naomi Osaka

Explanation:-Naomi Osaka from Japan has won the women's singles 2019 Australian Open Tennis

Tournament. She beat Czech Petra Kvitova in a seesaw Australian Open final by 7-6 (7-2) 5-7 6-4 to win

Page 91: Monthly Current Affairs of SEPTEMBER - SSCE

SSCE 8981426494/8296260082 Downloaded from :- www.onlinessce.com Page 91

back-to-back Grand Slams and become the new World No.1. Osaka is known for her multi-ethnic

background and her shy, candid personality. On the court, she has an aggressive playing style with a

powerful serve.

11)Which abandoned airstrips will be developed into a modern airport by the Indian Government?

Ans:-Dhalbhumgarh

Explanation:-The Union Government has decided to renovate and develop around 400 abandoned airstrips

across the country to strengthen air-connectivity. The first such airstrip to be developed into a modern

airport will be Dhalbhumgarh in East Singhbhum district of Jharkhand.

12)Name the country which partnered with India to set up a Energy banking mechanism, recently.

Ans:-Nepal

Explanation:-In Pokhara, India and Nepal agreed to set up energy banking mechanism to make use of the

surplus electricity in a essential and a useful way. Nepal's demand for electricity increased during the

summer months, whereas in India demand increases during the monsoon season and slumps during winter.

This partnership will help the Nepal export its surplus electricity to neighbour India during the monsoon

season and import power during winter season.

13)With which bank, Etihad Guest tie-up to launch premium card for international travel?

Ans:-SBI Card

Explanation:-SBI Card, the country's second largest credit card issuer and Etihad Guest, the loyalty

programme of Etihad Airways have weld their hand together for a specific visa credit card to come into play

for members and Indian travellers. It has been launched in order to confront the constraints of international

travel. It is a promise to backtrack the lifestyles of affluent and urban Indians to complement their travel

experience. Besides that, it is anticipated that the Indians will boost their travelling frequency which in turn

leads to global exposure substantially

14)What is the theme of the 2019 International Customs Day?

Ans:-SMART borders for seamless Trade, Travel and Transport

Explanation:-The International Customs Day (ICD) is observed every year on January 26 to recognize the

role of custom officials and agencies in maintaining border security. It focuses on the working conditions

and challenges that customs officers face in their jobs. The 2019 theme is "SMART borders for seamless

Trade, Travel and Transport". This reflects the fact that the expectations of travellers and businesses around

the world are changing.

15) Where was the National Conference for Jan Shikshan Sansthan (JSSs) held?

Ans:-New Delhi

Explanation:- Dharmendra Pradhan, Minister of skill development implemented new reforms for Jan

Shikshan Sansthan (JSS) to enhance skill ecosystem and entrepreneurship in most of the remotest areas of

India under centre's flagship programme Skill India at the National Conference for Jan Shikshan Sansthan

(JSSs) in New Delhi. The Ministry also released the new JSS site (www.jss.gov.in) which will furnish

Page 92: Monthly Current Affairs of SEPTEMBER - SSCE

SSCE 8981426494/8296260082 Downloaded from :- www.onlinessce.com Page 92

information and facts on action ideas, databases of beneficiaries and stakeholders, details of finance and

expenditure and other ongoing development projects that are on process.

16)ndia's first indigenously developed NGRAM was tested in __________.

Ans:-Hyderabad

Explanation:-India's first indigenously developed Next Generation Anti-Radiation Missile (NGRAM), an

air-to-surface one, which can be fired from Sukhoi fighter jets, was successfully tested at the missile testing

range in Balasore, Hyderabad. The missile can be launched from different altitudes and all the parameters

were met during the testing. The missile was developed by Abdul Kalam Missile Complex comprising

DLRL, which plays the lead role, Research Centre Imarat, Advanced Systems Laboratory (ASL). The

missile system is being developed by the BEL and an unidentified private industry partner.

Daily Current Affairs 28th January, 2019

1)International holocaust remembrance day was observed on which date?

Ans:-27th January

Explanation:-International Holocaust Remembrance Day was observed all over the world on 27th January

commemorating the tragedy of the Holocaust that occurred during the Second World War. It was designated

by the United Nations General Assembly resolution 60/7 on 1st November 2005 in the 42nd Plenary session.

The largest Nazi concentration and death camp, Auschwitz-Birkenau was liberated by the Red Army on 27th

January 1945.

2)Which country will become the world's biggest retail market in 2019, as per the prediction of

research firm eMarket?

Ans:-China

Explanation:-Research firm eMarket predicted that China will become world's biggest retail market in

2019. According to the report, the sale boom is a result of China's rising incomes and thriving e-commerce.

The report says China will overtake US in 2019 and total retail sales will grow 7.5% to reach 5.636 trillion

USD in 2019, while US will grow 3.3% to 5.529 trillion. One- fifth of the country's total sales in 2018 was

from e-commerce which is a major driver of China's retail economy. As per predictions US share of the

global e-commerce market is expected to drop to 15 percent by 2022. By the end of 2019, China will have

55.8 percent of all online retail sales globally which is also expected to exceed 63% by 2022.

3)Who launched "Yuva Swabhiman Yojana" for Urban Youth?

Ans:-Kamal Nath

Explanation:-Madhya Pradesh Chief Minister Kamal Nath announced the launch of the 'Yuva Swabhiman

Yojana' for youths from the economically weaker sections in urban areas. Under this scheme, 100 days of

employment will be provided to youths from economically weaker sections in urban areas. They will also be

given skill training.

4)As per the report of CISCO, what is the position of India in General Data Protection Regulation

(GDPR) Index?

Ans:-6th

Explanation:-As per the report released by CISCO on 2019 Data Privacy Benchmark Study ranked India in

6th position in General Data Protection Regulation (GDPR) readiness index. As per the report 65% of India

organization are better prepared for GDPR making India the 6th best prepared nation to deal with the nee

data privacy regulation which all organizations operating within the European Union and European

Economic Area are mandated to comply with.

Page 93: Monthly Current Affairs of SEPTEMBER - SSCE

SSCE 8981426494/8296260082 Downloaded from :- www.onlinessce.com Page 93

5)Krishna Sobti passed away, in New Delhi, She was a __________.

Ans:-Writer

Explanation:-Demise of Hindi fiction writer and essayist,Jnanpith awardee as well, Krishna Sobti passed

away,in New Delhi at the age of 93. She was born on 18th February, 1925 in Gujarat Punjab, British India.

Her writings insinuate Punjabi, Urdu, and afterwards Rajasthani languages. Her works mainly underscores

the issues of female identity. Notable writings- Mitro Marajani, Daar se Bichuchuri, Surajmukhi Andhere

Ke. Notable Awards- Sahitya Akademi Award (1980) , Sahitya Akademi Fellowship (1996) , Jnanpith

Award (2017).

6)Which state launched the Voters Helpline Number - 1950 and mobile app for people with disabilities

on the occasion of National Voters Day?

Ans:-Mizoram

Explanation:-The Governor of Mizoram Shri Kummanam Rajasekharan launched the Voters Helpline

Number - 1950 and mobile app for people with disabilities on the occasion of National Voters Day. The

Mobile will help the Persons with Disability (PwD) to cast their votes and this move is aimed to enhance

participation of People with Disability in the in the upcoming elections. Mizoram governor appealed to all

the voters to cast their votes in the Lok Sabha elections expected to be held in May-June 2019 and ensure

peaceful conduct of the polls.

7)Name the Indian Badminton player who won Women's Singles in DAIHATSU Indonesia Masters

2019.

Ans:-Saina Nehwal

Explanation:-The 2019 Indonesia Masters, officially the DAIHATSU Indonesia Masters 2019, is a

badminton tournament that takes place at the Istora Gelora Bung Karno in Indonesia and has a total purse of

$350,000. Anders Antonsen won the men's singles title and Saina Nehwal won the women's singles title.

8)Which ministry launched a mobile app 'RDP India 2019'?

Ans:-Ministry of Defence

Explanation:-As a new initiative in sync with the Digital India campaign of the Government, Ministry of

Defence had launched a mobile app 'RDP India 2019' on the Republic Day, with the intent of making

available the highlights of the Republic Day event, not only to the spectators at Rajpath, but also to the

general public all over the world. This app contains information about the Parade on Rajpath, New Delhi,

including Order of the March, details of the tableaux presented by different States and Ministries, children

cultural performances, fly past and names of recipients of Pradhan Mantri Rashtriya Bal Puraskar 2019.

9)Which company joins with Universal Robots to Co-Create Robots for MSMEs?

Ans:-Bharat Fritz Werner

Explanation:-Bharat Fritz Werner Ltd (BFW), a Bengaluru-based machining solution provider, has

partnered with Universal Robots, a Danish robot maker and a leader in collaborative robots, to co-create

robots for MSME sector. BFW and Universal Robots plan to redefine the production processes by deploying

the robotic arms through modular, flexible and user-friendly integration solution platform - Arjun.

10)Which country has announced that it will fund South Asian Nitrogen Hub, a research project to

study Nitrogen pollution in India and South Asia?

Ans:-United Kingdom

Explanation:-United Kingdom (UK) government has made an announcement that it will support South

Asian Nitrogen Hub, a research project to study Nitrogen pollution in India and South Asia. South Asian

Nitrogen Hub will be led by United Kingdom's "Centre for Ecology and Hydrology" under the new

partnership programme. The Project will include 50 organizations from across the United Kingdom (UK)

Page 94: Monthly Current Affairs of SEPTEMBER - SSCE

SSCE 8981426494/8296260082 Downloaded from :- www.onlinessce.com Page 94

and South Asia. The Research Project which is a five year programme with estimated worth of 19.6 million

pounds (182 crore) is established by the funding from United Kingdom Research and Innovation ((UKRI).

India is the only South Asian country which has completed its Nitrogen Assessment and now India is Co-

leading the South Asian Assessment for the United Nations (UN) Environment.

11)Name of the union minister who inaugurated Atal Setu in Goa?

Ans:-Shri Nitin Gadkari

Explanation:-Union Minister Nitin Gadkari inaugurated four-lane, 5.1-kilometer long "Atal Setu", the third

Cable bridge across river Mandovi in Panaji, Goa. This is the third bridge across Mandovi River. This bridge

would overcome the bottlenecks and curb traffic jams at Panaji Circle and Panaji entrance.

12)Which state government has floated a 'Pravasi Dividend Pension Scheme' to provide regular

pension to Non-Resident Keralites on a one-time payment of Rs 5 lakh?

Ans:-Kerala

Explanation:-The Kerala government has floated a 'Pravasi Dividend Pension Scheme' to provide regular

pension to Non-Resident Keralites on a one-time payment of Rs 5 lakh. Governor P. Sathasivam announced

the scheme during his address to the state Assembly. There are 2.1 million migrants from Kerala across the

world. A non-resident Keralite can invest an amount ranging from Rs 5 lakh to Rs 50 lakh, as a whole or as

6 installments paid within 3 years. Each beneficiary will be entitled to receive pensions ranging from Rs

5,000-50,000 every month, 3 years after the total amount is deposited, at a dividend rate of 12%.

13)Where the Operation Olympics Commenced?

Ans:-Kerala

Explanation:-An expert programme 'Operation Olympics' managed by Kerala's Sports Department and

State Sports Council was commenced to train 123 athletes in 11 disciplines for 2020 Tokyo Olympics. The

programme has the objective to provide international standard training to the athletes with the help of

eminent trainers from both India and abroad. Under the Programme, Expert training will be provided in

athletics, boxing, cycling, swimming, shooting, wrestling, badminton, fencing, rowing and archery. Training

in wrestling, cycling, swimming, and boxing has been started at the various stadium in Thiruvananthapuram

district whereas training for other streams will be at Kollam, Ernakulam, Alappuzha and Kannur districts.

14)The Oxford Dictionaries announced __________ as the Hindi Word of the Year.

Ans:-Nari Shakti

Explanation:- The Oxford Dictionaries announced 'Nari Shakti' as the Hindi Word of the Year. The Hindi

Word of the Year chosen every year, reflects the conversations, ethos of a particular year. The word this

year, Nari Shakti, has been derived from Sanskrit and when broken down, the word 'nari' means women and

'shakti' means power.

15)Who conferred with the Assam Republic Journalism Award?

Ans:-D.N. Chakraborty

Explanation:-Assam government entitled the Republic Day Journalism Award 2019 to veteran journalist

and former editor Dhirendra Nath Chakraborty. The award comprises Rupees one lakh in cash, a citation,

the Assamese 'xoraai' (the bell metal memento presented as an honor), 'jaapi' (headgear) and an angavastra.

16)Which country declared sugarcane juice as the "national drink"?

Ans:-Pakistan

Explanation:-The Pakistan government declared sugarcane juice as the "national drink" of the country.

They came at this decision after asking People's opinion in twitter to choose one among orange, carrot and

Page 95: Monthly Current Affairs of SEPTEMBER - SSCE

SSCE 8981426494/8296260082 Downloaded from :- www.onlinessce.com Page 95

sugarcane. According to the poll, 7,616 people or 81% cast their votes favouring sugarcane juice, 15% voted

for orange juice while 4% chose carrot.

17)Who conferred with the Assam Republic Journalism Award?

Ans:- D.N. Chakraborty

Explanation:-Assam government entitled the Republic Day Journalism Award 2019 to veteran journalist

and former editor Dhirendra Nath Chakraborty. The award comprises Rupees one lakh in cash, a citation,

the Assamese 'xoraai' (the bell metal memento presented as an honor), 'jaapi' (headgear) and an angavastra.

18)Which team won the maiden women's Big Bash title?

Ans:-Brisbane Heat

Explanation:-Australian T20 cricket team "Brisbane Heat" won their maiden Women's Big Bash League

(WBBL) title by defeating Sydney Sixers in the final by three wickets at Drummoyne Oval in Sydney,

Australia. Sydney Sixers opted to bat first but couldn't get off to a flying start that they unusually followed

throughout the season and could score only 131 runs which was chased by Brisbane Heat with four ball

remaining. Beth Mooney from 'Brisbane Heat' was awarded as the Player of the match for her knock of 65

runs from just 46 balls.

19)India's fastest indigenous train "Train 18" was renamed as;

Ans:-Vande Bharat Express

Explanation:-India's fastest indigenous train which will run from Delhi to Varanasi has been named as

Vande Bharat Express. Its previous name was Train 18. The new name has been given after taking

suggestions from the general public. It is a train built completely in India by Indian engineers in a span of 18

months and it is an example that it is possible to make world-class trains under Make in India. The 16-coach

train has been built by the Integral Coach Factory, Chennai and cover the distance in 8 hours. It is the first

locomotive-less train in the country.

Daily Current Affairs 29th January, 2019

1)Who became the first woman to perform bike stunts as part of daredevils, during Republic Day

celebrations of India?

Ans:-Capt Shikha Surabhi

Explanation:-India Celebrated its 70th Republic Day. Capt Shikha Surabhi from the Corps of Signals

became the first woman to perform bike stunts, alongside her male teammates as part of daredevils. For First

Time,IAF Flies Plane Using Blend Of Jet Fuel And Biofuel During Republic Day Parade.

2)Who became the first woman officer to lead an all-men Army contingent in the Republic Day

parade?

Ans:-Lieutenant Bhavana Kasturi

Explanation:-On 26 January 2019, Lieutenant Bhavana Kasturi became the first woman officer to lead an

all-men Army contingent in the Republic Day parade. She led the Army Services Corps contingent. An all-

women contingent of the Assam Rifles was led by Major Khushboo Kanwar. Captain Shikha Surbhi of the

Army Signals Corps performed a standing salute on a moving motorcycle.

3)As per the latest report of World Steel Association, which is the world's second-largest steel

producing country?

Page 96: Monthly Current Affairs of SEPTEMBER - SSCE

SSCE 8981426494/8296260082 Downloaded from :- www.onlinessce.com Page 96

Ans:-India

Explanation:- India has replaced Japan as the world's second-largest steel producing country, only behind

China, which is the largest producer of crude steel accounting for more than 51 per cent of production, as

per the latest report by World Steel Association. The other nations in the list of top 10 steel producing

countries include the United States, which is at the 4th position as the country produced 86.7 MT of crude

steel in 2018, followed by South Korea at the 5th place with 72.5 MT of crude steel, Russia at 6th with 71.7

MT, Germany at 7th with 42.4 MT, Turkey at 8th with 37.3 MT, Brazil at 9th with 34.7 MT and Iran at the

10th position with 25 MT crude steel in 2018.

4)Who is the author of the novel "Nireeshwaran" which won the best novel award in the Kerala

Sahitya Akademi Award 2017, announced on 23rd January 2019?

Ans:-VJ James

Explanation:-Kerala Sahitya Akademi Award 2017 was announced, Malayalam Novelist VJ James's, Novel

Nireeshwaran won the best novel award. V J James was born in Vazhapally, Changanassery in Kottayam

district of Kerala. His debut novel was Purappadinte Pusthakam. Other Accolades he received:DC Silver

Jubilee Award, Malayattor Prize in 1999, Rotary Literary Award for Purappadinte Pusthakam, Thoppil Ravi

Award, Kerala Bhasha Institute Basheer Award (2015) for Nireeshwaran.

5)Which railway zone has bagged No.1 position for cleanliness in India?

Ans:-Southern Railway

Explanation:-The Southern Railway has bagged the number one position for cleanliness in the country.

This achievement was under the category of other than Premium Category trains in the recent Trains

Cleanliness Survey conducted by IRCTC on all India level. During the year 2018, Southern Railway's gross

earnings also stood at more than Rs six thousand crores rupees which is almost 10% higher than the last year

earnings. To cater to the growing demands of passengers, Southern Railway has also introduced 54 trains

over past four years.

6)Name the mountaineer who was conferred the Padma Bhushan 2019 award?

Ans:-Bachendri Pal

Explanation:-64-year-old legendary mountaineer, Bachendri Pal, became the first Indian woman to reach

the summit of Mount Everest in 1984, was awarded the Padma Bhushan for distinguished service of a high

order by the Modi government.

7)Who has won the 2018 DSC Prize for South Asian Literature?

Ans:-Jayant Kaikini

Explanation:-Jayant Kaikini, renowned Kannada poet short-story writer, columnist and playwright and

lyricist, has won the prestigious 2018 DSC Prize for South Asian Literature for 'No Presents Please: Mumbai

Stories' at the Tata Steel Kolkata Literary Meet. The novel is a collection of short stories set in Mumbai

dating back to the 1980s. It offers a fine-grained, ground-up view of arguably India's greatest metropolis.

The book was originally written in the southern Indian language of Kannada and translated into English by

the award-winning translator Tejaswini Niranjana. The $25,000 (£19,100) prize rewards the best writing

about south Asian culture from writers of any ethnicity and from all over the world, will be split equally

between author and translator. It will be split equally between author and translator.

Page 97: Monthly Current Affairs of SEPTEMBER - SSCE

SSCE 8981426494/8296260082 Downloaded from :- www.onlinessce.com Page 97

8)Which country will participate in the International Student Assessment (PISA) in 2021?

Ans:-India

Explanation:-Programme for International Student Assessment (PISA) is a triennial international survey

which aims to evaluate education systems worldwide by testing the skills and knowledge of 15-year-old

students. India would be participating in the PISA assessment in 2012. India participated in PISA 2009.

9)India signed the phytosanitary protocol with which country to facilitate the export of Indian

tobacco?

Ans:-China

Explanation:-A protocol was signed between India's Commerce Secretary Anup Wadhawan and Zhang

Jiwen, Vice Minister of General Administration of China Customs (GACC) accountable for examining

market access and quarantine issues for India's agriculture and allied products. Mr. Wadhawan was on a

two-day visit to Beijing, China where his efforts with Chinese officials facilitated for the tobacco export to

China. The revival of the phyto-sanitary protocol with China will turn economically better for the farmers

and also ameliorate Indian tobacco exports as China is the largest consumer and producer of tobacco with

over 350 million smokers (world's highest).

10)Who has been appointed as Regional Director for WHO SOUTH EAST Asia?

Ans:- Dr Poonam Khetrapal Singh

Explanation:-Dr. Poonam Khetrapal Singh appointed Regional Director for World Health Organization

South-East Asia, for a second five-year term. Her second term of office will begin on 1 February 2019. She

is the first woman to become Regional Director of WHO South-East Asia Region Previous year,Thailand

became the first in Asia-Pacific to eliminate mother-to-child transmission of HIV and syphilis.

11)Who had launched the 2020 Presidential bid?

Ans:-Kamala Harris

Explanation:- Indian-origin US Senator Kamala Harris launched her 2020 presidential bid. In a scathing

criticism of President Donald Trump's policies. The US is at an inflection point in the history due to the

attack on democracy like never before. Addressing a cheering crowd in downtown Oakland, she said free

press has been bullied and attacked, and democratic institutions undermined under the current leadership.

Harris also slammed Trump's policies at the border, and his decision to shut down the government. She

denounced his plan to build a wall along the US-Mexico border as a medieval vanity project. A 54-year-old

Harris, who was elected to the Senate in 2016, announced her run for the presidency last week. She has been

voted on top of the list of Democratic leaders aspiring to defeat Trump in the November 2020 election.

12)Who won the New South Wales men's Amateur Golf Championship in Australia?

Ans:- Kartik Sharma

Explanation:-Kartik Sharma from India has won the New South Wales men's Amateur Golf Championship

in Australia. Kartik notched up a magnificent win over Australia's Nathan Barberie in the scheduled 36-hole

final to emerge as champion. Kartik has been playing for India since the age of 10 as part of the National

Squad. He had been selected to represent Indian Golf Union for three events in Australia.

Page 98: Monthly Current Affairs of SEPTEMBER - SSCE

SSCE 8981426494/8296260082 Downloaded from :- www.onlinessce.com Page 98

13)Union Minister for Minority Affairs Mukhtar Abbas Naqvi stated that the GST on Haj reduced

from 18% to __________.

Ans:-5%

Explanation:- Union Minister for Minority Affairs Mukhtar Abbas Naqvi stated that GST on Haj has been

reduced from 18% to 5% and this will significantly lower the airfare thus saving Haj pilgrims' 113 crore

rupees this year. Mr. Naqvi also stated that for the first time since the independence 2,340 women from

India will go on 2019 Haj without Mehram.

14)Who inaugurated an Integrated Refinery Expansion Project Complex (IREP) in Kochi, Kerala?

Ans:- Narendra Modi

Explanation:- Prime Minister Narendra Modi inaugurated the Bharat Petroleum Corporation Limited's

Integrated Refinery Expansion Project Complex (IREP) in Kochi on 27 January 2019. IREP will be a

modern expansion complex and will be equipped for production of cleaner fuels for India. It will double the

production of LPG & diesel and commence production of feedstock for petrochemical projects.

15)Data Protection Day was observed on;

Ans:-28 January

Explanation:-The Council of Europe had launched a Data Protection Day to be celebrated each year on 28

January, the date on which the Council of Europe's data protection convention, known as "Convention 108",

was opened to the signature. Data Protection Day is now celebrated globally and is called Data Privacy Day

outside Europe. The data protection convention, which is the only international treaty in this field, is being

updated to ensure that its data protection principles are still in line with today's needs.

16)Uttarakhand's first tulip garden will be opened in which district?

Ans:-Pithoragarh

Explanation:- Uttarakhand's first tulip garden will come up over 50 hectares of forest land in Pithoragarh

district at a cost of Rs 50 crore. The state government has obtained the Centre's nod for development of the

garden. To be developed by the ONGC, under its corporate social responsibility programme, the proposed

tulip garden will be the country's second after the one located in Srinagar (J&K). It has been selected under

the State Government's '13districts, 13 new destinations' scheme, to attract more tourists to the district.

17)Indian politician George Mathew Fernandes was passed away on January 29, 2019. He was a

former minister for which of the following department?

Ans:-Defence

Explanation:-Former Defence Minister George Fernandes passed away on January 29, 2019 following a

prolonged illness. He was 88. Fernandes was suffering from Alzheimer's disease; and recently, he contracted

swine flu. Fernandes held the portfolio of the Union Defence Minister from 2001 to 2004 under the Atal

Bihari Vajpayee-led NDA government. He was born in Mangaluru in British India on June 3, 1930.

18)Who becomes 1st Top-Ranked Windies Test All-Rounder in the International Cricket Council Test

Player?

Page 99: Monthly Current Affairs of SEPTEMBER - SSCE

SSCE 8981426494/8296260082 Downloaded from :- www.onlinessce.com Page 99

Ans:-Jason Holder

Explanation:-Jason Holder becomes the first player from the country in 45 years to top the list of all-

rounders in the International Cricket Council Test Player Rankings after slamming an unbeaten double-

century that helped script a memorable 381-run win over England in the first Test in Bridgetown. West

Indies legend Gary Sobers was at the top last in March 1974 and Holder now emulated him following his

202 not out off 229 balls in the second innings in Bridgetown which is the third highest score by a number

eight batsman in Test history after the Pakistan pair of Wasim Akram (257 not out) and Imtiaz Ahmed

(209).

19)Ministry of Textiles holds a National Conclave on Technical Textiles in __________.

Ans:-Mumbai

Explanation:-Ministry of Textiles will hold National Conclave on Technical Textiles on 29 January 2019 in

Mumbai. The release of HSN codes for technical textiles will be one of the major highlights of the conclave.

Technical textiles are textile material and products manufactured primarily for technical performance and

functional properties rather than aesthetic and decorative characteristics.

20)Name the app developed by Indian Institute of Technology (IIT), Ropar to help the Visually

Impaired to recognise currency notes?.

Ans:-Roshni

Explanation:-The Indian Institute of Technology (IIT), Ropar, has developed an app Roshni that would

help the Visually Impaired to recognise currency notes. The app can recognise both old and new notes and

gives audio information as to which note you are holding in your hands.This app was launched in

Chandigarh. Roshni is the first Android app,which recognise INR currency notes. The user have to bring the

currency note in front of phone camera and the App would give a audio intimation about the currency note

denomination. This app is now formed enough to recognise Indian money of any denominations, whereas

we can expect its upgrade version in further even to identify the Indian coins as well as foreign currency

notes.. This app is bug free and is available in beta version of the play store.

Daily Current Affairs 30th January, 2019

1)Where did Chief Election Commissioner of India, Sunil Arora and Election Commissioner Ashok

Lavasa inaugurate an International Conference on "Making our Elections Inclusive and Accessible",

recently?

Ans:-New Delhi

Explanation:-The Chief Election Commissioner of India, Sunil Arora and Election Commissioner Shri

Ashok Lavasa inaugurated an International Conference on 'Making our Elections Inclusive and Accessible

"on the occasion of 9th National Voters Day held 'in New Delhi. This conference held with a motive to

enhance transparency in the upcoming election. National Voters Day is observed on 25th January every

year. Theme of 2019 voters day is "My Vote Matters".

2)Recently, National Salt Satyagraha Memorial was inagurated at __________, Gujarat.

Ans:-Dandi

Page 100: Monthly Current Affairs of SEPTEMBER - SSCE

SSCE 8981426494/8296260082 Downloaded from :- www.onlinessce.com Page 100

Explanation:-In commemoration of the 150th Birth Anniversary of Mahatma Gandhi, Prime Minister of

India Narendra Modi dedicated grand and iconic 'National Salt Satyagraha Memorial' constructed by the

Central Public Works Department at Dandi, Gujarat to the Nation. This prestigious work entrusted to CPWD

by the Ministry of Culture, Govt. of India has been completed by the dedicated team of CPWD in 20 months

time well within the sanctioned amount of Rs. 72.23 crores.

3)How many agreement are signed between Syria and Iran recently?

Ans:-11

Explanation:-Syria and Iran signed 11 agreements and MOU including a "long-term strategic economic

cooperation". The agreements covered a range of fields including economy, culture, education,

infrastructure, investment and housing. The deal is signed by Syrian PM Imad Khamais and Iranian Vice-

President Eshaq Jahangiri.

4)Which bank will inject Rs 37,500 crore through OMO in February?

Ans:-RBI

Explanation:-Reserve Bank of India it will inject Rs 37,500 crore into the system through the purchase of

government securities in February to increase liquidity. Reserve Bank of India has been monitoring the

evolving liquidity conditions and durable liquidity requirements of the system. The government to conduct

the purchase of government securities under Open Market Operations (OMOs) for an aggregate amount of

Rs 375 billion in the month of February 2019 through three auctions of Rs 125 billion each during the 2nd,

3rd and 4th week.

5)Which cocuntry received the Award of Excellence for 'Best in Show' at the New York Times Travel

Show 2019?

Ans:- India

Explanation:-India received the Award of Excellence for 'Best in Show' at the New York Times Travel

Show 2019, the largest travel show in North America. India was honoured during the Closing Bell

Ceremony at the New York Stock Exchange. The Ministry of Tourism, Government of India participated as

the 'Presenting Partner' in the New York Times Travel Show (NYTTS 2019) organised at Jacob K Javits

Centre, New York with a view to scale up its tourism promotional efforts in the US, enhance India's

visibility and to increase its share of US Outbound travel market.

6)The first Hindu woman who have been appointed as a civil judge in Pakistan.

Ans:-Suman Kumari

Explanation:-Suman Kumari has become the first Hindu woman to have been appointed as a civil judge in

Pakistan after passing an examination for induction of judicial officers. Kumari, who hails from Qambar-

Shahdadkot, will serve in her native district. The first judge from the Hindu community was Justice Rana

Bhagwandas, who served as the acting chief justice for brief periods between 2005 and 2007.

7)Where was the country's first Geographical Indication (GI) store launched?

Ans:-Goa

Page 101: Monthly Current Affairs of SEPTEMBER - SSCE

SSCE 8981426494/8296260082 Downloaded from :- www.onlinessce.com Page 101

Explanation:- The Civil Aviation ministry launched the country's first Geographical Indication (GI) store at

Dabolim International Airport in Goa. Union Minister for Civil Aviation Suresh Prabhu stated that the GI

stores will promote local products made by the artisans and handicrafts at the airport. The store is set up by

Airports Authority of India in partnership with The Cashew Export Promotional Council of India.

8)Dilip Sadarangani was appointed as the Chairman of which of the following bank?

Ans:-Federal Bank

Explanation:-South India based private sector lender Federal Bank appointed Dilip Sadarangani as the part-

time Chairman. He has been on bank's board since 2013. The appointment of Mr.Dilip Sadarangani who has

been guiding person in technology in multiple banks, as Chairman of the board of Federal Bank comes at a

time when the bank makes confident strides in gaining market share in India. Mr. Sadarangani played key

roles in setting up of IT infrastructure in different banks in India, Australia and Kuwait. He was a core

member of the team which automated the first 50 branches of State Bank of India.

9)The University of Madras has received Rs.50 crore from which of the following ministry?

Ans:-Ministry of Human Resource Development

Explanation:-The University of Madras has received ₹50 crore from the Ministry of Human Resource

Development for upgrading its research capabilities. A total of 10 universities belonging to category I will

get Rs. 100 crore and 17 universities falling into category II will receive of ₹50 crore. University of Madras

is one of the oldest educational institutions in India established in the year 1857.

10)Who inagurated an international indoor exhibition center in Hapania?

Ans:-Biplab Kumar Deb

Explanation:-Tripura Chief Minister Biplab Kumar Deb on Monday inaugurated an international indoor

exhibition center here, the largest facility of its kind in the Northeastern region. Built at a cost of Rs 66.52

crore at Hapania with an area of 10,800 sq metre The indoor centre has the capacity to house 170 stalls and

accommodate 10,000 visitors.

11)Who has become the first Indian transgender to be honoured with Padma Shri?

Ans:-Narthaki Nataraj

Explanation:-Narthaki Nataraj, a renowned dancer from Tamil Nadu, has become the first Indian

transgender to be honoured with Padma Shri, the third highest civilian award of India. She is a

Bharatanatyam exponent and specialises in the Thanjavur-based Nayaki Bhava tradition. Born in the temple

town of Madurai, Nataraj learned the art under Tanjore Shri K P Kittappa Pillai and gave her on advances to

the Tanjore Quartet of Bharatanatyam (Nayaki Bhava tradition) through a dance school she established.

12)As per the Global Corruption Index 2018, which is the top most country in the list of 180 countries

in the world?

Ans:-Denmark

Explanation:-Transparency International has released the Global Corruption Index 2018. India rose by 3

points to 78 in the list of 180 countries in the world with a score of 41. China lagged far behind at 87 and

Page 102: Monthly Current Affairs of SEPTEMBER - SSCE

SSCE 8981426494/8296260082 Downloaded from :- www.onlinessce.com Page 102

Pakistan ranked 117. The top countries on the list are Denmark and New Zealand, with scores of 88 and 87.

Somalia, Syria and South Sudan remained at the bottom, with scores of 10, 13 and 13.

13)The Central government allocated Rs.900 crore to this state towards drought relief package for

2018-19 Kharif season.

Ans:-Andhra Pradesh

Explanation:- The Central government allocated Rs.900 crore to the State towards drought relief package to

Andhra Pradesh for the 2018-19 Kharif season. The fund allocation is a part of the relief package worth

Rs.6,680 crore earmarked for Andhra Pradesh, Maharashtra, Gujarat, and Karnataka. Andhra Pradesh got

Rs.900 crore. Maharashtra has got an allocation of Rs.4700 crore, while Karnataka will get Rs.950 crore and

Gujarat Rs.130 crore. Ministry of Agriculture & Farmers' Welfare: Headquarters: New Delhi Minister:

Radha Mohan Singh Its formulation and administration of the rules and regulations and laws related to

agriculture in India.

14)This states has been declared best state for active participation in Swasth Bharat Yatra.

Ans:-Tamil Nadu

Explanation:-Tamil Nadu was declared the 'Best State Overall' for its active participation in the national

campaign 'Swasth Bharat Yatra'. The campaign was a 100-day consumer outreach programme of the Food

Safety and Standards Authority of India (FSSAI), which was launched on 16th October 2018 to amplify its

Eat Right India initiative. Gujarat, Madhya Pradesh, Uttar Pradesh and Maharashtra were adjudged the Best

States with population above 3 crore. Punjab, Goa and Delhi were recognised as the Best States having

population less than 3 crore. Meghalaya, Jammu and Kashmir and Uttrakhand were given special awards for

their participation despite their hilly and difficult terrain. The Yatra saw the involvement of 10 lakh

participants and reached to around 2.5 crore people across various States and UTs. It also led to the creation

of 21,000 local 'Eat Right Champions' who would sustain this movement in the future.

15)How many commitees has been started by the Employees' Provident Fund Organisation?

Ans:-3

Explanation:-Employees Provident Fund Organisation has notified three new committees on Finance,

Investment and Audit Committee (FIAC), the Pension and EDLI Implementation Committee (PEIC) and the

Exempted Establishments' Committee (EEC). All the 3 committees will continue to have central government

representatives as well as 2 representatives each of the employers and the employees and domain experts.

While the FIAC will oversee investments made by portfolio managers and ensure timely investment of PF

kitty for optimum returns, PEIC will review the function of the Employees' Pension Scheme, 1995 and the

Employees Deposit Linked Insurance Scheme, 1976. The EEC will oversee the working of exempted

establishments and move suggestions for CBT for improvement in the working of these exempted

establishments.

16)First Astro Turf Football Pitch inaugurated in the state of;

Ans:-Nagaland

Explanation:-Nagaland CM Neiphiu Rio inaugurates state's first Astro Turf Football Pitch at Indira Gandhi

Stadium in Kohima on the occasion of 100th Birth Anniversary of the legendary footballer, Dr. T. Ao The

project was conceived in the year 2013-14 with total cost of 498 lakh rupees. The Astro turf has been

installed following specifications of FIFA. The size of the play field is 105?66 meters.

Page 103: Monthly Current Affairs of SEPTEMBER - SSCE

SSCE 8981426494/8296260082 Downloaded from :- www.onlinessce.com Page 103

17)This international Airport has donated Rs.10 crore to the Chief Minister's Distress Relief Fund

(CMDRF).

Ans:- Cochin International Airport

Explanation:-Cochin International Airport Limited (CIAL) has donated Rs.10 crore to the Chief Minister's

Distress Relief Fund (CMDRF). CMDRF is an emergency assistance release mechanism granting immediate

relief to families and individuals distressed by calamity, loss of life due to accidents and chronic diseases.

18)Name the Indian player has been banned from bowling in international cricket.

Ans:-Ambati Rayudu

Explanation:-India's Ambati Rayudu has been banned from bowling in international cricket because of a

suspect action. The suspension was done in "accordance with clause 4.2 of the ICC regulations" which deals

with the legality of bowling actions. He is a right-handed middle-order batsman, who occasionally keeps the

wicket, and a right-arm off break bowler.

19)The 'Atal Setu' bridge named after former prime minister of India late Atal Bihari Bajpai, which

is the third Cable bridge across river __________ in Panaji, Goa.

Ans:-Mandovi

Explanation:-The Third bridge was inaugurated by Union Minister Nitin Gadkari in the presence of Chief

Minister Manohar Parrikar .The bridge has been named as 'Atal Setu' after former prime minister of India

late Atal Bihari Bajpai. It is four-lane, 5.1-kilometer long and the third Cable Bridge across river Mandovi in

Panaji, Goa. The bridge weighs 2.5 lakh tonnes which is equivalent to the weight of aircraft 570 Boeing. The

bridge is constructed by the GIDC (Goa Infrastructure Development Corporation) and engineering and

construction major Larsen and Toubro. The bridge is built at a cost of Rs 850 crore. The two bridges already

present in Goa are Mandovi Bridge and Zurai Bridge. The bridge will be open for four wheeler vehicles

from Jan 29 but two and three wheelers will not be allowed on the bridge.

20)Ministry of Textiles launched a program to strengthen the handlooms and textiles sector of India.

It is;

Ans:- Artisan Speak

Explanation:-Ministry of Textiles and Government of India launched Artisan Speak at Elephanta Caves

(UNESCO heritage site) to strengthen the handlooms and textiles sector of India. The event has been held to

showcase the GI Textiles of India, through a confluence of fashion, music, and dance. Many textiles

companies signed agreements with the Office of Development Commissioner (Handlooms), Ministry of

Textiles on the occasion. Under the agreements, the Weavers Service Centres (WSCs) under the Office of

DC (Handlooms) will enable textile firms to directly source their requirements from handloom clusters, as

per defined quality, cost and time constraints. This will also result in better price realization and improved

market understanding for weavers.

Daily Current Affairs 31stJanuary, 2019

1)Who was appointed as India's Ambassador to the Maldives?

Ans:-Sunjay Sudhir

Page 104: Monthly Current Affairs of SEPTEMBER - SSCE

SSCE 8981426494/8296260082 Downloaded from :- www.onlinessce.com Page 104

Explanation:-Senior diplomat Sunjay Sudhir was appointed as India's Ambassador to the Maldives.He

presently works as a Joint Secretary in the Ministry of Petroleum and Natural Gas,which he will be resigning

shortly. Also,government appointed Vinay Singh Chauhan as the Ambassador of India to Burkina Faso who

is presently Consulate General of India.

2)The second edition of "Pariksha Pe Charcha 2.0" held in which city?

Ans:-New Delhi

Explanation:-Prime Minister Shri Narendra Modi discussed ways to handle the exam pressure in the second

edition of "Pariksha Pe Charcha 2.0" at Talkatora Stadium in New Delhi.The Programme was organized by

Ministry of Human Resource Development(MHRD). On this program, Prime Minister interacted with

students, teachers and parents and shared his personal experiences with students and suggested means to

handle exam stress. 2200 students, parents,teachers and college students from 24 states and Union

Territories took part in the event. Prime Minister, Narendra Modi answered the queries of Students, teachers

and parents and motivated the students with his personal experiences. Rini Bhattacharjee(differently abled

student of class XI) from West Bengal enthralled the audience with her performance on the keyboard with

the help of her feet only.

3)Which state government has recently launched a special scheme to curb infant deaths?

Ans:-Maharashtra

Explanation:-The Maharashtra government has launched a special scheme to curb infant deaths. The

scheme is applicable only for the first child and will benefit around four lakh women across the state,

Women. Each kit contains a blanket, a small mattress, a towel, a thermometer, baby oil, shampoo, toys, nail-

cutter, gloves, socks, among other things. The cost of the kit is around Rs 2,000. For this cause, Child

Development Minister Pankaja Munde distributed baby-care kits to the children born in primary health

centres and government hospitals.

4)2019 International Stock Taking Conference on Tiger Conservation was held in __________.

Ans:- New Delhi

Explanation:-The 3rd International Stock Taking Conference on Tiger Conservation was held in New Delhi

on January 28, 2019. The 2-day conference was hosted by the National Tiger Conservation Authority

(NTCA) in close collaboration with the Global Tiger Forum (GTA) which is an International,

Intergovernmental Organization for conserving tigers in the world. In it, the 13 tiger range countries

discussed on the status of the Global Tiger Recovery Program (GTRP) besides deliberations on combating

wildlife trafficking. This is the second stock taking conference held in India after 2012. The major outcome

of the Conference is expected to be course correction in strategies envisaged in Global and National Tiger

Recovery Programs (GTRP/NTRP) which shall take tiger conservation forward to the intended goal as

outlined in St. Petersburg, Russia.

5)ISRO Unveils Human Space Flight Centre in __________.

Ans:-Bengaluru

Explanation:-ISRO Unveils Human Space Flight Centre in Bengaluru. It aims to send humans to space by

December 2021 through its Gaganyaan mission. The Centre is dedicated to developing critical technologies

for human space missions.

Page 105: Monthly Current Affairs of SEPTEMBER - SSCE

SSCE 8981426494/8296260082 Downloaded from :- www.onlinessce.com Page 105

6)Who became the first Indian to compete in the Ironman Triathlon Malaysia?

Ans:-Mukul Nagpaul

Explanation:-Mukul Nagpaul became the first Indian from Ludhiana in Punjab to compete in the Ironman

Triathlon Malaysia. Nagpaul, who has in the past ran the World's Highest Barefoot Marathon to raise funds

for the rehabilitation of stray dogs affected by the floods in Kerala. And the triathlon includes swimming a

distance of 3.8 km, cycling a distance of 180 km and running a distance of 42.2 km in one stretch.

7)On which date, India's Martyrs Day was celebrated?

Ans:-January 30

Explanation:-January 30 is celebrated as Martyrs Day to honour the brave martyrs of India who have

sacrificed their lives for the country. On the occasion of Martyrs Day, the President, the Vice President, the

Prime Minister, the Defence Minister, and the three Service Chiefs gather at the Gandhiji's samadhi at Raj

Ghat memorial and lay wreaths. Bugles are blown sounding the Last Post. A two-minute silence in memory

of Indian martyrs is observed across the country at 11 AM and participants hold all-religion prayers and sing

tributes in the honour of Martyrs.

8)This city acquired a ODF++ status by the Swachh Bharat Mission.

Ans:-Hyderabad

Explanation:-Hyderabad Metropolitan Water Supply and Sewerage Board (HMWSSB) has been conferred

ODF++ (Open Defecation Free Plus Plus) status by the Swachh Bharat Mission in the run-up to the Swachh

Survekshan rankings. Swachh Bharat Mission: Launched on: October 2014 Launched by: Prime Minister

Narendra Modi Aim: To eliminate open defecation through the construction of household-owned and

community-owned toilets During the period of 2014 to 2019, it aimed to clean up the streets, roads, and

infrastructure of India's cities, towns, and rural areas.

9)National Agricultural Higher Education Project (NAHEP) was launched by;

Ans:-ICAR

Explanation:-Indian Council of Agricultural Research (ICAR) has launched the National Agricultural

Higher Education Project (NAHEP) to attract talent and strengthen higher agricultural education in the

country. It is inaugurated at the fourth Convention of 2-day "Agrivision-2019" organized by Vidyarthi

Kalyan Nyas in Pusa, New Delhi. This project (Rs 1100 crore) is to be funded by the World Bank and the

Indian Government on a 50:50 basis. To enhance the participation of students in agricultural business,

READY (Rural Entrepreneurship Awareness Development Yojana) is also provided to the students.

10)Which state cabinet has approved construction of a four-lane Ganga Expressway?

Ans:-Uttar Pradesh

Explanation:-Uttar Pradesh state cabinet has approved construction of a four-lane Ganga Expressway

connecting Prayagraj to Western Utter Pradesh which would cost about 36,000 crore rupees. Chief Minister

Yogi Adityanath stated that this expressway will provide better connectivity to Prayagraj. Ganga

Expressway will be 600 kilometres long and can be expanded to six-lane in future. Chief Minister also stated

that after completion it would be longest in the world.

Page 106: Monthly Current Affairs of SEPTEMBER - SSCE

SSCE 8981426494/8296260082 Downloaded from :- www.onlinessce.com Page 106

11)Southern Micro, Small and Medium Enterprises (MSME) Association organize the South India

MSME Summit-2019 in __________.

Ans:-Bengaluru

Explanation:-Southern Micro, Small and Medium Enterprises (MSME) Association organized the South

India MSME Summit-2019 at Bengaluru. Basavaraj S Javali chaired the event. The summit is held with a

motive to find solution for their major problems like absence of good infrastructure, technology issues,

marketing, procurement issues, regulatory issues, red tape, harassment and victimization.

12)Which will be the host country of G20 annual summit in 2022?

Ans:-India

Explanation:-India is going to host the G20 annual summit is 2022 when it celebrates its 75th year of

independence and in order exploit it to the full extent, capacity augmentation of available resources and

efficient addressal of organizational challenges must be done on time to fill the infrastructure, management

and intellectual gap. Indian Prime Minister, Narendra Modi requested Italy to host the 2021 G20 summit

instead of 2022 summit which was accepted by Italy.

13)30th January 2019, is celebrated as World Leprosy Eradication Day. The theme for 2019 is;

Ans:- Ending discrimination, stigma, and prejudice

Explanation:-30th January 2019, is celebrated as World Leprosy Eradication Day which stresses the need

of eradicating the disease. It also brings to light the discrimination and stigma that people suffer from society

every day. This year's theme for World Leprosy Day is 'ending discrimination, stigma, and prejudice'.

Leprosy, also known as Hansen's disease is a highly contagious infection caused by the bacterium

Mycobacterium leprae.

14)Scientists Develop Artificial Pancreas Smartphone app for whom?

Ans:-Diabetics

Explanation:-Scientists have developed an artificial pancreas smartphone app that will help in regulating

blood sugar levels in diabetes patients. This app is capable of interfacing wirelessly with glucose monitors,

insulin pump devices, and decision-making algorithms. A clinical trial showed that the artificial pancreas

system (iAPS) app is safe. The app is effective in regulating glucose levels under challenging conditions and

is suitable for use in unconstrained environments.

15)Which Airport retained as the world's busiest airport for international customer numbers?

Ans:-Dubai International Airport

Explanation:-Dubai International Airport (DXB) retained its position as the world's busiest airport for

international customer numbers for the fifth consecutive year with annual traffic for 2018 with Indian

travellers emerging as the largest source of traffic for the airport which registered a record footfall of over 89

million. It overtook the crown of the world's busiest hub for international travel from London's Heathrow in

2014. The Dubai International Airport (DXB) was opened for civil aviation in July 1937.

16)Name the Union Minister, who wll be conferred with the Carnot Prize.

Page 107: Monthly Current Affairs of SEPTEMBER - SSCE

SSCE 8981426494/8296260082 Downloaded from :- www.onlinessce.com Page 107

Ans:-Piyush Goyal

Explanation:-Union Minister Piyush Goyal received the Carnot prize for his work towards sustainable

energy solutions from University of Pennsylvania in New Delhi. This award rewards the works done by

Piyush Goyal to reform the electricity sector and extended access to electricity to 18,000 villages.

17)Who is the head of Lokpal search panel to select members of anti-corruption ombudsman Lokpal?

Ans:-Ranjana Prakash Desai

Explanation:- The 8-member Lokpal search panel hold its first meeting to select members of anti-

corruption ombudsman Lokpal. The committee, headed by former Supreme Court judge Ranjana Prakash

Desai, is understood to have discussed modalities related to the appointments of the chief and members of

Lokpal. The meeting comes days after the Supreme Court set up a February-end deadline for the search

committee to send a panel of names who could be considered for the appointment as chairman and members

of Lokpal by Prime Minister Narendra Modi-led selection committee. The Lokpal Act, which envisages

establishment of the anti-graft body Lokpal at the Centre and Lokayuktas in states to look into cases of

corruption against certain categories of public servants, was passed in 2013.

18)Who declared 2019 as the International Year of the Period Table of Chemical Elements?

Ans:-UNESCO

Explanation:-UNESCO has declared 2019 as the International Year of the Period Table of Chemical

Elements to commemorate the 150th birthday of the periodic table of chemical elements. The Periodic table

of the Chemical elements was first published by Russian scientist Dmitry Mendeleev in the year 1869.

19)Union Minister for Tourism K.J. Alphons, has inaugurated the first project under the Swadesh

Darshan Project in __________.

Ans:-Sikkim

Explanation:-K.J. Alphons, Union Minister for Tourism has inaugurated the first project under the Swadesh

Darshan Project of the Union Ministry of Tourism at the Zero Point, Gangtok, Sikkim. This project was

sanctioned by the Ministry of Tourism in June 2015 for Rs. 98.05 crores. Swadesh Darshan Scheme is one

of the flagship scheme of Ministry of tourism for development of thematic circuits in the country in a

planned manner. Under this scheme, the Government is focussing on the development of quality

infrastructure in the country with the objective of providing better experience and facilities to the visitors

and on fostering economic growth.

20)Which two countries have launched a common digital currency called "Aber"?

Ans:- UAE and Saudi Arabia

Explanation:-The central banks of the UAE and Saudi Arabia have launched a common digital currency

called "Aber", which will be used in financial settlements between the two countries through Blockchains

and Distributed Ledgers technologies. Through this digital currency, both the United Arab Emirates Central

Bank (UAECB) and the Saudi Arabian Monetary Authority (SAMA) are studying the impact on the

improvement and reduction of remittance costs and the assessments of risks.

Page 108: Monthly Current Affairs of SEPTEMBER - SSCE

SSCE 8981426494/8296260082 Downloaded from :- www.onlinessce.com Page 108

Like our face book page www.facebook.com/onlinessce

Visit us: www.onlinessce.com

Visit us: www.onlinessce.com

Write us:-

[email protected]

SEN’S SCHOOL OF COMPETIVE

EXAMINATIONS

One of the Best Institutes in Hooghly

Our New Batch will open on and from 11

Feb 2019,

• Call Now : 7003071953, 8981426494,

8296260082

Hindmotor Hooghly

Page 109: Monthly Current Affairs of SEPTEMBER - SSCE

SSCE 8981426494/8296260082 Downloaded from :- www.onlinessce.com Page 109